348 - 61275 - BA223 - 2019 - 1 - 1 - 1 - 2021 - Lecture Notes - Math 3

Download as pdf or txt
Download as pdf or txt
You are on page 1of 176

Arab Academy for Science, Technology &

Maritime Transport
Department of Basic & Applied Sciences

Mathematics 3
Lecture & Exercise Notes

Professor Nehad N. Morsi

2021
Contents

1 First-order Di¤erential Equations 1


1.1 Separable and Homogeneous Di¤erential Equations . . . . . . 1
1.1.1 Characterization and Methods of Solution . . . . . . . 1
1.1.2 Lecture Examples . . . . . . . . . . . . . . . . . . . . . 2
1.1.3 Classroom Exercises . . . . . . . . . . . . . . . . . . . 3
1.1.4 Revision Problems . . . . . . . . . . . . . . . . . . . . 5
1.2 Exact Di¤erential Equations . . . . . . . . . . . . . . . . . . . 7
1.2.1 Their Identi…cation and Method of Solution . . . . . . 7
1.2.2 Lecture Examples . . . . . . . . . . . . . . . . . . . . . 9
1.2.3 Classroom Exercises . . . . . . . . . . . . . . . . . . . 10
1.2.4 Revision Problems . . . . . . . . . . . . . . . . . . . . 12
1.3 Linear Di¤erential Equations . . . . . . . . . . . . . . . . . . . 14
1.3.1 Lecture Examples . . . . . . . . . . . . . . . . . . . . . 16
1.3.2 Classroom Exercises . . . . . . . . . . . . . . . . . . . 16
1.3.3 Revision Problems . . . . . . . . . . . . . . . . . . . . 17
1.4 Bernoulli Di¤erential Equations . . . . . . . . . . . . . . . . . 20
1.4.1 Lecture Examples . . . . . . . . . . . . . . . . . . . . . 20
1.4.2 Classroom Exercises . . . . . . . . . . . . . . . . . . . 21
1.4.3 Revision Problems . . . . . . . . . . . . . . . . . . . . 22
1.5 Further Revision Problems on Chapter 1 . . . . . . . . . . . . 25
1.6 Some Applications of First-order
Di¤erential Equations in Engineering . . . . . . . . . . . . . . 29

2 Second-order Linear Di¤erential Equations 30


2.1 Form of the General Solution . . . . . . . . . . . . . . . . . . 30
2.2 Order Reduction of Linear Di¤erential
Equations . . . . . . . . . . . . . . . . . . . . . . . . . . . . . 33
2.3 The Homogeneous Solutions yh (x) of
Linear Di¤erential Equations with
Constant Coe¢ cients . . . . . . . . . . . . . . . . . . . . . . . 34
2.4 Solution by the Method of Variation of Parameters . . . . . . 36

1
CONTENTS 2

2.4.1 Describing The Method . . . . . . . . . . . . . . . . . 36


2.4.2 Steps of the Method of Variation of Parameters: . . . . 38
2.4.3 Lecture Examples . . . . . . . . . . . . . . . . . . . . . 39
2.4.4 Classroom Exercises . . . . . . . . . . . . . . . . . . . 40
2.4.5 Revision Problems . . . . . . . . . . . . . . . . . . . . 44
2.5 Particular Solutions yp (x) of Equations with Constant Coef-
…cients - by the Method of Undetermined Coe¢ cients . . . . . 49
2.5.1 Lecture Examples . . . . . . . . . . . . . . . . . . . . . 50
2.5.2 Classroom Exercises . . . . . . . . . . . . . . . . . . . 52
2.5.3 Revision Problems . . . . . . . . . . . . . . . . . . . . 55
2.6 Solution of Cauchy-Euler Di¤erential
Equations . . . . . . . . . . . . . . . . . . . . . . . . . . . . . 61
2.6.1 Description and Method of Solution . . . . . . . . . . . 61
2.6.2 Lecture Examples . . . . . . . . . . . . . . . . . . . . . 63
2.6.3 Classroom Exercises . . . . . . . . . . . . . . . . . . . 63
2.6.4 Revision Problems . . . . . . . . . . . . . . . . . . . . 65

3 The Laplace Transform 68


3.1 De…nition and Properties . . . . . . . . . . . . . . . . . . . . . 68
3.2 Problems on Direct Laplace Transform . . . . . . . . . . . . . 72
3.2.1 Lecture Examples . . . . . . . . . . . . . . . . . . . . . 72
3.2.2 Classroom Exercises . . . . . . . . . . . . . . . . . . . 76
3.2.3 Revision Problems . . . . . . . . . . . . . . . . . . . . 79
3.3 Inverse Laplace Transform . . . . . . . . . . . . . . . . . . . . 84
3.3.1 Lecture Examples . . . . . . . . . . . . . . . . . . . . . 84
3.3.2 Classroom Exercises . . . . . . . . . . . . . . . . . . . 85
3.3.3 Revision Problems . . . . . . . . . . . . . . . . . . . . 87
3.3.4 Problems on the Inverse Form of the
Convolution Theorem . . . . . . . . . . . . . . . . . . . 91
3.4 Fredholm Integral Equations . . . . . . . . . . . . . . . . . . . 92
3.4.1 Lecture Examples . . . . . . . . . . . . . . . . . . . . . 92
3.4.2 Classroom Exercises . . . . . . . . . . . . . . . . . . . 92
3.4.3 Revision Problems . . . . . . . . . . . . . . . . . . . . 93
3.5 Laplace Transform and Initial-value
Problems: . . . . . . . . . . . . . . . . . . . . . . . . . . . . . 96
3.5.1 Lecture Examples . . . . . . . . . . . . . . . . . . . . . 96
3.5.2 Classroom Exercises . . . . . . . . . . . . . . . . . . . 97
3.5.3 Revision Problems . . . . . . . . . . . . . . . . . . . . 98
3.6 Laplace Transform and Systems of
Di¤erential Equations . . . . . . . . . . . . . . . . . . . . . . . 102
3.6.1 Lecture Examples . . . . . . . . . . . . . . . . . . . . . 102
CONTENTS 3

3.6.2 Classroom Exercises . . . . . . . . . . . . . . . . . . . 102


3.6.3 Revision Problems . . . . . . . . . . . . . . . . . . . . 104

4 The Fourier Series 109


4.1 Basics of the Fourier Series . . . . . . . . . . . . . . . . . . . . 109
4.2 Classroom Exercises on Fourier Series . . . . . . . . . . . . . . 111

A Table of Famous Integrals 115

B Assignments 116
B.1 Assignment 1 on Separable and
Homogeneous Di¤erential Equations . . . . . . . . . . . . . . 117
B.2 Assignment 2 on Exact Di¤erential
Equations . . . . . . . . . . . . . . . . . . . . . . . . . . . . . 122
B.3 Assignment 3 on Linear Di¤erential
Equations . . . . . . . . . . . . . . . . . . . . . . . . . . . . . 127
B.4 Assignment 4 on Bernoulli Di¤erential Equations . . . . . . . 132
B.5 Assignment 5 on Method of Variation of Parameters . . . . . . 137
B.6 Assignment 6 on Method of Undetermined Coe¢ cients . . . . 142
B.7 Assignment 7 on Cauchy-Euler
Di¤erential Equations . . . . . . . . . . . . . . . . . . . . . . . 148
B.8 Assignment 8 on Direct Laplace
Transform . . . . . . . . . . . . . . . . . . . . . . . . . . . . . 152
B.9 Assignment 9 on Inverse Laplace
Transform . . . . . . . . . . . . . . . . . . . . . . . . . . . . . 154
B.10 Assignment 10 on Solution of Fredholm Integral Equations . . 158
B.11 Assignment 11 on Laplace Transform and Initial-value Prob-
lems (1) . . . . . . . . . . . . . . . . . . . . . . . . . . . . . . 163
B.12 Assignment 12 on Laplace Transform and Initial-value Prob-
lems (2) . . . . . . . . . . . . . . . . . . . . . . . . . . . . . . 168
Chapter 1

First-order Di¤erential
Equations

1.1 Separable and Homogeneous Di¤erential


Equations
1.1.1 Characterization and Methods of Solution
A …rst-order di¤erential equation (in an unknown function y (x)) takes the
form
dy
= f (x; y (x)) : (1.1)
dx
It is said to be Separable if it takes the form :
dy A (x)
= , (1.2)
dx B (y (x))
dy
which is rewritten as B (y (x)) dx = A (x). To solve this separable D.E., we
R dy
R
integrate both sides, and get B (y (x)) dx dx = A (x) dx+C. When we use
R dy
R
the substitution y = y (x), we …nd that B (y (x)) dx dx = B (y) dy y=y(x) .
Therefore, the general solution of (1.2) is implicit in the equation
Z Z
B (y) dy = A (x) dx + C. (1.3)
y=y(x)

A …rst-order di¤erential equation is said to be Homogeneous if it can


be put in the following form :
dy y (x)
=f .
dx x

1
CHAPTER 1. FIRST-ORDER DIFFERENTIAL EQUATIONS 2

The substitution u = xy (and so, dx dy


= u + x du
dx
) turns a homogeneous D.E.
into the following separable one (in the unknown function u (x)):

u + x du
dx
= f (u); that is,

du
x = f (u) u:
dx

Remark 1.1.1 Throughout these Lecture Notes, we shall be seeking only


those solutions that are real analytic functions.
For instance, consider the following …rst-order separable (also,
homogeneous and linear) di¤erential equation:
dy
x = 3y:
dx
All di¤ erentiable solutions of this equation are given by

C1 x3 ; x 0
y (x) = ;
C2 x3 ; x<0

for all choices of the two arbitrary constants C1 and C2 .


However, such a function becomes analytic if and only if C1 = C2 .
Therefore, we limit our solutions for this di¤erential equation to the following
set of analytic functions only:

y (x) = C1 x3 :

1.1.2 Lecture Examples


Solve the following …rst-order di¤erential equations:
2
Problem 1.1.1 (4y + yx2 ) y 0 = (y 2 + 25) :

dy 2 R ydy
R dx
y (4 + x2 ) dx = (y 2 + 25) Separable D.E. (y 2 +25)2
= 4+x2
R R dx
) 2y (y 2 + 25) dy = 2 4+x
2
2

) ) y 2 + 25 =
1
(y 2 + 25) = 22 tan 1 x
C 1
.
2 C tan 1
( x2 )
CHAPTER 1. FIRST-ORDER DIFFERENTIAL EQUATIONS 3

dy
Problem 1.1.2 (4xy 3 + 2x3 y) dx = 3y 4 + x2 y 2 + 5x4 :
y 3 y 4 y 2
Divide by x4 : ) 4 x
+2 y
x
dy
dx
=3 x
+ x
+5
Homogeneous D.E., Put u = y
x
.) y = xu ) dy
dx
= u + x du
dx
) (4u3 + 2u) u + x du
dx
= 3u4 + u2 + 5
) 4u4 + 2u2 + (4u3 + 2u) x du
dx
= 3u4 + u2 + 5
) (4u + 2u) x dx = (u + u
3 du 4 2
5) Separable D.E. in u (x).
R 4u3 +2u R dx
) u4 +u2 5 du = x
) ln (u + u2 5) = ln (C1 x) = ln
4 C
x
y4 2
) u4 + u2 5= C
x
) x4
+ xy 2 5= C
x
) y 4 + x2 y 2 5x4 = Cx3 .

1.1.3 Classroom Exercises


dy
p
Problem 1.1.3 cos2 x dx = x 1 y2:
R R xdx R
Separable D.E. ) pdy 2 = cos 2x = x sec2 x dx
1 y
R
) sin 1 y = x tan x tan xdx = x tan x + ln (cos x) + C
) y = sin (x tan x + ln (cos x) + C) .

dy
Problem 1.1.4 dx
5x4 = 5x4 y 2 ; y (0) = 1:
R dy R 4
dy
dx
= 5x4 (1 + y 2 ) Separable D.E. ) 1+y 2 = 5x dx
) tan y = C + x ; i.e. y = tan (C + x ) :
1 5 5

At x = 0; y = 1: ) tan 1 (1) = C + 0 ) 4 =C
) y = tan 4
+ x5 .

2y dy 2y 2y
Problem 1.1.5 2x3 cos x
+ 15xy 2 dx
= 2x2 y cos x
+x3 sin x
+20y 3 :
y 2
Divide by x3 : ) 2 cos 2y
x
+ 15 x
dy
dx
= 2 xy cos 2y
x
+ sin 2y
x
+
y 3
20 x
)Homogeneous D.E. Put u = y
x
. ) y = xu ) dy
dx
= u + x du
dx
) (2 cos (2u) + 15u2 ) u + x du
dx
= 2u cos (2u) + sin (2u) + 20u3
) (2 cos (2u) + 15u ) x dx = sin (2u) + 5u3
2 du
Separable D.E. in u (x)
R 2 cos(2u)+15u2 R dx
) sin(2u)+5u3
du = x ) ln (sin (2u) + 5u3 ) = ln (Cx)
y 3
) sin (2u)+5u3 = Cx ) sin 2y
x
+5 x
= Cx ) x3 sin 2y
x
+ 5y 3 = Cx4 .
CHAPTER 1. FIRST-ORDER DIFFERENTIAL EQUATIONS 4

dy
Problem 1.1.6 (x7 8x4 y 3 + 7xy 6 ) dx = 4y 7 2x3 y 4 2x6 y; y (1) =
1:

y 3 y 6 dy y 7 y 4 y
1 8 x
+7 x dx
=4 x
2 x
2 x
: Homogeneous
D.E.
Put u = y
x
. ) y = xu ) dy
dx
= u + x du
dx
) (1 8u3 + 7u6 ) u + x du
dx
= 4u7 2u4 2u
) (1 8u3 + 7u6 ) x du
dx
+u 8u4 + 7u7 = 4u7 2u4 2u
) (1 8u3 + 7u6 ) x dudx
= 3u7 + 6u4 3u: Separable D.E. in u (x).
R R
) 1u7 8u2u+7u
3 6
4 +u du = 3 dx x

) ln (u7 2u4 + u) = 3 ln (Cx) = ln (C 3


x 3)
y7 4
) u7 2u4 + u = C 3
x 3
) x7
2 xy 4 + y
x
=C 3
x 3

) y7 2x3 y 4 + x6 y = Kx4 :
At x = 1, y = 1, )K= 1 2 1= 4:
) y7 2x3 y 4 + x6 y = 4x4 :

p q
dy y+ x2 y 2 y y 2
Problem 1.1.7 dx
= x
= x
+ 1 x
; y (1) = 0

Homogeneous D.E. Put u = xy ) y = xu ) dx


dy
= u + x du
dx
p p
) Ru + x dx = Ru + 1 u
du 2 ) x dx = 1 u Separable D.E.
du 2

) p du
1 u2
= x dx
) sin u = ln Cx
1

) x = u = sin (ln Cx) :


y
At x = 1, y = 0. ) 0 = sin (ln C)
) 0 = ln C )C=1 ) y = x sin (ln x) :

dy y
Problem 1.1.8 x dx = y + x cot x
:

) dy
dx
= y
x
+ cot y
x
Homogeneous D.E. Put u = y
x
.
) y = xu ) dx
dy
= u + x du
dx
) u + x du
dx
= u + cot u
)x du
R dx = cot u R Separable D.E. in u (x).
) tan u du = dx x
) ln (sec u) = ln (Cx)
) sec u = Cx ) y
x
= u = sec 1
(Cx) ) y = x sec 1
(Cx) .
CHAPTER 1. FIRST-ORDER DIFFERENTIAL EQUATIONS 5

1.1.4 Revision Problems


dy
Problem 1.1.9 3x2 dx = 9x2 + 3xy + y 2 :
y
Homogeneous D.E. Put u = x
) y = xu ) dx
dy
= u + x du
dx
) 3x2 u + x dudx
= 9x2 + 3x2 u + x2 u2
)u = 3 + u + u3 ) x du
2 2 2
x du
R +3du dx R dx
= 3 + u3 = 9+u
3
Separable D.E. in u (x)
) 9+u2 = dx x
) tan 1 u3 = ln (Cx)
) y
3x
= u
3
= tan (ln (Cx)) : ) y = 3x tan (ln (Cx)) .

dy 2y 2 +7xy+5x2
Problem 1.1.10 dx
= xy+x2
; y (1) = 3:
y 2
2( x ) +7 xy +5
) dy
dx
= y
+1
: Homogeneous D.E.
x

Put u = xy . ) y = xu ) dx
dy
= u + x du
dx
) u + x dx = u+1
du 2u2 +7u+5

) x du
2 +7u+5 2
= 2u u+1 u = u +6u+5 = (u+1)(u+5) =u +5
R dxdu R dx u+1 u+1
) u+5 = x ) ln (u + 5) = ln (Cx) :
) x + 5 = u + 5 = Cx:
y
that is, y = Cx2 5x .
At x = 1; y = 3: ) C = 8: 2
) y = 8x 5x .

dy
Problem 1.1.11 (5xy 4 3x3 y 2 + 2x5 ) dx = 2y 5 4x4 y:

y 4 y 2 dy y 5
Divide by x5 : 5 x
3 x
+2 dx
=2 x
4 xy :

Homogeneous D.E. Put u = y


x
. ) dy
dx
= dxu
dx
= u + x du
dx

) (5u4 3u2 + 2) u + x du
dx
= 2u5 4u
) 5u 5 3
3u + 2u + (5u 4
3u + 2) x du
2
dx
= 2u5 4u
) (5u4 3u2 + 2) x du
dx
= 3u5 + 3u3 6u = 3 (u5 u3 + 2u)
(5u4 3u2 +2)du
Separable D.E. in u (x). ) u5 u3 +2u = 3dx
x
R 4 3u2 +2 R dx
) 5u5 3
u u +2u
du = 3 x
= 3 ln Cx
) ln (u5 u3 + 2u) = ln C1
x3
y5 y3
) u5 u3 + 2u = C1
x3
: ) x5
+ 2 xy = Cx31 :
x3
Multiply by x5 : ) y5 x y + 2x4 y = C1 x2 :
2 3
CHAPTER 1. FIRST-ORDER DIFFERENTIAL EQUATIONS 6

dy
Problem 1.1.12 (x2 2x + 1) dx 2y = 0:
Final Answer: y = Ce2=1 x

Problem 1.1.13 (cos2 x cos y) y 0 + x sin y = 0:


x tan x
Final Answer: sin y = C sec xe

Problem 1.1.14 (x2 + 9) y 0 + xy = 0:


Final Answer: y= pC
x2 +9

Problem 1.1.15 (x2 + x2 y) y 0 + (1 x) y 2 = 0:


1 1
Final Answer: ln y y x
ln x = C

Problem 1.1.16 y 0 3x2 (1 + y 2 ) = 0:


Final Answer: y = tan (x3 + C)

6y
Problem 1.1.17 2 sec 2xdy e dx = 0:
Final Answer: 2e6y = 3 sin 2x + C

y dy ex
Problem 1.1.18 x dx
= ln y
:

Final Answer: y 2 (1 2 ln y) = 4ex (1 x) + C

y
Problem 1.1.19 xy 0 = y + x cot x
:
1
Final Answer: y = x sec (Cx)

dy
Problem 1.1.20 x2 + 3y 2 2xy dx = 0:
p
Final Answer: y = x Cx 1

p dy
p
Problem 1.1.21 xy x2 + y 2 dx = x3 + y 2 x2 + y 2 :
2=3
Final Answer: y 2 = x2 [ln (Cx3 )] x2
CHAPTER 1. FIRST-ORDER DIFFERENTIAL EQUATIONS 7

1.2 Exact Di¤erential Equations


1.2.1 Their Identi…cation and Method of Solution
All …rst-order di¤erential equations, in an unknown function y (x) ; can be
written as follows:
dy
M (x; y (x)) + N (x; y (x)) = 0; (1.4)
dx
In this section, we often simplify this writing as follows:

M (x; y) dx + N (x; y) dy = 0, (1.5)

Equation (1.4) is said to be Exact if it satis…es the following exactness


condition:
@M (x; y) @N (x; y)
= . (1.6)
@y @x
The solution of an exact di¤erential equation (1.4) consists of executing
the following three steps:
Step 1: Test the exactness condition (1.6). If it is valid for the given
equation, proceed to the next two steps.
Step 2: We recall the following theorem of Di¤erential Calculus:
Theorem 1 Two continuously di¤erentiable functions M (x; y) and N (x; y)
satisfy the exactness condition (1.6), in some region of the xy-plane, if and
only if a function (x; y) exists in that region, such that:

@ (x; y) @ (x; y)
= M (x; y) and = N (x; y) : (1.7)
@x @y
That function (x; y) is one and only one (up to additive constants).

Proof. First, suppose that M and N satisfy the exactness condition (1.6).
Choose a point (x ; y ) in their common region of de…nition. De…ne a function
(x; y) as follows:
Z x Z y
(x; y) = M (t; y) dt + N (x ; u) du: (1.8)
x y

Then we have @ @x (x;y)


= M (x; y) + 0; and
@ (x;y) R x @M (t;y) R x @N (t;y)
@y
= x @y
dt + N (x ; y) = x @t
dt + N (x ; y) (by (1.6)),
= N (x; y) N (x ; y) + N (x ; y) = N (x; y) ; as required.
CHAPTER 1. FIRST-ORDER DIFFERENTIAL EQUATIONS 8

Conversely, we suppose the existence of a function (x; y) that satis…es


the two identities in (1.7). Then we …nd that
@M (x;y) @ (x;y) 2 (x;y) 2 (x;y)

@y
@
= @y @x
= @ @y@x = @ @x@y @
= @x @ (x;y)
@y
= @N@x
(x;y)
;
that is, M and N must satisfy the exactness condition (1.6). This com-
pletes the proof.
Our task in Step 2 is to determine that function (x; y). We can do that
through computing the following two integrals, then comparing them with
each other:
Z
(x; y) = M (x; y) dx + A (y) ;
Z
(x; y) = N (x; y) dy + B (x) :

(Alternatively, you can compute from formula (1.8). But this could be a
bit more laborious!)
Step 3: We recall the following basic theorem of Di¤erential Calculus:.
Theorem 2 (Chain Rule in Several Variables) Let z (x), y (x) and (z; y)
be continuously di¤erentiable functions around points x and (z (x) ; y (x)), re-
spectively. Then at that point x we have:
d (z (x) ; y (x)) @ (z; y) dz @ (z; y) dy
= + :
dx @z jat (z(x);y(x)) dx @y jat (z(x);y(x))
dx
dz
In the special case that z (x) = x; we have dx
= 1. Therefore, the Chain
Rule becomes:
d (x; y (x)) @ (x; y) @ (x; y) dy
= + : (1.9)
dx @x jat (x;y(x)) @y jat (x;y(x))
dx

When we apply formula (1.9) to the particular function (x; y) (obtained


in Step 2) we can bene…t from the two identities in (1.7), and we come out
with:
d (x; y (x)) dy
= M (x; y (x)) + N (x; y (x)) :
dx dx
We combine this with our di¤erential equation (1.4), to …nd that
d (x; y (x))
= 0:
dx
CHAPTER 1. FIRST-ORDER DIFFERENTIAL EQUATIONS 9

But, the only functions of one variable whose derivatives are identically 0 are
the constant functions. We conclude that the general solution y (x) of the
di¤erential equation (1.4) is implicit in the equation:
(x; y (x)) = C; (1.10)
for each value of the arbitrary constant C.

1.2.2 Lecture Examples


4
Problem 1.2.1 (y 3 y 2 sin x + 3e3x ) dx + 3xy 2 + 2y cos x + y
dy = 0:

@M (x;y) @ (y 3 y 2 sin x+3e3x )


M dx + N dy = 0 @y
= @y
= 3y 2 2y sin x;
@N (x;y) @ (3xy 2 +2y cos x+ y4 )
@x
= @x
= 3y 2 2y sin x:
) @M@y (x;y)
= @N@x (x;y)
: ) Exact D.E.
R R
(x; y) = M (x; y) dx + A (y) = (y 3 y 2 sin x + 3e3x ) dx + A (y)
= xy 3 + y 2 cos x + e3x + A (y) ;
R R
(x; y) = N (x; y) dy + B (x) = 3xy 2 + 2y cos x + y4 dy + B (x)
= xy 3 + y 2 cos x + 4 ln y + B (x) :
) (x; y (x)) = xy 3 + y 2 cos x + e3x + 4 ln y = C :

Problem 1.2.2 (9 cos x + y 2 cos x) dx + 2y sin x dy = 0; y 2


= 4:

M (x; y) dx + N (x; y) dy = 0
@M (x;y) @ (9 cos x+y 2 cos x)
@y
= @y
= 2y cos x;
@N (x;y)
= @2y@xsin x = 2y cos x. ) @M@y(x;y)
= @N@x
(x;y)
: ) Exact D.E.
@x R R
(x; y) = M (x; y) dx+A (y) = (9 + y ) cos x dx+A (y) = (9 + y 2 ) sin x+
2

A (y) ; R R
(x; y) = N (x; y) dy + B (x) = 2y sin x dy + B (x) = y 2 sin x + B (x) :
p
(x; y (x)) = (9 + y 2 ) sin x = C: So by initial condition y = 25 csc x 9 .
2
Another
R cos Solution:
R 2ySeparable D.E. (9 + y ) cos x dx = 2y sin x dy
) x
sin x
dx = 9+y2 dy
) ln (C sin x) = ln (9 + y 2 ) ) 9 + y 2 = C sin
1
x
:
p
At x = 2 , y = 4. ) C = 1
25
and y = 25 csc x 9 .
CHAPTER 1. FIRST-ORDER DIFFERENTIAL EQUATIONS 10

1.2.3 Classroom Exercises


Problem 1.2.3 Solve:
x4 dy
4y
4 sin x sin3 y cos y + sec y tan y dx
= x3 ln y + cos x sin4 y:

4
x3 ln y cos x sin4 y dx + x4y 4 sin x sin3 y cos y + sec y tan y dy = 0:
M (x; y) dx + N (x; y) dy = 0
@M (x;y) @ (x3 ln y cos x sin4 y ) 3
@y
= @y
= xy 4 cos x sin3 y cos y;
4
@N (x;y) @ x4y 4 sin x sin3 y cos y+sec y tan y 3
@x
= @x
= xy 4 cos x sin3 y cos y + 0
) @M@y (x;y)
= @N@x(x;y)
: ) Exact D.E.
R R 3
(x; y) = M (x; y) dx + A (y) = x ln y cos x sin4 y dx + A (y)
4
= x4 ln y sin x sin4 y + A (y) ;
R R x4
(x; y) = N (x; y) dy+B (x) = 4y
4 sin x sin3 y cos y + sec y tan y dy+
B (x)
x4
= 4
ln y sin x sin4 y + sec y + B (x) :

) (x; y) = x4
4
ln y sin x sin4 y + sec y = C :

dy
Problem 1.2.4 (x3 sec2 y + x sec y tan y 2e2y ) dx = (3x2 tan y + cos x + sec y) :

) (3x2 tan y + cos x + sec y) dx + (x3 sec2 y + x sec y tan y 2e2y ) dy = 0:


M (x; y) dx + N (x; y) dy = 0
@M (x;y) @ (3x2 tan y+cos x+sec y )
@y
= @y
= 3x2 sec2 y + 0 + sec y tan y
@N (x;y) @ (x3 sec2 y+x sec y tan y 2e2y )
@x
= @x
= 3x2 sec2 y + sec y tan y 0:
) @M (x;y)
@y
= @N@x
(x;y)
: ) Exact D.E.
R R
(x; y) = M (x; y) dx = (3x2 tan y + cos x + sec y) dx + A (y)
= x3 tan y + sin x + x sec y + A (y)
R R
(x; y) = N (x; y) dy = (x3 sec2 y + x sec y tan y 2e2y ) dy + B (x)
= x3 tan y + x sec y e2y + B (x)
(x; y (x)) = x3 tan y + sin x + x sec y e2y = C .
(by taking A (y) = e2y and B (x) = sin x)
CHAPTER 1. FIRST-ORDER DIFFERENTIAL EQUATIONS 11

Problem 1.2.5 (cos x y sin x) dx + (2y + cos x) dy = 0; y (0) = 2


M (x; y) dx + N (x; y) dy = 0
@M (x;y)
@y
= @(cos x@yy sin x) = sin x; @N (x;y)
@x
= @(2y+cos x)
@x
= sin x:
) @M (x;y)
@y
= @N (x;y)
@x
: ) Exact D.E.
R R
(x; y) = M (x; y) dx + A (y) = (cos x y sin x) dx + A (y)
= sin x + y cos x + A (y) ;
R R
(x; y) = N (x; y) dy + B (x) = (2y + cos x) dy + B (x)
= y 2 + y cos x + B (x) :
) (x; y (x)) = sin x + y cos x + y 2 = C:
At x = 0; y = 2: ) sin 0 + 2 cos 0 + 4 = C. ) C = 6
) sin x + y cos x + y = 6 .
2

3
Problem 1.2.6 (2xey + cos x) dx + x2 ey y
dy = 0:
M (x; y) dx + N (x; y) dy = 0
3
@M (x;y) y @ (x2 ey )
@y
= @(2xe @y+cos x) = 2xey ; @N (x;y)
@x
= @x
y
= 2xey :
) @M@y(x;y)
= @N@x
(x;y)
: ) Exact D.E.
R R
(x; y) = M (x; y) dx + A (y) = (2xey + cos x) dx + A (y)
= x2 ey + sin x + A (y) ;
R R
(x; y) = N (x; y) dy + B (x) = x2 ey y3 dy + B (x)
= x2 ey 3 ln y + B (x) :
) (x; y (x)) = x2 ey + sin x 3 ln y = C:

x4 dy
Problem 1.2.7 2e2x sin y + 4x3 tan 1
y + e2x cos y + 1+y 2
+ sec2 y dx
= 0:
M (x; y) dx + N (x; y) dy = 0
@M (x;y) @ (2e2x sin y+4x3 tan 1 y ) 4x3
@y
= @y
= 2e2x cos y + 1+y 2
;
4
@ e2x cos y+ x 2 +sec2 y
@N (x;y) 4x3
@x
= @x
1+y
= 2e2x cos y + 1+y 2
:
) @M@y (x;y)
= @N@x(x;y)
: ) Exact D.E.
R R
(x; y) = M (x; y) dx + A (y) = (2e sin y + 4x3 tan 1 y) dx + A (y)
2x

= e2x sin y + x4 tan 1 y + A (y) ;


R R 2x x4 2
(x; y) = N (x; y) dy + B (x) = e cos y + 1+y 2 + sec y dy + B (x)
= e2x sin y + x4 tan 1 y + tan y + B (x) :
) (x; y (x)) = e2x sin y + x4 tan 1 y + tan y = C .
CHAPTER 1. FIRST-ORDER DIFFERENTIAL EQUATIONS 12

1.2.4 Revision Problems


dy
Problem 1.2.8 (x5 sec2 y + 5y 4 ) dx = 5x4 tan y sec2 x:

) (5x4 tan y + sec2 x) dx + (x5 sec2 y + 5y 4 ) dy = 0:


M (x; y) dx + N (x; y) dy = 0
@M (x;y) @ (5x4 tan y+sec2 x)
@y
= @y
= 5x4 sec2 y;
@N (x;y) @ (x5 sec2 y+5y 4 )
@x
= @x
= 5x4 sec2 y:
) @y = @x :
@M (x;y) @N (x;y)
) Exact D.E.
R R
(x; y) = M (x; y) dx + A (y) = (5x4 tan y + sec2 x) dx + A (y)
= x5 tan
R y + tan x + A (y) ; R
(x; y) = N (x; y) dy + B (x) = (x5 sec2 y + 5y 4 ) dy + B (x)
= x5 tan y + y 5 + B (x) :
) (x; y (x)) = x5 tan y + tan x + y 5 = C:

1
Problem 1.2.9 (sec2 x + yexy ) dx + y
+ xexy dy = 0; y (0) = 1

M (x; y) dx + N (x; y) dy = 0
@M (x;y) @ (sec2 x+yexy )
@y
= @y
= exy + xyexy ;
@N (x;y) @ ( y1 +xexy )
@x
= @x
= exy + xyexy :
) @y = @x :
@M (x;y) @N (x;y)
) Exact D.E.
R R
(x; y) = M (x; y) dx + A (y) = (sec2 x + yexy ) dx + A (y)
= tan x + exy + A (y) ;
R R 1
(x; y) = N (x; y) dy + B (x) = y
+ xexy dy + B (x)
= ln y + exy + B (x) :
) (x; y (x)) = tan x + exy + ln y = C:

At x = 0; y = 1: ) 0 + 1 + 0 = C: ) tan x + exy + ln y = 1 .
CHAPTER 1. FIRST-ORDER DIFFERENTIAL EQUATIONS 13

2 dy
Problem 1.2.10 4y 3 tan x 5 xy + 6e2y dx
= y 4 sec2 x + 10x ln y:

2
4y 3 tan x 5 xy + 6e2y dy + (y 4 sec2 x 10x ln y) dx = 0
N (x; y) dy + M (x; y) dx = 0
@M (x;y) @ (y 4 sec2 x 10x ln y )
@y
= @y
= 4y 3 sec2 x 10 xy
2
@N (x;y) @ 4y 3 tan x 5 xy +6e2y
@x
= @x
= 4y 3 sec2 x 10 xy + 0:

) @M (x;y)
@y
= @N@x
(x;y)
: ) Exact D.E.
R R
(x; y) = M (x; y) dx = (y 4 sec2 x 10x ln y) dx + A (y)
= y 4 tan x 5x2 ln y + A (y)
R R 2
(x; y) = N (x; y) dy = 4y 3 tan x 5 xy + 6e2y dy + B (x)

= y 4 tan x 5x2 ln y + 3e2y + B (x)


(x; y (x)) = y 4 tan x 5x2 ln y + 3e2y = C .
(by taking A (y) = 3e2y and B (x) = 0)

Problem 1.2.11 (2yx2 + 4) y 0 + (2y 2 x 6) = 0:


Final Answer: Exact D.E. y 2 x2 + 4y 6x = C
CHAPTER 1. FIRST-ORDER DIFFERENTIAL EQUATIONS 14

1.3 Linear Di¤erential Equations


A …rst-order di¤erential equation is said to be Linear if it takes the form :
dy
+ p (x) y = q (x) : (1.11)
dx
The integrating factor of this equation is the following function:
R
p(x) dx
(x) = e . (1.12)

Theorem 3 The general solution of the linear di¤erential equation (1.11) is


given by Z
1
yG:S: (x) = C+ (x) q (x) dx ; (1.13)
(x)
where C denotes an arbitrary constant.
d d
R
p(x) dx
R
p(x) dx d
R
Proof. First, we have dx
= dx
e =e : dx
p (x) dx :

d
) = p: (1.14)
dx
So, given any di¤erentiable function y (x), we …nd that
dy
d
dx
( y) = dx + ddx y = dx
dy
+ py = dy
dx
+ py :
Therefore, a function y (x) becomes a solution of the di¤erential equation
dy d
dx
+ py = q if and only if dx ( y) = q:
When we integrate both sides w.r.t. x, we obtain the following formula
for all solutions:
R
y=C+ q dx; as claimed in (1.13).

Another, longer, derivation of the solution (1.13):


Proof. Equation (1.11) is not exact, unless p (x) = 0. But, when we multiply
both sides of (1.11) by (x), we obtain the following di¤erential equation:

dy
(x) + (x) p (x) y (x) q (x) = 0:
dx
In the form N (x; y (x)) dy + M (x; y (x)) dx = 0; this equation becomes

(x) dy + [ (x) p (x) y (x) q (x)] dx = 0: (1.15)

We show that this di¤erential equation (1.15) is exact, with:


M (x; y) = (x) p (x) y (x) q (x) and N (x; y) = (x).
CHAPTER 1. FIRST-ORDER DIFFERENTIAL EQUATIONS 15

@M (x;y) @
@y
= @y
( (x) p (x) y (x) q (x)) = p:
@N (x;y)
Also, from (1.14), ddx = p. Therefore, @x
= d
dx
= p:
So, @M@y
(x;y)
= @M@y
(x;y)
, ensuring that equation (1.15) is exact.
Next, we obtain the function (x; y) such that
@ (x;y)
@x
= M (x; y) and @ @y (x;y)
= N (x; y).
R R
R (x;
d
y) = R M (x; y) dx = ( (x) p (x) y (x) q (x)) dx
= y dx dx (x) Rq (x) dx
) (x; y) = y R (x) q (x) dxR+ A (y)
Also, (x; y) = N (x; y) dy = (x) dy = y + B (x) :
By comparing these two Rbits of information on the function ,
we have to take B (x) = R (x) q (x) dx and A (y) = 0:
Thus (x; y) = y (x) q (x) dx.
Then the solution of the exactR equation (1.15) is given by
(x; y (x)) = (x) y (x) q (x) dx = C.
This leads immediately to the formula (1.13).

Remark 1.3.1 The solution of a …rst-order linear di¤erential equation:


y 0 + p (x) y = q (x) (1.16)
always takes the form:
yG:S: = Cy1 + yp (1.17)
R
1 p(x) dx
whereby C is an arbitrary constant. The function y1 (x) = =e (x)
is a nonzero solution of the corresponding di¤erential equation:
y 0 + p (x) y = 0 (1.18)
C
(which is both linear and separable). The functions Cy1 = are all the
solutions of equation
R (1.18).
(x)q(x) dx
Also, yp (x) = (x)
is one of the solutions of equation (1.16) itself.
It is easy to see that this function y1 can be replaced, in the form (1.17), by
any other nonzero solution of equation (1.18). Likewise, the function yp can
be replaced in (1.17) by any other solution of equation (1.16).
Remark 1.3.2 In di¤erential equations, this form of the general solution
yG:S: = Cy1 + yp is valid for …rst-order linear di¤erential equations only.
That equation has to be the following one (in the unknown function y):
y0 y yp0 yp
= :
y10 y1 y10 y1
CHAPTER 1. FIRST-ORDER DIFFERENTIAL EQUATIONS 16

1.3.1 Lecture Examples


dy 5x
Problem 1.3.1 dx
+ (tan x) y = 10e cos x; y (0) = 0:

dy
Linear D.E., dx
+ p (x) y = q (x) :
5x
R R
p = tan x; q = 10e cos x; p dx = tan x dx = ln (sec x) :
R
p dx
Integrating factor: =e = eln(sec x) = sec x; q = 10e 5x
:
R R
)y= 1 C+ q dx = cos x C + 10e 5x dx = cos x (C 2e 5x
):
At x = 0, y = 0, )0=C 2 ) C = 2:
5x
) y = 2 cos x (1 e ):

dy 1 2
Problem 1.3.2 dx
+ x
+ sec x y = x
cos x:
dy
Linear D.E., dx
+ p (x) y = q (x) :
1 2
p= x
+ sec x q= x
cos x:
R R 1
p dx = x
+ sec x dx = ln x+ln (sec x + tan x) = ln (x (sec x + tan x)) :
R
p dx
Integrating factor: =e = eln(x(sec x+tan x)) :
) = x (sec x + tan x) = x(1+sin
cos x
x)
:
R R x(1+sin x) R
q dx = 2 x cos x
cos x dx = (2 + 2 sin x) dx = 2x 2 cos x:
R
)y= 1 C+ cos x
q dx = x(1+sin x)
(C + 2x 2 cos x) :
)y= 1 sin x
x
(C sec x + 2x sec x 2) :

1.3.2 Classroom Exercises


dy
Problem 1.3.3 dx
(tan x) y = 4 sin x:

dy
Linear D.E., dx
+ p (x) y = q (x) :
p= tan x q = 4 sin x:
R R
p dx tan x dx
Integrating factor: =e =e = eln(cos x) = cos x:
R R
)y= 1 C+ q dx = cos1 x C + 4 sin x cos xdx
2
) y = sec x C + 2 sin x
Also, y = sec x (C1 2 cos2 x) ) y = C1 sec x 2 cos x:
CHAPTER 1. FIRST-ORDER DIFFERENTIAL EQUATIONS 17

dy
Problem 1.3.4 dx
+ (1 + tan x) y = cos x:

dy
Linear D.E., dx
+ p (x) y = q (x) :
p = 1 + tan x q = cos x:
R R
p dx = (1 + tan x) dx = x + ln (sec x) :
R
Integrating factor: = e p dx = ex+ln(sec x) = ex eln(sec x) = ex sec x:
R R R
q dx = ex sec x cos x dx = ex dx = ex :
R
)y= 1 C+ 1
q dx = ex sec x
(C + ex )

) y = Ce x
cos x + cos x :

dy
Problem 1.3.5 dx
+ (2 3 cot 3x) y = 8 sin 3x:

dy
Linear D.E., dx
+ p (x) y = q (x) :
p = 2 3 cot 3x q = 8 sin 3x:
R R
p dx = (2 3 cot 3x) dx = 2x ln (sin 3x) :
R
e2x e2x
Integrating factor: = e p dx = e2x ln(sin 3x) = eln(sin 3x)
= sin 3x
:
R R e2x R
q dx = 8 sin 3x
sin 3x dx = 8 e2x dx = 4e2x :
R
)y= 1 C+ q dx = e 2x sin 3x (C + 4e2x )

) y = Ce 2x
sin 3x + 4 sin 3x :

1.3.3 Revision Problems


dy
Problem 1.3.6 x2 dx + 3xy = 3ex :
dy 3ex 3ex
Linear D.E. dx
+ x3 y = x2
:, p= 3
x
q= x2
R R 3 3
p dx dx
Integrating factor: =e =e x = e3 ln x = eln x = x3 :
C+
R
q dx C 3
R
x ex dx 1
R x
y= = x3
= x3
C 3 x ex e dx

)y= 1
x3
(C 3 (x ex ex )) : )y= 1
x3
(C 3x ex + 3ex ) :
CHAPTER 1. FIRST-ORDER DIFFERENTIAL EQUATIONS 18

dy
Problem 1.3.7 dx
+ 3 (tan x) y = ex cos3 x; y (0) = 5:

dy
Linear D.E., dx
+ p (x) y = q (x) :
p = 3 tan x q = ex cos3 x:
R R
p dx 3 tan x dx
Integrating factor: =e =e = e3 ln(sec x) = sec3 x:
R R
)y= 1 C+ q dx = sec13 x C + sec3 x ex cos3 xdx
R x 3
) y = cos3 x C + ex dx ) y = (C + e ) cos x:
At x = 0, y = 5, ) 5 = 1 (C + 1) ) C = 4:
x 3
)y = (4 + e ) cos x.

dy 2x 2
Problem 1.3.8 dx
+ x2 16
y = x 4
:

dy
Linear D.E., dx
+ p (x) y = q (x) :
p = x22x16 q = x2 4 :
R 2x
R dx
= eln(x 16)
p dx x2 16 2
Integrating factor: =e =e = x2 16:
R R 2(x2 16) R
y= 1 C+ q dx = x2 1 16 C+ x 4
dx = 1
x2 16
C+ 2x + 8dx

) y= x2 +8x+C
x2 16
:

Problem 1.3.9 y 0 (tan x) y = 3x2 x3 tan x:


dy
Linear D.E., dx
+ p (x) y = q (x) :
p= tan x q = 3x2 x3 tan x:
R R
p dxR = tan dx = ln (sec x) = ln (cos x)
= e p dx = cos x; q = 3x2 cos x x3 sin x:
Using
R integration
R 2 by parts, we R …nd that
3
Rq dx = 3x cos x dx x sin
R x 2dx
= 3x2 cos x dx R x 3
cos x + 3x cos x dx = x3 cos x:
)y= 1 C+ q dx = cos1 x (C + x3 cos x) = C sec x + x3 :
CHAPTER 1. FIRST-ORDER DIFFERENTIAL EQUATIONS 19

dy
Problem 1.3.10 dx
+ (1 + tan x) y = sin x + sec x:

dy
Linear D.E., dx
+ p (x) y = q (x) :
p = 1 + tan x q = sin x + sec x:
R R
p dx = (1 + tan x) dx = x + ln (sec x) :
R
Integrating factor: = e p dx = ex+ln(sec x) = ex eln(sec x) = ex sec x:
R R x R R
R x q dx = e sec x (sin xR + sec x) dx = ex tan x dx + ex sec2 x dx
= e tan x dx + ex tan x ex tan x dx = ex tan x:
R
)y= 1 C+ q dx = ex sec 1
x
(C + ex tan x)

) y = Ce x
cos x + sin x :

dy 2
Problem 1.3.11 dx x
y = 4x2 sec2 x
dy
Linear D.E., + p (x) y = q (x) :
dx
2
R R 2
p= x
q = 4x2 sec2 x: p dx = x
dx = 2 ln x
R
p dx ln(x 2 )
Integrating factor: =e =e = x12 :
R R
)y= 1 C+ q dx = x2 C + 4 sec2 xdx

) y = x2 (C + 4 tan x) :

dy
Problem 1.3.12 dx
+ (tan x) y = sec x:
dy
Linear D.E., dx + p (x) y =R q (x) ; R p = tan x q = sec x:
p dx tan x dx ln(sec x)
Integrating factor:
R = e =
R e = e = sec x:
1 2
y= C+ q dx = cos x C + sec x dx = cos x (C + tan x)
) y = C cos x + sin x:

2y 2 +7xy+5x2
Problem 1.3.13 y 0 = xy+x2
; y (1) = 3: (See Problem 1.1.10.)
2y 2 +7xy+5x2 (2y+5x)(y+x)
y0 = xy+x2
= x(y+x)
:
) y0 = 2y+5x
x
= x2 y + 5 ) y0 2
x
y = 5:
Final Answer: Linear D.E., y = Cx2 5x:
At x = 1; y = 3: ) C = 8: ) y = 8x2 5x .
CHAPTER 1. FIRST-ORDER DIFFERENTIAL EQUATIONS 20

1.4 Bernoulli Di¤erential Equations


A …rst-order di¤erential equation is said to be a Bernoulli’s D.E. if it takes
the following form (n is a constant 6= 0; 1):

dy
+ g (x) y = h (x) y n : (1.19)
dx
Method of Solution: We multiply both sides by
n
(1 n) y (1.20)

and get
n dy
(1 n) y + (1 n) g (x) y 1 n
= (1 n) h (x) :
dx
The substitution
z = y1 n
(1.21)
dz dy
(and so, dx = (1 n) y n dx ) turns this Bernoulli D.E. into the following
linear di¤erential equation in the unknown function z (x) :

dz
+ (1 n) g (x) z = (1 n) h (x) :
dx
We obtain its solution z (x). Then we get y (x) from (1.21).

1.4.1 Lecture Examples


dy
Problem 1.4.1 x dx + 2y x6 y 3 = 0:
dy
Bernoulli D.E., n = 3. dx
+ x2 y = x5 y 3
Multiply by (1 n) y n = 2y 3 . ) 2y 3 dx dy 4
x
y 2 = 2x5
Put z = y 2: ) dx
dz
= 2y 3 dx dy
:
) D.E. becomes dz
dx
4
x
z = 2x :5

Linear D.E. in z, p = Rx4 q =R 2x5 :


4 4
Integrating Rfactor: = e p dx R= e x dx = e 4 ln x
= eln x = x 4:
z= 1 C+ q dx = x4 C 2 x dx
1
y2
= z = x4 (C x2 )
) y= x2
p1
C x2
:
CHAPTER 1. FIRST-ORDER DIFFERENTIAL EQUATIONS 21

1.4.2 Classroom Exercises


dy
Problem 1.4.2 x dx + 2y = (x5 cos x) y 3 :
dy
+ x2 y = (x4 cos x) y 3 : Bernoulli D.E, n = 3.
dx
Multiply by (1 n) y n = 2y 3 . ) 2y 3 dx dy 4
x
y 2 = 2x4 cos x
Put z=y 2. ) dx
dz
= 2y 3 dxdy
:
) D.E. becomes R dx x zR= 2x cos x
dz 4 4

Linear D.E. in z, p dx = R x4 dx = 4 ln x = ln x14


p dx
Integrating factor:
R =e = x14R:
1 1 4
y2
=z= C+ q dx = x C 2 cos x dx = x4 (C 2 sin x)
) y= p 1
x2 C 2 sin x
:

dy
Problem 1.4.3 dx
(cos x) y = x3 e4 sin x y 3 : Bernoulli D.E., n = 3:
n
Multiply by (1 n) y = 4y 3 .
) 4y 3 dx
dy
4 (cos x) y 4 = 4x3 e4 sin x : Put z = y 4 . ) dz
dx
dy
= 4y 3 dx :
dz
) D.E. becomes Linear in z (x) : dx
4 (cos x) z = 4x3 e4 sin x :

p= 4 cos x q = 4x3 e4 sin x :


R R
p dx 4 cos x dx 4 sin x
Integrating factor: =e =e =e :
R R
)y =z=4
C+ 1
q dx = e 4 sin x
C + 4x dx = e4 sin x (C + x4 ) 3

p p
) y = 4 e4 sin x (C + x4 ) = esin x 4 C + x4 :

dy p
Problem 1.4.4 dx
+ 2y = 6e2x y:
n 1
Bernoulli D.E, n = 1=2 Multiply by (1 n) y = p
2 y
.
p p
) 2p1 y dx
dy
+ y = 3e2x Put z = y: ) dx
dz
= 1 dy
p
2 y dx
:

) D.E. becomes dz
dx
+ z = 3e2x :
Linear D.E. in z, p = 1, q = 3e2x :
R
p dx
Integrating factor: =e = ex :
p R R
y=z= 1 C+ q dx = e x
C+ 3e3x dx :
p
) ) y = (Ce
x 2
y=e (C + e3x ) = Ce x
+ e2x x
+ e2x ) :
CHAPTER 1. FIRST-ORDER DIFFERENTIAL EQUATIONS 22

1.4.3 Revision Problems


Problem 1.4.5 4y 0 + y cos x = y 3
cos x; y (0) = 2:

y 0 + 41 y cos x = 14 y 3 cos x Bernoulli D.E, n = 3 Multiply by


(1 n) y n = 4y 3
) 4y 3 y 0 + y 4 cos x = cos x
Put z = y4 . ) dx
dz dy
= 4y 3 dx :
) D.E. becomes dz
dx
+ (cos x) z = cos x
R
p dx
R
Linear D.E. in z,R Integrating factor: R = e = e cos x dx
= esin x :
4 1 sin x sin x sin x
y =z= C+ q dx = e C+ e cos x dx = e C + esin x
p
) y = 1 + Ce
4 sin x
)y= 4
1 + Ce sin x :
At x = 0; y = 2: ) 16 = 1 + C; C = 15:
p
4
Thus, y= 1 + 15e sin x :

dy
Problem 1.4.6 4 cos x dx + (sin x) y = y 3 :
(1 n)y n =4y 3
Bernoulli D.E., n = 3. Multiply by 4 cos x
:
) 4y 3 dx
dy
+ (tan x) y 4 = sec x: Put z=y : ) dx
dz 4 dy
= 4y 3 dx :
) D.E. becomes dz
dx
+ (tan x) z = sec x
R LinearR D.E. in z, p = tan x, q = sec x: Integrating factor: =
e p dx = e tan x Rdx = eln(sec x) = sec x: R
z= 1 C+ q dx = cos x C + sec2 x dx = cos x (C + tan x)
p
y 4 = z = C cos x + sin x: ) y = 4 C cos x + sin x:

dy
Problem 1.4.7 dx
+ 14 (tan x) y = 2 (e x
cos x) y 3 :

n
Bernoulli D.E., n = 3: Multiply by (1 n) y = 4y 3 .
) 4y 3 dx
dy
+ (tan x) y 4 = 8e x
cos x: Put z = y 4 . ) dz
dx
dy
= 4y 3 dx :
dz x
) D.E. becomes Linear in z : dx
+ (tan x) z = 8e cos x: p = tan x
q = 8e x cos x::
R R
p dx tan x dx
Integrating factor: =e =e = eln sec x = sec x:
R R
) y4 = z = 1
C+ q dx = cos x C + 8e x
dx = cos x (C 8e x ) :
p
) ) y= 4
cos x (C 8e x ):
CHAPTER 1. FIRST-ORDER DIFFERENTIAL EQUATIONS 23

dy
Problem 1.4.8 x2 dx xy = y 2 :
2
) dx
dy y
x
= xy Bernoulli D.E., n = 2
Multiply by (1 n) y n = y 2 . ) y 2 dxdy
+ x1 y 1
= 1
x2
Put z = y 1: ) dx
dz
= y 2 dxdy
: ) D.E. becomes dz
dx
+ x1 z =
1
x2
: R
p dx
Linear D.E. in zR (x), Integrating factor:
R = e = eln x = x:
1 1 1 1 1
y
=z= C+ q dx = x C x
dx = x (C ln x)
) y= x
C ln x
= x
ln K+ln x
= x
ln(Kx)
:

Another Solution: Homogeneous D.E.


2
dy
dx
y
x
= xy Put u = xy . ) y = xu ) dy
dx
= u + x du
dx
)u du
R + x2 dx uR =dxu
2
) x du
dx
= u2
) u du = x ) u = yx = ln (Kx)
1
)y= x
ln(Kx)
:

dy 1
Problem 1.4.9 (sec x) dx 3
(sec2 x) y = y4; y (0) = 1:

) dy
dx
1
3
(sec x) y = (cos x) y 4 :
n
Bernoulli D.E, n = 4 Multiply by (1 n) y = 3y 4 .
) 3y 4 dy
dx
+ (sec x) y 3
= 3 cos x:

Put z = y 3
. ) dz
dx
= 3y 4 dy
dx
:
) D.E. becomes dz
dx
+ (sec x) z = 3 cos x:
R R
Linear D.E. in z, p dx = sec x dx = ln (sec x + tan x) :
R
p dx
Integrating factor: =e
= sec x + tan x:
R R
y 3=z= 1 C+ q dx = sec x+tan1
x
C + 3 cos x (sec x + tan x) dx
R
) y 3 = sec x+tan
1
x
C + 3 + 3 sin x dx = C+3x 3 cos x
sec x+tan x
:
q
) y = 3 C+3x sec x+tan x
3 cos x
:

At x = 0, y = 1. )1= C+0 3
1+0
) C = 4.
q
) y = 3 4+3x
sec x+tan x
3 cos x
:
CHAPTER 1. FIRST-ORDER DIFFERENTIAL EQUATIONS 24

dy
Problem 1.4.10 dx
+ x3 y = 2x9 y 3 ; y (1) = 1:

dy
Problem 1.4.11 (2x3 9xy 2 ) dx = 2x3 + 6x2 y 15y 3 ; y (1) = 1:

dy
Problem 1.4.12 (6xy 2 3x2 y) dx = 10y 3 6xy 2 + 2x3 ; y (1) = 2:

dy
Problem 1.4.13 dx
(cot x) y = 2 (cos x) y 2 :

dy
Problem 1.4.14 x dx + 3y = (x7 tan x) y 3 :

dy 4
Problem 1.4.15 5 cos x dx + (sin x) y = y :

dy 1
Problem 1.4.16 dx 3
(sec x) y = (cos x) y 4 ; y (0) = 1:

dy
Problem 1.4.17 dx
+ 34 y = (e 3x
cos 2x) y 3 :

dy
Problem 1.4.18 dx
+ 14 (tan x) y = 3 (e3x cos x) y 3 :

dy
Problem 1.4.19 dx
+ x2 y = 2x4 y 3 :

dy
Problem 1.4.20 dx
+ x1 y = 1 2x
x3
e y 2:

dy
Problem 1.4.21 3 dx (tan x) y = 2 (sin x) y 2 :

dy 1
Problem 1.4.22 dx x
y = x2 y 2 :

dy
Problem 1.4.23 x2 dx xy = y 2 :
CHAPTER 1. FIRST-ORDER DIFFERENTIAL EQUATIONS 25

1.5 Further Revision Problems on Chapter 1


dy y 2 5xy
Problem 1.5.1 dx
= y 2 xy
; y (1) = 2

Homogeneous d.e. Put u = y


x
. ) y = xu ) dy
dx
= u + x du
dx
) u + x du
2 2 2
dx
= xx2uu2 5x u
x2 u
= uu 51
) x du
2 2
dx
= uu 15 u = u 5u u1 +u = 2u u 5 1 u
R R dx R R
) u(u2 2u+5
1)du
= x
) u22u2u+5
2
du = 2 dx
x
) ln (u2 2u + 5) = 2 ln (Cx) = ln xK2
) u2 2u + 5 = xK2 : ) K = u2 x2 2ux2 + 5x2 = y 2 2yx + 5x2
p
At x = 1; y = 2: ) K = 5: ) y 2 2yx + 5x2 = 5 . ) y = x + 5 4x2 .

dy 1 ex 4
Problem 1.5.2 dx x
y = x2
y
n
Bernoulli D.E., n = 4. Multiply by (1 n) y = 3y 4 .
) 3y dx + 3 x y = x2
4 dy 1 3 3ex

Put z = y 3: ) dx
dz
= 3y 4 dxdy
:
) D.E. becomes dz
dx
3 3ex
+ x z = x2 :
x
Linear D.E. in z, p = x3 R q = Rx3e2 :
3 3
Integrating factor: R = e p dx = e x dx = e3 ln x = eln x = x3 :
C+
R
q dx C 3 x ex dx R x
z= = x3
= x13 C 3 x ex e dx
) 1
y3
=z= 1
x3
(C 3x ex + 3ex ) ) y= p
3
x
C 3x ex +3ex
:

Problem 1.5.3 x2 y 0 = y xy, y (1) = 2e :

Final Answer: Separable d.e. y = x2 e 1=x

dy
p
Problem 1.5.4 y dx = 2x 9 + y2, y (0) = 4:

Final Answer:
q Separable d.e.
2
p
y= (x2 + 5) 9= x4 + 10x2 + 16

dy y y
Problem 1.5.5 dx
= x
+ sin2 x
, y (1) = 4 :
1 e
Final Answer: Homogeneous d.e. y = x cot ln x
CHAPTER 1. FIRST-ORDER DIFFERENTIAL EQUATIONS 26

+ (x + y)2 = 0,
2 dy
Problem 1.5.6 2xy + x2 2yey dx
y (3) = 0:
2
Final Answer: Exact d.e. x3 + 3x2 y + 3xy 2 3ey = 24

1
Problem 1.5.7 (2y 3 e2x + cos x) dx + 3y 2 e2x + y
dy = 0; y (0) = 1:

Final Answer: Exact d.e. y 3 e2x + sin x + ln y = 1

1
Problem 1.5.8 y + x2 + x
dx + (x sec2 y) dy = 0:
x3
Final Answer: Exact D.E. xy + 3
+ ln x tan y = C:

Problem 1.5.9 Exact(7y + 3x 2) y 0 + (3y + 12x2 ) = 0:


7 2
Final Answer: Exact d.e. 2
y + 3xy 2y + 4x3 = C

dy
Problem 1.5.10 y 2 + 2x sin y ex + (2y + 2xy + x2 cos y) dx = 0; y (0) = 0:

Final Answer: Exact d.e. xy 2 + x2 sin y ex + y 2 = 1

Problem 1.5.11 (y sin x + cos x) dx cos x dy = 0:

Final Answer: Linear and Exact d.e. y = tan x + C sec x

dy
Problem 1.5.12 dx
+ 3y = 7e4x , y (0) = 3:

Final Answer: Linear d.e. y = e4x + 2e 3x

Problem 1.5.13 y 0 y = y 3 e2x :


p x
Final Answer: Bernoulli D.E., y = p 2 e
C e4x

dy
Problem 1.5.14 x2 dx 2xy = 3y 4 :
x2
Final Answer: Bernoulli D.E., y = p
3
C (9x5 =5)
CHAPTER 1. FIRST-ORDER DIFFERENTIAL EQUATIONS 27

Problem 1.5.15 Solve the following …rst-order di¤erential equations:


dy
1. (xy 2 + x3 ) dx = 2y 3 + 4x2 y:

dy
2. (xy + 3x2 ) dx = 2y 2 + 9xy 7x2 ; y (1) = 2:

dy
3. (3xy 2 + 6x2 y) dx = 2y 3 + 3xy 2 2x3 :

dy
4. (4xy 3 + 2x3 y) dx = 3y 4 + x2 y 2 + 5x4 :

dy
5. (4xy 3 15x2 y 2 ) dx = 2y 4 5xy 3 + 2x4 :

dy y
6. x dx = 3y ln x
+ y; y (1) = 1:

dy
7. (3xy 2 4x2 y + 3x3 ) dx = 5y 3 8xy 2 + 9x2 y:

dy
8. (3xy 2 + 3x3 ) dx = 2y 3 + 4x3 ; y (1) = 0:

dy
9. (9x2 y 2 + 4xy 3 ) dx = 2y 4 + 3xy 3 2x4 :

dy
10. (12x2 y 3 10xy 4 ) dx = 3x5 + 3xy 4 4y 5 :

dy
11. (3xy + x2 ) dx = 6y 2 + 3xy; y (1) = 2:

dy
12. x2 dx xy = y 2 :

1 dy
13. (3e2y cos 3x sin x) + 2e2y sin 3x + 1+y 2 dx
=0; y 3
= 1:

y2 1
14. 3x2 sin y x
+ 1+x2
dx + (x3 cos y 2y ln x) dy = 0; y (1) = :

3
15. (2e2x + 3x2 tan y) dx + y
+ x3 sec2 y dy = 0; y (0) = 1:

1
16. (3x2 y 2 + ey sec x tan x cos x) dx + 2x3 y + ey sec x + y
dy = 0 :

4
17. (2e2x sin 3y + sec2 x) dx + 3e2x cos 3y + y
dy = 0 :
CHAPTER 1. FIRST-ORDER DIFFERENTIAL EQUATIONS 28

y2 dy
18. x
sin ( x) + 2x tan y + (3y 2 + 2y ln x + x2 sec2 y) dx =0;
y (1) = 0:

19. 2xy 3 + ey + p 1 dx + (3x2 y 2 + xey + sec2 y) dy = 0; y (0) = 4 :


1 x2

20. (2x + 2xy + y 2 cos x) dx + (x2 + 2y sin x ey ) dy = 0; y (0) = 0:

2y
21. (2x cos y + sec2 x) dx + 1+y 2
x2 sin y dy = 0; y (0) = 0:

dy
22. (2 cos 2x tan y + 3e3x ) + (sin 2x sec2 y + 5y 4 ) dx =0; y (0) = 1:
3
23. y 2 ex + y x
dx + (2yex + x + cos y) dy = 0:

24. (y 3 sec2 x + 4x3 cos y) dx + (3y 2 tan x x4 sin y + ey ) dy = 0:


dy
25. dx
+ (sec x) y = 3 cos x:

dy
26. dx
+ y tan x = 5 cos2 x; y (0) = 7:

dy
27. dx
+ x5 y = 6 sin x
x5
; y (1) = 4

dy 2x 2
28. dx
+ x2 16
y = x 4
; y (0) = 1:

dy
29. e3x dx + 3e3x y = 4 cos 2x:

dy
30. (csc x) dx (sec x) y = 4:

dy x
31. dx
+y =e cos x; y (1) = 2:

dy 4
32. dx
4x3 y = 3ex cos 3x; y (0) = 2:

dy
33. x dx 2y = x3 sec2 x; y( ) = 2
:

x2
34. y 0 + 2xy = 2x: Answer y = 1 + Ce :

35. y 0 + y cos x = cos x: Answer y = 1 + Ce sin x


:
dy 2x 2x
36. dx
+ 2y = 8xe ; y (0) = 1: Answer y = e (1 + 4x2 ) :
CHAPTER 1. FIRST-ORDER DIFFERENTIAL EQUATIONS 29

1.6 Some Applications of First-order


Di¤erential Equations in Engineering
Problem 1.6.1 A body is projected vertically from the surface of earth. At
distance x from the centre of earth, the acceleration is given by
d2 x dv gR2
= = ; x R; (1.22)
dt2 dt x2
where t denotes time, v (t) is the velocity at time t, g is the earth gravitational
constant and R is the earth’s radius. By using the substitution dv dt
= dv(x(t))
dt
=
dv dx dv
dx dt
= v dx , the di¤erential equation (1.22) becomes the following separable
D.E., describing the velocity v in terms of the distance x:
dv gR2
v = ; x R: (1.23)
dx x2
Given that the initial velocity is v (at x = R), show that
r
2gR2
v (x) = (v 2 2gR) + :
x
Hence, show that the escape velocity from the earth’s surface (ignoring air
resistance) equals p
v escape = 2gR.

Problem 1.6.2 RIn a series RC-circuit, the current I (t) is governed by the
equation RI + C1 I dt = E (t), where E (t) is the electric potential applied to
the circuit. When we di¤erentiate both sides, we get the di¤erential equation
dI 1 dE
R + I= :
dt C dt
Given that E (t) = E e kt and the initial current is 0, …nd I (t).
8
kE C
< kRC 1
e kt e t=RC ; if kRC 6= 1;
Final Answer: I (t) =
: kER t e t=RC ; if kRC = 1:

Problem 1.6.3 The di¤erential equation for a series RL-circuit is


dI
L + RI = E (t) :
dt
kt
Given that E (t) = E e and the initial current is 0, …nd I (t).
( E
R kL
e kt e Rt=L ; if R 6= kL;
Final Answer: I (t) = E
:
kt
L
te ; if R = kL:
Chapter 2

Second-order Linear
Di¤erential Equations

2.1 Form of the General Solution


A second-order linear di¤erential equation (in an unknown function y (x))
takes the following form:
d2 y dy
a (x) 2 + b (x) + c (x) y = Q (x) : (2.1)
dx dx
All solutions of this D.E. are provided within its general solution yG:S: (x).
The corresponding homogeneous equation is
d2 y dy
a (x) 2
+ b (x) + c (x) y = 0: (2.2)
dx dx
Its general solution is called the homogeneous solution of (2.1) (also called
its cpmplementary function), and is denoted by yh (x).
We say that two functions y1 (x) and y2 (x) are independent if their ratio
y1 (x)
y2 (x)
is not a constant function.

Theorem 4 (Form of the General Solution) (1) The homogeneous


equation (2.2) has solutions of dimension 2. This means that it has two
independent solutions.
(2) Let y1 and y2 be any two independent solutions of (2.2).Then the
general solution yh (x) of (2.2) is given by

yh (x) = C1 y1 (x) + C2 y2 (x) ; (2.3)

in which C1 and C2 are arbitrary constants.

30
CHAPTER 2. SECOND-ORDER LINEAR DIFFERENTIAL EQUATIONS31

(3) Let let yp (x) be any solution of (2.1). Then the general solution
yG:S: (x) of (2.1) is given by

yG:S: (x) = yp (x) + yh (x) (2.4)


= yp (x) + C1 y1 (x) + C2 y2 (x) :

(The function yp (x) in (2.4) is usually said to be a particular solution of


the di¤erential equation (2.1).)
Proof. Let y1 be any nonzero solution of equation (2.2). (Such solution y1
exists, by virtue of the Theory of Di¤erential Equations).
We use the substitution:
y (x)
V (x) = : (2.5)
y1 (x)
2 2
dy d2 V
So, y = V y1 . Hence, dx = V dydx
1
+ dV y and dx
dx 1
d y d y1 dV dy1
2 = V dx2 + 2 dx dx + dx2
y1 :
Then the di¤erential equation (2.1) becomes
2 2
a V ddxy21 + 2 dV dy1
dx dx
+ ddxV2 y1 + b V dy dx
1
+ dV y + cV y1 = Q
dx 1
2
) V a ddxy21 + b dy
2
dx
1
+ cy1 + ay1 ddxV2 + 2a dy
dx
1
+ by1 dV
dx
= Q:
2
Since a ddxy21
+ b dy
+ cy1 = 0, we are left with the following di¤erential
dx
1

equation:
2
ay1 ddxV2 + 2a dy
dx
1
+ by1 dV
dx
= Q: Then we use a second substitution:

dV
z (x) = (2.6)
dx
dz 2
Hence, dx = ddxV2 . And we come out with the following …rst-order, linear
di¤erential equation in z (x) : dz
ay1 dx + 2a dy
dx
1
+ by1 z = Q; that
dz dy1 Q
is dx + 2a
y1 dx
+ ab z = ay1
: Thus, we have a …rst-order linear di¤erential
equation in z (x) :
dz
+ p (x) z = q (x) ;
dx
dy1
where p (x) = 2a
y1 dx
+ ab and q (x) = ayQ1 .
Its solution, as usual, is
Z
1
z (x) = C2 + qdx = C2 z2 (x) + z (x) ;

say, for some nonzero functions z2 (x) and z (x) :


But, as dV
dx
= z (x) (2.6), then
CHAPTER 2. SECOND-ORDER LINEAR DIFFERENTIAL EQUATIONS32
R R R
) V (x) = C1 + z (x) dx = C1 + C2 z2 (x) dx + z (x) dx:
We apply (2.5) and get yG:S:R(x) = V (x) y1 (x) : R
) yG:S: (x) = C1 y1 (x) + C2 z2 (x) dx y1 (x) + z (x) dx y1 (x)

) yG:S: (x) = C1 y1 (x) + C2 y2 (x) + yp (x) ;

for three functions y1 (x) ; y2 (x) ; yp (x).


On the one hand, when we choose C1 = C2 = 0, we …nd that yp is one of
the solutions of the di¤erential equation (2.1) itself.
On the one hand, when we choose C1 = 0 and C2 = 1, we …nd that
y2 = yG:S: yp ; from which we …nd that y2 is a solution of the homoge-
neous equation (2.2). Also, yy21 is nonconstant obviously, so y1 (x) ; y2 (x) are
independent functions.

Remark 2.1.1 Elementary Linear Algebra allows us to replace, in the form


(2.4), the function yp (x) by any other solution of (2.1), and replace y1 (x)
and y2 (x) by any two independent solutions of (2.2).

Remark 2.1.2 In di¤erential equations, the form of the


general solution yG:S: = C1 y1 + C2 y2 + yp (in which C1 and C2 are
arbitrary constants, and y1 ; y2 are two independent functions) is valid for
second-order linear di¤erential equations only.
That equation has to be the following one (in the unknown function y):

y 00 y0 y yp00 yp0 yp

y100 y10 y1 = y100 y10 y1 :

y200 y20 y2 y200 y20 y2

Remark 2.1.3 The general solution of a third-order linear di¤erential


equation:
d3 y d2 y dy
a (x) dx 3 + b (x) dx2 + c (x) dx + k (x) y = Q (x) :

takes the form:


yG:S: = C1 y1 + C2 y2 + C3 y3 + yp ;
in which yp is a solution of that di¤erential equation, y1 ; y2 ; y3 are three
independent solutions of the corresponding homogeneous equation, and C1 ,C2 ; C3
are arbitrary constants. This can be established by a proof very similar to that
of Theorem 4, above, which also makes use of that theorem.
Proceeding in this manner, a similar conclusion is shown to hold for the
forms of the general solutions, of linear di¤erential equations of all orders.
CHAPTER 2. SECOND-ORDER LINEAR DIFFERENTIAL EQUATIONS33

2.2 Order Reduction of Linear Di¤erential


Equations
If we are seeking the general solution of a second order linear di¤erential
equation:
d2 y dy
a (x) 2 + b (x) + c (x) y = Q (x) ; (2.7)
dx dx
and if we know one nonzero solution y1 (x) of its corresponding homogeneous
equation
d2 y dy
a (x) 2 + b (x) + c (x) y = 0; (2.8)
dx dx
then we can retrace all the steps in the proof of Theorem 4, above, in order
to come out with the required general solution.
Speci…cally, we apply to equation (2.7) the following two successive
substitutions:
y (x)
V (x) = ;
y1 (x)
and
dV
z (x) = :
dx
This results in some …rst-order linear di¤erential equation in z (x) :

dz
+ p (x) z = q (x) :
dx
We solve that last equation. Then we return to the required y (x).
An example of the application of this method is given in Problem 2.5.22,
below.

Remark 2.2.1 This is a general method for solving an nth-order linear


di¤erential equation, through solving another (n-1) th-order linear di¤erential
equation, which we can obtain by applying these two particular substitutions
to the given equation.
Although the steps of this method are quite simple, their performance is
usually rather long, and requires the computation of many integrals, which
may or may not be amenable!
We shall, therefore, re…ne this method in Section 2.4 to a computationally-
easier one, called the Method of Variation of Parameters, which requires the
before-hand knowledge of the whole of the homogeneous solution yh (x) :
CHAPTER 2. SECOND-ORDER LINEAR DIFFERENTIAL EQUATIONS34

2.3 The Homogeneous Solutions yh (x) of


Linear Di¤erential Equations with
Constant Coe¢ cients
In this section and the next one, we consider only second-order linear di¤er-
ential equations with constant coe¢ cients; that is di¤erential equations
d2 y dy
a 2
+ b + cy = Q (x) : (2.9)
dx dx
whose three coe¢ cients a; b; c are constants.
The homogeneous solution (the cpmplementary function) of (2.9) yh (x) =
C1 y1 +C2 y2 comprises all solutions of the corresponding homogeneous equa-
tion
d2 y dy
a 2 + b + cy = 0: (2.10)
dx dx
Two independent solutions y1 and y2 of (2.10) are obtained by examining the
following auxiliary equation:
am2 + bm + c = 0: (2.11)
Its two solutions (roots) are
p
b b2 4ac
m1;2 = : (2.12)
2a
The real number b2 4ac is called the discriminant of equation (2.11).
These solutions m1;2 satisfy the following unique factorization of the
auxiliary equation:
a (m m1 ) (m m2 ) = 0: (2.13)

Theorem 5 Let m1 ; m2 be the two roots of the auxiliary equation:


am2 + bm + c = 0:
We take yh (x) in accordance with the following table (C1 ; C2 are arbitrary
constants):
# Discriminant Roots m1 ; m2 yh = C1 y1 + C2 y2
1 positive real, distinct yh = C1 em1 x + C2 em2 x
2 0 real, identical yh = C1 em1 x + C2 xem1 x
= em1 x (C1 + C2 x)
3 negative complex conjugates yh = C1 e x sin x + C2 e x cos x
m1;2 = i = e x (C1 sin x + C2 cos x)
CHAPTER 2. SECOND-ORDER LINEAR DIFFERENTIAL EQUATIONS35

Proof. From formula (2.12) we …ndpthat


2
m1 + m2 = ab ; m1 m2 = b a 4ac ; m1 m2 = ac :
We use the following substitution:
y (x) = e x z (x) , where = m1 +m
2
2
= b
2a
.
dy x dz
Then dx
=e dx
+ z ; and
d2 y x d2 z dz 2
dx2
=e +2 dx2
+ z : dx
We perform this substitution in the given homogeneous d.e. (2.10), and
it becomes the following d.e. in the unknown function z (x) :
d2 z dz dz
e x a dx 2 + 2a dx + a
2
z + b dx + b z + cz = 0:
x
We divide by ae , and get
d2 z
dx2
+ 2 + ab dx dz
+ 2 + ab + ac z = 0
) d2 z
dx2
dz
+ 0 dx + b2
4a2
b2
2a2
+ c
a
z = 0; that is,

d2 z b2 4ac
= z:: (2.14)
dx2 4a2
Case 1: As m1 ; m2 are distinct and real, then b2 4ac is a nonzero real
number. It isp direct to verify that
p
the solution of d.e. (2.14) becomes:
b2 4acx=2a b2 4acx=2a
zh = C1 e + C2 e ; (using (2.3) of Theorem 4).
p p
) yh (x) = e x zh = e bx=2a C1 e b 4acx=2a + C2 e b 4acx=2a
2 2

p p
= C1 e( b+ b 4ac)x=2a + C2 e( b2 4ac)x=2a
2 b
; that is,
yh (x) = C1 em1 x + C2 em2 x :
Case 2: As m1 = m2 = , then the discriminant b2 4ac = 0:
d2 z
So, equation (2.14) becomes here: dx 2 = 0:

Its solution is clearly: zh = C1 + C2 x:


Therefore, yh (x) = e x zh = em1 x (C1 + C2 x) :
Case 3: Since b2 4ac is negative then
b
m1;2 = p i , where = 2a ; as before,
b2 4ac b2 4ac 2
and i = 2a ; that is, 4a2 = :
d z 2 2
Consequently, (2.14) becomes in this case: dx 2 + z = 0:
It is direct to verify that the general solution of this last d.e. is
zh = C1 sin x + C2 cos x:
) yh (x) = e x zh = e x (C1 sin x + C2 cos x) :
CHAPTER 2. SECOND-ORDER LINEAR DIFFERENTIAL EQUATIONS36

2.4 Solution by the Method of Variation of


Parameters
2.4.1 Describing The Method
This Method of Variation of Parameters is a general, and much easier,
method for computing a particular solution yp (x) for a second-order linear
di¤erential equation (in an unknown function y (x)):

d2 y dy
2
+ b (x) + c (x) y = Q (x) ; (2.15)
dx dx
d y 2
in which the coe¢ cient of dx2 is 1 (If otherwise, divide by its given coef-

…cient.)
This time, we need a prior knowledge of the whole of the homogeneous
solution (the complementary function) of (2.15)

yh (x) = C1 y1 (x) + C2 y2 (x) ; (2.16)

which contains all solutions of the homogeneous equation:

d2 y dy
2
+ b (x) + c (x) y = 0: (2.17)
dx dx
We put the required particular solution yp (x) of (2.15) in the form:

yp (x) = V1 (x) y1 (x) + V2 (x) y2 (x) ; (2.18)

in which V1 (x) and V2 (x) are two unknown functions, that we need to de-
termine.
We are going to substitute this yp (x) for y (x) in equation (2.15). This
will result in one di¤erential equation only in the two unknown functions
V1 (x) and V2 (x). This is consistent with our prior knowledge that equation
(2.15) has an in…nite number of solutions, any one of them can serve as the
required yp (x) : So, in order to come out with one solution only for yp (x) ;
we impose another di¤erential equation on the two unknowns V1 and V2 .
We choose it in such a manner that it also facilitates their solution greatly,
without causing any contradictions That expedient equation is:

y1 V10 + y2 V20 = 0: (2.19)

As yp = y1 V1 + y2 V2 , then using (2.19),


yp0 = y10 V1 + y20 V2 + y1 V10 + y2 V20 = y10 V1 + y20 V2 + 0,
CHAPTER 2. SECOND-ORDER LINEAR DIFFERENTIAL EQUATIONS37

Hence, yp00 = y100 V1 + y200 V2 + y10 V10 + y20 V20 :


We next substitute yp (x) for y (x) in equation (2.15) and get:
Q (x) = yp00 + byp0 + cyp
= y100 V1 + y200 V2 + y10 V10 + y20 V20 + by10 V1 + by20 V2 + cy1 V1 + cy2 V2
= y100 V1 + by10 V1 + cy1 V1 + y200 V2 + by20 V2 + cy2 V2 + y10 V10 + y20 V20
= (y100 + by10 + cy1 ) V1 + (y200 + by20 + cy2 ) V2 + y10 V10 + y20 V20
= 0V1 + 0V2 + y10 V10 + y20 V20 ;
because both y1 ; y2 satisfy (2.17).
So, our second equation in V1 ; V2 turns out to be:

y10 V10 + y20 V20 = Q (x) : (2.20)

We were expecting two di¤erential equations in V1 ; V2 : Instead, we got


two algebraic linear equations (2.19) and (2.20) in V10 ; V20 ; which are much
easier to solve.
After we solve them and determine V10 ; V20 ; we obtain V1 ; V2 by integration.
Then our substitution (2.18) yields a particular solution yp :
CHAPTER 2. SECOND-ORDER LINEAR DIFFERENTIAL EQUATIONS38

2.4.2 Steps of the Method of Variation of Parameters:


We want to solve a second-order linear di¤erential equation
d2 y dy
2
+ b (x) + c (x) y = Q (x) ; (2.21)
dx dx
Step 1: Find the homogeneous solution yh (x) (the solution of the
homogeneous equation (2.17)):
yh (x) = C1 y1 (x) + C2 y2 (x) : (2.22)
Step 2: Write the particular solution yp (x) as:
yp (x) = V1 y1 + V2 y2 ;
whereby V1 (x) and V2 (x) are two unknown functions, to be determined.
Step 3: Write down the following two equations:
y1 V10 + y2 V20 = 0: (2.23)
y10 V10 + y20 V20 = Q (x) : (2.24)
Step 4: Compute the following determinant (the Wronskian of y1 ; y2 ):
y1 y2
W = : (2.25)
y10 y20
Note that the independence of the two solutions y1 (x) ; y2 (x) of (2.17)
ensures that their Wronskian W is a nonzero function. Which means that
the two algebraic equations (2.23) and (2.24) are mutually consistent (non-
contradictory), as we had hoped.
Step 5: Solve these two algebraic equations in the unknown functions
V1 and V20 : using Cramer’s Rule, which yields the solution as:
0

0 y2 y1 0
y2 Q (x) Q (x) y20 y1 Q (x) 0
y1 Q (x)
V10 = = ; V20 = = : (2.26)
W y1 y2 W y1 y2
y10 y20 y10 y20
Step 6: Obtain the two functions V1 (x) and V2 (x), by integrating V10 (x)
and V20 (x), respectively.
Step 7: Obtain a particular solution yp (x) as:
yp (x) = V1 y1 + V2 y2 . (2.27)
Step 8: Then write the general solution yG:S: (x) of (2.21) as:
yG:S: = yp + yh .
CHAPTER 2. SECOND-ORDER LINEAR DIFFERENTIAL EQUATIONS39

2.4.3 Lecture Examples


Problem 2.4.1 Verify that the function yh (x) = C1 sec x + C2 tan x is the
homogeneous solution of the following linear di¤erential equation:
y 00 (tan x) y 0 (sec2 x) y = 12 sin x cos x.
Then use this yh (x) to …nd the its general solution, by means of the
Method of Variation of Parameters.

The student should verify that both (independent) functions:


y1 = sec x and y2 = tan x
are solutions of the corresponding homogeneous d.e..
Their Wronskian is:
y1 y2 sec x tan x
W = = = sec3 x sec x tan2 x:
y10 y20 sec x tan x sec2 x
) W = sec x (sec2 x tan2 x) = sec x:
We solve the following two equations, in which Q (x) = 12 sin x cos x:

y1 V10 + y2 V20 = 0;
y10 V10 + y20 V20 = Q (x) :
y2 Q
V10 = W
= 12 tan x sin x cos x
sec x
= 12 sin2 x cos x:
Z Z
) V1 = V10 dx = 12 sin2 x cos x dx = 4 sin3 x:
y1 Q sec xQ
V20 = W
= = Q = 12 sin x cos x:
sec x
Z Z
) V2 = 0
V2 dx = 12 sin x cos x dx = 6 cos2 x:

Consequently,
yp (x) = V1 y1 + V2 y2 = 4 sin3 x sec x + 6 cos2 x tan x
= 4 sin2 x tan x + 4 cos2 x tan x + 2 cos2 x tan x:
) yp (x) = 4 tan x + 2 sin x cos x: Finally,

) yG:S: = yp + yh = 2 sin x cos x + C1 sec x + K2 tan x:


CHAPTER 2. SECOND-ORDER LINEAR DIFFERENTIAL EQUATIONS40

2.4.4 Classroom Exercises


Problem 2.4.2 Find the general solution of the following second-order
linear di¤erential equation, using the Method of Variation of Parameters:
d2 y dy
dx2
+ 2 dx 8y = 36xe2x :

Auxiliary Equation: m2 + 2m 8=0


) (m + 4) (m 2) = 0 ) m1 = 4; m2 = 2
) yh = C1 e 4x
+ C2 e2x :
4x
We take y1 (x) = e and y2 (x) = e2x ; and write
4x
yp (x) = V1 y1 + V2 y2 = V1 e + V2 e2x :
We solve the two following equations:

y1 V10 + y2 V20 = V10 e 4x + V20 e2x = 0:


y10 V10 + y20 V20 = 4V10 e 4x + 2V20 e2x = Q (x) = 36xe2x :

The Wronskian of y1 ; y2 is
y1 y2 e 4x e2x 2x
W = = = 6e :
y10 y20 4e 4x
2e2x
y2 Q 36xe2x e2x
V10 = W
= 6e 2x
= 6xe6x :
Z Z Z
V1 = V10 dx = 6xe 6x
dx = xe 6x
+ e6x dx:

) V1 = xe6x + 16 e6x
y1 Q 36xe 4x e2x
V20 = W
= 6e 2x = 6x
Z Z
) V2 = V20 dx = 6x dx = 3x2

) yp (x) = V1 e 4x
+ V2 e2x = xe2x + 61 e2x + 3x2 e2x
) yp (x) = 3x2 x+ 1
6
e2x

) yG:S: = yp + yh = (3x2 x + C2 ) e2x + C1 e 4x


:
Check:
d2 (3x2 x+ 16 )e2x d(3x2 x+ 16 )e2x 1
dx2
+2 dx
8 3x2 x+ 6
e2x = 36xe2x :
CHAPTER 2. SECOND-ORDER LINEAR DIFFERENTIAL EQUATIONS41

Problem 2.4.3 Verify that the function yh (x) = C1 sec2 x + C2 sec x tan x
is the homogeneous solution of the following linear di¤erential equation:
y 00 3 (tan x) y 0 2y = sec3 x.
Then use this yh (x) to …nd the its general solution, by means of the
Method of Variation of Parameters.

The student should verify that both (independent) functions:


y1 = sec2 x and y2 = sec x tan x
are solutions of the corresponding homogeneous d.e..
y1 y2 sec2 x sec x tan x
Their Wronskian is: W = =
y10 y20 2 sec2 x tan x sec x tan2 x + sec3 x
= sec3 x (tan2 x + sec2 x 2 tan2 x) = sec3 x (sec2 x tan2 x)
) W = sec3 x:
We solve the following two equations, in which Q (x) = sec3 x :

y1 V10 + y2 V20 = 0;
y10 V10 + y20 V20 = Q (x)
y2 Q
V10 = W
= sec x tan x
Z Z
0
V1 = V1 dx = sec x tan x dx = sec x:
y1 Q
V20 = W
sec2 x:
=
Z Z
) V2 = V2 dx =
0
sec2 x dx = tan x
Consequently,
yp (x) = V1 e x + V2 e 2x = sec3 x sec x tan2 x
) yp (x) = sec x (sec2 x tan2 x) = sec x: Finally,

) yG:S: = yp + yh = sec x + C1 sec2 x + C2 sec x tan x:


CHAPTER 2. SECOND-ORDER LINEAR DIFFERENTIAL EQUATIONS42

Problem 2.4.4 Find the general solution of the following second-order


linear di¤erential equation, using the Method of Variation of Parameters:
1
y 00 + 3y 0 + 2y = 1+e x

Auxiliary Equation: m2 + 3m + 2 = 0 ) (m + 1) (m + 2) = 0
) m1 = 1 ; m2 = 2 ) yh = C1 e x
+ C2 e 2x :
We take y1 (x) = e x and y2 (x) = e 2x
; and write
yp (x) = V1 y1 + V2 y2 = V1 e x + V2 e 2x :
We solve the two following equations:
y1 V10 + y2 V20 = 0:
1
y10 V10 + y20 V20 = Q (x) = :
1 + ex
y1 y2 e x e 2x 3x
The Wronskian of y1 ; y2 is = = e :
y10 y20 e x
2e 2x
0 y2 0 e 2x
1
Q (x) y20 1+ex
2e 2x e 2x ex
V10 = W
= e 3x
= e3x 1+ex
= 1+ex
:
Z Z
ex
V1 = V10 dx = 1+ex
dx = ln (1 + ex )
y1 0
y10 Q (x) e x 0 e2x
V20 = = e3x 1 =
W e x 1+ex
1+ex
Z Z Z
e2x ex
V2 = V20 dx = 1+ex
dx = 1+ex
ex dx:
We use
Z the.substitution
Z z = ex :
z 1
V2 = 1+z
dz = 1 + 1+z dz = z + ln (1 + z) = ex + ln (1 + ex ) :
Consequently,
yp (x) = V1 e x + V2 e 2x = e x ln (1 + ex ) + e 2x
( ex + ln (1 + ex ))
) yp (x) = (e x + e 2x ) ln (1 + ex ) e x : Finally,
) yG:S: = yp + yh = (e x
+e 2x
) ln (1 + ex ) + C3 e x
+ C2 e 2x
:
Check: e2x yp (x) = (ex + 1) ln (1 + ex ) ex
We di¤erentiate both sides:
ex
e2x yp0 + 2e2x yp = ex ln (1 + ex ) + (ex + 1) 1+e x ex = ex ln (1 + ex )
) ex yp0 + 2ex yp = ln (1 + ex ) (divide by ex )
ex
We di¤erentiate both sides: ex yp00 + ex yp0 + 2ex yp0 + 2ex yp = 1+e x

) ex yp00 + 3ex yp0 + 2ex yp = 1+e


ex
x :
1
We divide by ex , and get yp00 + 3yp0 + 2yp = 1+e x ; as requested.
CHAPTER 2. SECOND-ORDER LINEAR DIFFERENTIAL EQUATIONS43

Problem 2.4.5 Find the general solution of the following linear di¤erential
equation, using the Method of Variation of Parameters:

d2 y dy
6 + 10y = 2e3x cos x:
dx2 dx

Auxiliary Equation: m2 6m + 10 = 0
p p p
) m1;2 = b b2
2a
4ac
= 6 36 40
2
= 6
2
4

) m1;2 = 3 i= i with = 3 and =1


) Homogeneous solution is: yh = C1 e3x sin x + C2 e3x cos x:
We take y1 = e3x sin x and y2 = e3x cos x.
Their Wronskian is:
y1 y2 e3x sin x e3x cos x
W = 0 0 =
y1 y2 3e sin x + e cos x 3e cos x e3x sin x
3x 3x 3x

= e6x 3 sin x cos x sin2 x 3 sin x cos x cos2 x


)W = sin2 x + cos2 x e6x = e6x
We solve the following two equations, in which Q (x) = 2e3x cos x:

y1 V10 + y2 V20 = 0;
y10 V10 + y20 V20 = Q (x)
y2 Q 3x 3x
V10 = Z W = e cos x2e cos x
Z e6x = 2 cos2 x = 1 + cos 2x
V1 = V10 dx = (1 + cos 2x) dx = x + 21 sin 2x = x + sin x cos x:
y1 Q e3x sin x2e3x cos x
V20 = W
= e2 x
= 2 sin x cos x
Z Z
V2 = V20 dx = 2 sin x cos x dx = sin2 x:
Consequently,
yp (x) = V1 e3x sin x+V2 e3x cos x = (x + sin x cos x) e3x sin x sin2 xe3x cos x =
3x
xe sin x
Finally, yG:S: = yp + yh = (C1 + x) e3x sin x + C2 e3x cos x:
CHAPTER 2. SECOND-ORDER LINEAR DIFFERENTIAL EQUATIONS44

2.4.5 Revision Problems


Problem 2.4.6 Find the general solution of the following second-order
linear di¤erential equation, using the Method of Variation of Parameters:
y 00 + y = sec x tan x

Auxiliary Equation:
p m2 + 1 = 0 ) m2 = 1
) m1;2 = 1= i= i
) =0 and =1
) yh = e (C1 sin x + C2 cos x) = C1 sin x + C2 cos x :
x

We take y1 (x) = sin x and y2 (x) = cos x; and write


yp (x) = V1 y1 + V2 y2 = V1 sin x + V2 cos x:
We solve the following two equations:

y1 V10 + y2 V20 = V10 sin x + V20 cos x = 0:


y10 V10 + y20 V20 = V10 cos x V20 sin x = Q (x) = sin (ex ) :

The Wronskian of y1 ; y2 is
y1 y2 sin x cos x
W = = = sin2 x cos2 x = 1:
y10 y20 cos x sin x
y2 Q
V10 = W = y2 Q = cos x sec x tan x = tan x:
Z Z
0
V1 = V1 dx = tan x dx = ln (sec x) :
y1 Q
V20 = W
= y1 Q = sin x sec x tan x
) V20 = Z tan2 x = 1Z sec2 x
) V2 = V20 dx = 1 sec2 x dx = x tan x
Consequently,
yp (x) = V1 e x + V2 e 2x = sin x ln (sec x) + x cos x cos x tan x
) yp (x) = sin x ln (sec x) + x cos x sin x: Finally,

) yG:S: = yp + yh = sin x ln (sec x) + x cos x + C3 sin x + C2 cos x:


CHAPTER 2. SECOND-ORDER LINEAR DIFFERENTIAL EQUATIONS45

Problem 2.4.7 Find the general solution of the following second-order lin-
ear di¤erential equation, using the Method of Variation of Parameters:
y 00 + 3y 0 + 2y = sin (ex ) :

Auxiliary Equation: m2 + 3m + 2 = 0 ) (m + 1) (m + 2) = 0
) m 1 = 1 ; m2 = 2 ) yh = C1 e + C2 e 2x :
x

We take y1 (x) = e x and y2 (x) = e 2x ; and write


yp (x) = V1 y1 + V2 y2 = V1 e x + V2 e 2x : We solve the following 2
equations:
y1 V10 + y2 V20 = 0:
y10 V10 + y20 V20 = Q (x) = sin (ex ) :
y1 y2 e x e 2x 3x
Wronskian of y1 ; y2 is W = = = e :
y10 y20 e x 2e 2x
0 y2 0 e 2x
V10 = =W = =e 3x = e3x ( e 2x
sin (ex ))
Q (x) y20 x
sin (e ) 2e 2x
) V10 = x x x
Z e sin (e )Z: Next, we use the.substitution z = e :
V1 = V10 dx = ex sin (ex ) dx = cos (ex )
y1 0 e x 0
V20 = 0 =W = e3x = e2x sin (ex )
y1 Q (x) e x sin (ex )
Z Z Z
) V2 = V1 dx =
0 2x x
e sin (e ) dx = z sin z dz (z = ex )
Z
) V2 = z cos z cos z dz = z cos z sin z = ex cos (ex ) sin (ex ) :
Consequently, yp (x) = V1 e x + V2 e 2x
= e x cos (ex ) + e 2x (ex cos (ex ) sin (ex ))
) yp (x) = 0 e 2x sin (ex ) : Finally,

) yG:S: = yp + yh = e 2x
sin (ex ) + C1 e x
+ C2 e 2x
:

Check: e2x yp (x) = sin (ex ) : We di¤erentiate both sides:


e yp + 2e yp = ex cos (ex )
2x 0 2x

We divide by ex : ex yp0 + 2ex yp = cos (ex )


We again di¤erentiate both sides w.r.t. x:
ex yp00 + ex yp0 + 2ex yp0 + 2ex yp = ex sin (ex )
) ex yp00 + 3ex yp0 + 2ex yp = ex sin (ex ) :
When we divide by ex , we get yp00 + 3yp0 + 2yp = sin (ex ) ;
which veri…es that this yp is a solution of the given di¤erential equation.
CHAPTER 2. SECOND-ORDER LINEAR DIFFERENTIAL EQUATIONS46

Problem 2.4.8 Find the general solution of the following linear


di¤erential equation, using the Method of Variation of Parameters:

d2 y dy
2 (cot x) + 1 + 2 cot2 x y = 2 sin x;
dx2 dx

knowing that its homogeneous solution is: yh = C1 sin x + C2 x sin x:

We have yh = C1 y1 + C2 y2 ; with y1 = sin x and y2 = x sin x.


Their Wronskian is:
y1 y2 sin x x sin x
W = 0 0 =
y1 y2 cos x x cos x + sin x
) W = x sin x cos x + sin2 x x sin x cos x = sin2 x:
We compute two functions V1 (x) and V2 (x) such that
yp (x) = y1 V1 + y2 V2 ;
by solving the following two equations in V10 and V20 :

y1 V10 + y2 V20 = 0;
y10 V10 + y20 V20 = Q (x) = 2 sin x:
y2 Q
By Cramer’s Rule, V10 = W
= 2 x sin x sin x
sin2 x
= 2x
Z Z
) V1 = V1 dx =
0
2xdx = x2 ;
y1 Q
V20 = = 2 sinsinx 2sinx x = 2
W
Z Z
) V2 = V2 dx = 2 dx = 2x:
0

Consequently,
yp (x) = y1 V1 + y2 V2 = x2 sin x + 2xx sin x = x2 sin x:
Finally, yG:S: = yh + yp = (C1 + C2 x + x2 ) sin x:
CHAPTER 2. SECOND-ORDER LINEAR DIFFERENTIAL EQUATIONS47

Problem 2.4.9 Solve y 00 + 2y 0 15y = 8e3x .

Auxiliary Equation: m2 + 2m 15 = 0
) (m 3) (m + 5) = 0 ) m1 = 3; m2 = 5:
) Homogeneous solution is: yh = C1 e3x + C2 e 5x
:
We take y1 (x) = e3x and y2 (x) = e 5x
; and write
yp (x) = V1 y1 + V2 y2 = V1 e + V2 e 5x :
3x

We solve the = 2 sin xtwo equations:

y1 V10 + y2 V20 = V10 e3x + V20 e 5x = 0:


y10 V10 + y20 V20 = 3V10 e3x 5V20 e 5x = Q (x) = 8e3x :

The Wronskian of y1 ; y2 is
y1 y2 e3x e 5x 2x 2x 2x
W = = = 5e 3e = 8e :
y10 y20 3e3x 5e 5x
y2 Q 2x
V10 = W = 8e
8e 2x
=1
Z Z
V1 = V10 dx = 1 dx = x:
y1 Q 8e6x
V20 = W
== 8e 2x
= e8x
Z Z
) V2 = V20 dx = e8x dx = 1 8x
8
e
1 3x
Consequently, yp (x) = V1 e3x + V2 e 5x
= xe3x 8
e
General Solution is yG:S: = yp + yh = xe3x + C1 e 3x
+ C2 e 5x
.

Problem 2.4.10 Solve the following second-order linear di¤erential


equations, using the Method of Variation of Parameters:

1. y 00 4y = 16x3 + 36x2 + 180x + 80.


49
Final Answer: yG:S: = 2
39x 9x2 + 4x3 + C1 e2x + C2 e 2x
:

2. y 00 + 6y 0 16y = 30e2x .
Final Answer: yG:S: = 3xe2x + C1 e2x + C2 e 8x
:

3. y 00 9y = 6e 3x
.
3x 3x
Final Answer: yG:S: = xe + C1 e + C2 e3x

4. y 00 + 9y = 6e 3x
.
1
Final Answer: yG:S: = 3
e 3x + C1 sin 3x + C2 cos 3x
CHAPTER 2. SECOND-ORDER LINEAR DIFFERENTIAL EQUATIONS48

5. y 00 6y 0 + 5y = 8ex :
Final Answer: yG:S: = 2xex + C1 e5x + C2 ex

6. y 00 + 6y 0 + 5y = 8ex :.
Final Answer: yG:S: = 23 ex + C1 e x
+ C2 e 5x

7. y 00 y0 6y = 15e 2x
.
2x
Final Answer: yG:S: = 3xe + C1 e3x + C2 e 2x
:

8. y 00 + 2y 0 8y = 18e2x :
Final Answer: yG:S: = 3xe2x + C1 e2x + C2 e 4x
:

9. y 00 + 4y 0 + 5y = 10ex :
Final Answer: yG:S: = ex + C1 e 2x
sin x + C2 e 2x
cos x:

10. y 00 + y = 6 sin x + 2 cos x.


Final Answer: yG:S: = (x + C1 ) sin x + (3x + C2 ) cos x:

11. y 00 2y 0 + y = 12xe3x .
Final Answer: yG:S: = C1 ex + C2 xex 3e3x + 3xe3x :

12. y 00 2y 0 + y = 12xex .
Final Answer: yG:S: = C1 ex + C2 xex + 2x3 ex :
CHAPTER 2. SECOND-ORDER LINEAR DIFFERENTIAL EQUATIONS49

2.5 Particular Solutions yp (x) of Equations with


Constant Coe¢ cients - by the Method of
Undetermined Coe¢ cients
A particular solution yp (x) of a second-order linear di¤erential equation (2.9)
is one (any one) solution of the equation itself.
When that equation has constant coe¢ cients; that is,
d2 y dy
a 2
+ b + cy = Q (x) : (2.28)
dx dx
whose three coe¢ cients a; b; c are constants, and a 6= 0, we may obtain a
particular solution yp (x) by means of the Method of Undetermined Coe¢ -
cients. This consists of the following three steps:
Step 1: Put Q (x) (the right-hand side of (2.9)) in a form W (x) that
is stable under di¤erentiation (cannot be altered by di¤erentiation). Then
take yp (x) in that same form W (x), but with undetermined coe¢ cients.
Examples of those stable forms are:
W (x) = A (constants),
W (x) = Aekx (exponential functions)
W (x) = Ax + B (polynomials of degree 1),
W (x) = Ax2 + Bx + C (polynomials of degree 2),
W (x) = Ax3 + Bx2 + Cx + D (polynomials of degree 3),
W (x) = A sin !x + B cos !x;
as well as multiples of the forms above, such as W (x) = (Ax + B) ekx .
Step 2: If the form W (x), suggested in Step 1, has terms in common
with yh (x), then modify it by multiplying the whole form by the variable x.
Repeat this step, if necessary, until the …nal form of yp (x) has no terms in
common with yh (x).
Step 3: Substitute the above suggested yp (x) for the unknown func-
tion y (x), in the D.E. (2.9). Use the resulting equation to compute all the
undetermined coe¢ cients A; B; in that suggestion.

We can see why the modi…cation carried out in Step 2 works, in the fol-
lowing simple case. Suppose the form W (x), suggested in Step 1, is contained
within yh (x), but xW (x) is not. Take yp (x) = xW (x) and substitute it for
y in (2.9). Then
2
Q = a ddxy2p + b dy
dx
p
+ cyp = a (xW00 + 2W0 ) + b (xW0 + W) + cxW
= x (aW00 + bW0 + cW) + (2aW0 + bW) = 0 + (2aW0 + bW)
(because W (x) is a solution of (2.10)).
CHAPTER 2. SECOND-ORDER LINEAR DIFFERENTIAL EQUATIONS50

) 2aW0 + bW = Q:
As the three terms 2aW0 , bW and Q lie in one form, then we can use
this equation to compute all undetermined coe¢ cients in yp (x) = xW (x).

2.5.1 Lecture Examples


Solve the following second-order linear di¤erential equations, us-
ing the Method of Undetermined Coe¢ cients:
Problem 2.5.1 y 00 + 4y 0 12y = 18e3x :

Auxiliary Equation: m2 + 4m 12 = 0
) (m 2) (m + 6) = 0 ) m1 = 2; m2 = 6:
) Homogeneous solution is: yh = C1 e2x + C2 e 6x
:
) Particular solution takes the form: yp = Ae3x
) yp0 = 3Ae3x ; ) yp00 = 9Ae3x :
) yp00 + 4yp0 12yp = 9Ae3x = 18e3x )A= 2
3x
General Solution is yG:S: = yp + yh = 2e + C1 e2x + C2 e 6x
.

Problem 2.5.2 y 00 + 5y 0 14y = 27e2x :

Auxiliary Equation: m2 + 5m 14 = 0
) (m + 7) (m 2) = 0 ) m1 = 7; m2 = 2:
) Homogeneous solution is: yh = C1 e 7x
+ C2 e2x :

) Particular solution takes the form yp = Axe2x :


) yp = 2Axe + Ae ;
0 2x 2x

) yp00 = 4Axe2x + 2Ae2x + 2Ae2x = 4Axe2x + 4Ae2x


) yp00 + 5yp0 14yp = Ae2x (4x + 4 + 10x + 5 14x) = 9Ae2x = 27e2x :
) A = 3: ) yp = 3xe2x :
General Solution is yG:S: = yp + yh = 3xe2x + C1 e 7x + C2 e2x :

Problem 2.5.3 y 00 + 4y 0 + 3y = 10 cos x

Auxiliary Equation: m2 + 4m + 3 = 0
) (m + 3) (m + 1) = 0 ) m1 = 3; m2 = 1:
) Homogeneous solution is: yh = C1 e 3x
+ C2 e x :

) Particular solution takes the form: yp = A sin x + B cos x


CHAPTER 2. SECOND-ORDER LINEAR DIFFERENTIAL EQUATIONS51

) yp0 = A cos x B sin x


) yp00 = A sin x B cos x
) yp00 + 4yp0 + 3yp = A sin x B cos x + 4A cos x 4B sin x + 3A sin x +
3B cos x
= (2A 4B) sin x + (4A + 2B) cos x = 0 sin x + 10 cos x
) 2A 4B = 0; 4A + 2B = 10: ) A = 2B
) 10B = 10 )B=1 ) A = 2:
General Solution is yG:S: = yp + yh = 2 sin x + cos x + C1 e 3x + C2 e x .

Problem 2.5.4 y 00 + 4y = 8x2 :

Auxiliary Equation:
p mp2 + 4 = 0 p
) m1;2 = b b2 4ac
2a
= 0 0 16
2
= 6 2 144 = 24i = 2i = i
) =0 and =2
) yh = e (C1 sin ( x) + C2 cos ( x)) = C1 sin (2x) + C2 cos (2x) :
x

) Particular solution takes the form yp = Ax2 + Bx + C:


) 0
yp = 2Ax + B; 00
yp = 2A:
) yp + 4yp = 4Ax + 4Bx + (2A + 4C) = 8x2 + 0x + 0
00 2

) 4A = 8; 4B = 0; 2A + 4C = 0
) A = 2; B = 0; C= 1 ) yp = 2x2 1:
) yG:S: = yp + yh = 2x2 1 + C1 sin (2x) + C2 cos (2x) :

Problem 2.5.5 y 00 4y 0 + 13y = 50xe3x

Auxiliary Equation:
p mp2 4m + 13 p = 0
) m1;2 = b 2
b 4ac 4 16 52 4 36
2a
= 2
= 2
= 4 26i
) m1;2 = 2 3i = i ) Homogeneous solution is:
) yh = e x (C1 sin ( x) + C2 cos ( x)) = e2x (C1 sin (3x) + C2 cos (3x)) :
) Particular solution takes the form yp = (Ax + B) e3x :
) yp0 = (3Ax + A + 3B) e3x ; yp00 = (9Ax + 6A + 9B) e3x :
) yp00 4yp0 +13yp = (9Ax + 6A + 9B 12Ax 4A 12B + 13Ax + 13B) e3x
= 10Axe3x + (2A + 10B) e3x = 50xe3x + 0e3x
) 10A = 50; ) A = 5. Also, 2A + 10B = 0 )B= 1
) yp = (5x 1) e3x )General Solution is
yG:S: = yp + yh = (5x 1) e3x + e2x (C1 sin (3x) + C2 cos (3x)) :
CHAPTER 2. SECOND-ORDER LINEAR DIFFERENTIAL EQUATIONS52

2.5.2 Classroom Exercises


Problem 2.5.6 y 00 + 4y = 8 sin (2x) + 12 cos (2x)

Homogeneous solution yh = C1 sin (2x) + C2 cos (2x) :(Problem (2.5.4))


) Particular solution takes the form yp = Ax sin (2x) + Bx cos (2x) :
) yp0 = 2Ax cos (2x) 2Bx sin (2x) + A sin (2x) + B cos (2x) ;
yp00 = 4Ax sin (2x) 4Bx cos (2x)+2A cos (2x) 2B sin (2x)+2A cos (2x)
2B sin (2x)
= ( 4Ax 4B) sin (2x) + ( 4Bx + 4A) cos (2x)
) yp +4yp = ( 4Ax 4B) sin (2x)+( 4Bx + 4A) cos (2x)+4Ax sin (2x)+
00

4Bx cos (2x)


= 4B sin (2x) + 4A cos (2x) = 8 sin (2x) + 12 cos (2x)
) A = 3; B=2 ) yp = 3x sin (2x) + 2x cos (2x) :
General Solution is yG:S: = yp + yh = (3x + C1 ) sin (2x) + (2x + C2 ) cos (2x) :

Problem 2.5.7 y 00 3y 0 + 2y = 2e2x

Auxiliary Equation: m2 3m + 2 = 0
) (m 1) (m 2) = 0 ) m1 = 1; m2 = 2:
) Homogeneous solution is: x
yh = C1 e + C2 e2x :

) Particular solution takes the form yp = Axe2x :


) yp0 = 2Axe2x + Ae2x ;
) yp00 = 4Axe2x + 2Ae2x + 2Ae2x = 4Axe2x + 4Ae2x
) yp00 3yp0 + 2yp = Ae2x (4x + 4 6x 3 + 2x) = Ae2x = 2e2x :
) A = 2: ) yp = 2xe2x :
General Solution is yG:S: = yp + yh = 2xe2x + C1 ex + C2 e2x :

Problem 2.5.8 y 00 + 4y 0 + 4y = 6e 2x
:

Auxiliary Equation: m2 + 4m + 4 = 0
) (m + 2)2 = 0 ) m1 = m2 = 2:
) Homogeneous solution is: yh = C1 e 2x
+ C2 xe 2x
:
) Particular solution takes the form yp = Ax2 e 2x :
) yp = 2Ax e
0 2 2x
+ 2Axe 2x
= Ae 2x
( 2x2 + 2x) ;
) yp00 = 4Ax2 e 2x 4Axe 2x 4Axe 2x + 2Ae 2x = Ae 2x
(4x2 8x + 2)
CHAPTER 2. SECOND-ORDER LINEAR DIFFERENTIAL EQUATIONS53

) yp00 + 4yp0 + 4yp = Ae 2x


(4x2 8x + 2 8x2 + 8x + 4x2 ) = 2Ae 2x
=
2x
6e
) A = 3: ) yp = 3x2 e 2x
:
General Solution is yG:S: = yp + yh = 3x2 e 2x
+ (C1 + C2 x) e 2x
:

Problem 2.5.9 y 00 + 6y 0 + 45y = 81 sin (6x) + 18 cos (6x)

Auxiliary Equation:
p m2p+ 6m + 45 =p0
) m1;2 = b 2ab 4ac = 6 236 180 = 6 2 144 = 6 2 12i
2

) m1;2 = 3 6i = i ) = 3 and =6
) Homogeneous solution is:
yh = e x (C1 sin ( x) + C2 cos ( x)) = e 3x (C1 sin (6x) + C2 cos (6x)) :
) Particular solution takes the form yp = A sin (6x) + B cos (6x) :
) yp = 6A cos (6x) 6B sin (6x) ;
0 00
yp = 36A sin (6x) 36B cos (6x) :
) yp00 +6yp0 +45yp = ( 36A 36B + 45A) sin (6x)+( 36B + 36A + 45B) cos (6x)
= (9A 36B) sin (6x)+(36A + 9B) cos (6x) = 81 sin (6x)+18 cos (6x)
) 9A 36B = 81; 36A + 9B = 18 ) A = 1. and B = 2
) yp = sin (6x) 2 cos (6x) : General Solution is
yG:S: = yp + yh = sin (6x) 2 cos (6x) + e 3x (C1 sin (6x) + C2 cos (6x)) :

Problem 2.5.10 y 00 6y 0 + 9y = (4 + 4x) ex ; y (0) = 1; y 0 (0) = 1

Auxiliary Equation: m2 6m + 9 = 0
) (m 3) = 0
2
) m1 = m2 = 3:
) Homogeneous solution is: yh = (C1 + C2 x) e3x :
) Particular solution takes the form: yp = (A + Bx) ex
) yp0 = (A + B + Bx) ex
) yp00 = (A + 2B + Bx) ex
) yp00 6yp0 + 9yp = (A + 2B + Bx 6A 6B 6Bx + 9A + 9Bx) ex
= (4A 4B) ex + 4Bxex = 4ex + 4xex
) 4A 4B = 4; 4B = 4: )B=1 ) A = 2:
General Solution is yG:S: = yp + yh = (2 + x) ex + (C1 + C2 x) e3x .
y (0) = 2 + C1 = 1 ) C1 = 3
y (x) = (3 + x) e + (2C1 + C2 + 2C2 x) e3x
0 x

y 0 (0) = 3 + 2C1 + C2 = 1 ) C4 = 4
) Solution is y (x) = (2 + x) ex + (4x 3) e3x
CHAPTER 2. SECOND-ORDER LINEAR DIFFERENTIAL EQUATIONS54

Problem 2.5.11 y 00 8y 0 + 25y = 75x2 23x 52

Auxiliary Equation:
p mp2 8m + 25p= 0
) m1;2 = b b2 4ac
2a
= 8 64 100
2
= 8 2 36 = 8 26i
) m1;2 = 4 3i = i ) Homogeneous solution is:
) yh = e (C1 sin ( x) + C2 cos ( x)) = e 4x (C1 sin (3x) + C2 cos (3x)) :
x

) Particular solution takes the form yp = Ax2 + Bx + C:


) yp0 = 2Ax + B; yp00 = 2A:
) yp 8yp + 25yp = 2A 16Ax 8B + 25Ax2 + 25Bx + 25C
00 0

= 25Ax2 + ( 16A + 25B) x + (2A 8B + 25C) = 75x2 23x 52


) 25A = 75; ) A = 3;
16A + 25B = 23 ) 25B = 16A 23 = 25 ) B = 1;
2A 8B + 25C = 52 ) 25C = 2A + 8B 52 = 6 + 8 52 = 50
)C= 2 General Solution is
yG:S: = yp + yh = 3x2 + x 2+e 4x
(C1 sin (3x) + C2 cos (3x)) :

Problem 2.5.12 y 00 2y 0 = 6x2 6x 8

Auxiliary Equation: m2 2m = 0
) m (m 2) = 0 ) m1 = 0; m2 = 2:
) Homogeneous solution is: yh = C1 + C2 e2x :
) Particular solution takes the form: yp = x (Ax2 + Bx + C) = Ax3 +
2
Bx + Cx
) yp0 = 3Ax2 + 2Bx + C: ) yp00 = 6Ax + 2B
) yp00 2yp0 = 6Ax + 2B 2 (3Ax2 + 2Bx + C)
= 6Ax2 + (6A 4B) x + (2B 2C) = 6x2 6x 8
) A = 1; 6A 4B = 6; ) B = 3;
2B 2C = 8 ) C = 7:
General Solution is yG:S: = yp + yh = x3 + 3x2 + 7x + C1 + C2 e2x .
CHAPTER 2. SECOND-ORDER LINEAR DIFFERENTIAL EQUATIONS55

2.5.3 Revision Problems


Problem 2.5.13 y 00 3y 0 + 2y = 0

Auxiliary Equation: m2 3m + 2 = 0
) (m 1) (m 2) = 0 ) m1 = 1; m2 = 2:
) General solution is: x
yh = C1 e + C2 e2x :

Problem 2.5.14 y 00 6y 0 + 9y = 0

Auxiliary Equation: m2 6m + 9 = 0
) (m 3) = 0
2
) m1 = m2 = 3:
) General solution is: yh = (C1 + C2 x) e3x :

Problem 2.5.15 y 00 + 8y 0 + 25y = 0

Auxiliary Equation:
p m2p+ 8m + 25 =p0
) m1;2 = b b2 4ac
2a
= 8 264 100 = 8 2 36 = 82 6i
) m1;2 = 4 3i = i ) = 4 and =3
x
and yh = e (C1 sin ( x) + C2 cos ( x)) :
) General solution is: yh = e 4x (C1 sin (3x) + C2 cos (3x)) :

Problem 2.5.16 y 00 4y = 16e2x

Auxiliary Equation: m2 4 = 0
) (m 2) (m + 2) = 0 ) m1 = 2; m2 = 2:
) Homogeneous solution is: yh = C1 e + C2 e 2x :
2x

) Particular solution takes the form: yp = Axe2x


) yp0 = (2x + 1) Ae2x ) yp00 = (4x + 4) Ae2x
) yp00 4yp = (4x + 4 4x) Ae2x = 4Ae2x = 16e2x :
) A = 4: ) yp = 4xe2x :
General Solution is yG:S: = yp + yh = (4x + C1 ) e2x + C2 e 2x
.
CHAPTER 2. SECOND-ORDER LINEAR DIFFERENTIAL EQUATIONS56

Problem 2.5.17 y 00 + 5y 0 + 6y = 2e 3x
; y (0) = 1; y 0 (0) = 4:
Auxiliary Equation: m2 + 5m + 6 = 0
) (m + 3) (m + 2) = 0 ) m1 = 3; m2 = 2:
) Homogeneous solution is: yh = C1 e 3x
+ C2 e 2x :
) Particular solution takes the form yp = Axe 3x :
) yp = 3Axe
0 3x
+ Ae ;3x

) yp = 9Axe
00 3x
3Ae 3x 3Ae 3x = 9Axe 3x 6Ae 3x
) yp + 5yp + 6yp = Ae 3x (9x 6 15x + 5 + 6x) = Ae
00 0 3x
= 2e 3x
:
) A = 2: ) yp = 2xe 3x :
3x 3x 2x
General Solution is yG:S: = yp + yh = 2xe + C1 e + C2 e :
Using the two given initial conditions, we deduce that C1 = 4 and C2 =
3:
) y (x) = 2xe 3x
+ 4e 3x
3e 2x
.

Problem 2.5.18 y 00 + y = 6 sin (x) 2 cos (x)


p
Auxiliary Equation: m2 + 1 = 0: ) m1;2 = 1= i= i
) yh = e (C1 sin ( x) + C2 cos ( x)) = C1 sin (x) + C2 cos (x) :
x

) Form of particular solution: yp = Ax sin (x) + Bx cos (x) : ) yp0 =


Ax cos (x) Bx sin (x) + A sin (x) + B cos (x) ;
yp00 = Ax sin (x) Bx cos (x)+A cos (x) B sin (x)+A cos (x) B sin (x)
= ( Ax 2B) sin (x) + ( Bx + 2A) cos (x)
) yp00 + yp = 2B sin (x) + 2A cos (x) = 6 sin (x) 2 cos x
) A = 1; B= 3 ) yp = x sin (x) 3x cos (x) : ) yG:S: =
yp + yh = ( x + C1 ) sin x + ( 3x + C2 ) cos x:

Problem 2.5.19 y 00 + y 0 6y = (14x 5) e4x


Auxiliary Equation: m2 + m 6 = 0: ) (m + 3) (m 2) = 0
) m1 = 3; m2 = 2: ) yh = C1 e 3x
+ C2 e2x :
) Particular solution takes the form yp = (Ax + B) e4x :
) yp0 = (4Ax + A + 4B) e4x ; yp00 = (16Ax + 4A + 16B + 4A) e4x :
) yp + yp 6yp = (16Ax + 8A + 16B + 4Ax + A + 4B 6Ax 6B) e4x
00 0

= 14Axe4x + (9A + 14B) e4x = 14xe4x 5e4x


) 14A = 14; ) A = 1. Also, 9A + 14B = 5 )B= 1
) yG:S: = yp + yh = (x 1) e4x + C1 e 3x
+ C2 e2x .
CHAPTER 2. SECOND-ORDER LINEAR DIFFERENTIAL EQUATIONS57

In the next three problems, we solve a second-order linear di¤erential


equation by means of the three methods of Variation of Parameters, Unde-
termined Coe¢ cients and Order Reduction.

Problem 2.5.20 Find the general solution of the following second-order


linear di¤erential equation, using the Method of Variation of Parameters:
y 00 + 9y = 27 cos2 3x 9:

Auxiliary Equation:
p m2 + 9 = 0 ) m2 = 9
) m1;2 = 9 = 3i = i
) =0 and =3
) yh = e (C1 sin x + C2 cos x) = C1 sin 3x + C2 cos 3x :
x

We take y1 (x) = sin 3x and y2 (x) = cos 3x; and write


yp (x) = V1 y1 + V2 y2 = V1 sin 3x + V2 cos 3x:
We solve the following two equations:
y1 V10 + y2 V20 = V10 sin 3x + V20 cos 3x = 0:
y10 V10 + y20 V20 = V10 cos 3x V20 sin 3x = Q (x) = 27 cos2 3x 9:
0 y2 0 cos 3x
Q (x) y20 27 cos2 3x 9 3 sin 3x 9 cos 3x 27 cos3 3x
V10 = = = 3 sin2 3x 3 cos2 x
y1 y2 sin 3x cos 3x
y10 y20 3 cos 3x 3 sin 3x
3(9 cos3 3x 3 cos 3x)
= 3
= 9 cos3 3x 3 cos 3x = 9 cos 3x 1 sin2 3x 3 cos 3x
) V10 = Z6 cos 3x 9Zsin2 3x cos 3x
) V1 = V10 dx = 6 cos 3x 9 sin2 3x cos 3x dx = 2 sin 3x sin3 3x:
y1 0 y y sin 3x 0
V20 = = 10 20 = = ( 3)
y10 Q (x) y1 y2 3 cos 3x 27 cos2 3x 9
) V20 = Z27 sin 3x cos 33xZ 9 sin 3x = 3 sin 3x 9 sin 3x cos2 3x:
2

) V2 = V10 dx = 3 sin 3x 9 sin 3x cos2 3x dx = cos 3x + cos3 3x


Consequently, yp (x) = V1 y1 + V2 y2 = V1 sin 3x + V2 cos 3x
= 2 sin2 3x sin4 3x cos2 3x + cos4 3x
2
= 2 sin2 3x sin4 3x 1 sin2 3x + 1 sin2 3x
= 2 sin2 3x sin4 3x 1 + sin2 3x + 1 2 sin2 3x + sin4 3x
) yp (x) = sin2 3x: Finally,

) yG:S: = yp + yh = sin2 3x + C1 sin 3x + C2 cos 3x:


CHAPTER 2. SECOND-ORDER LINEAR DIFFERENTIAL EQUATIONS58

Problem 2.5.21 Find the general solution of the following second-order


linear di¤erential equation, using the Method of Undetermined Coe¢ cients:
y 00 + 9y = 27 cos2 3x 9:

Problem 2.5.22 Find the general solution of the following second-order


linear di¤erential equation, using the Method of Order Reduction:
y 00 + 9y = 27 cos2 3x 9:

Auxiliary Equation:
p m2 + 9 = 0 ) m2 = 9
) m1;2 = 9 = 3i = i
) =0 and =3
) yh = e x (C1 sin x + C2 cos x) = C1 sin 3x + C2 cos 3x :
We take y1 (x) = sin 3x ) y 0 = 3 cos 3x; ) y 00 = 9 sin 3x:
y(x)
and let U (x) = y1 (x)
;
) y = U:y1 = U: sin 3x; y 0 = U:y10 + U 0 y1 = 3U cos 3x + U 0 sin 3x
) y" = U:y1 :U + 2U :y1 + U ":y1 = 9U sin 3x + 6U 0 cos 3x + U 00 sin 3x:
00 0 0

When we substitute in the given d.e., we obtain:


y 00 + 9y = 9U sin 3x + 6U 0 cos 3x + U 00 sin 3x + 9U sin 3x = 1 3 cos2 3x:
) 6U 0 cos 3x + U 00 sin 3x = 27 cos2 3x 9:
and we obtain U 00 + 6 (cot 3x) U 0 = 27 cos2 3x csc 3x 9 csc 3x:
This is a …rst-order linear di¤erential equation in the unknown function
U 0,
with p = 6 cot 3x , Q = csc 3x 3 cos2 3x csc 3x and Integrating factor:
R R

= e2 ln(sin 3x) = eln( 3x) = sin2 3x:


sin2
p dx 6 cot 3x dx
=e =e
R R
) U0 = 1
q dx + C = csc2 3x 27 cos2 3x: sin 3x 9 sin 3xdx + C
) U 0 = csc2 3x ( 3 cos3 3x + 3 cos 3x + C) = 3 csc2 3x: cos 3x ( cos2 3x + 1)+
C csc2 3x:
) U 0 = R3 csc2 3x: cos 3x: sin 2 2 2
R 3x + C csc R3x =23 cos 3x + C csc 3x
) U = U dx + K = 3 cos 3xdx + C csc 3xdx + K
0

) U = sin 3x C3 cot 3x + K:

) yG:S: = y1 U = sin2 3x + K sin 3x + C1 cos 3x:


CHAPTER 2. SECOND-ORDER LINEAR DIFFERENTIAL EQUATIONS59

Problem 2.5.23 y 00 7y 0 + 12y = 12x2 14x + 26:

Final Answer: y = x2 + 2 + C1 e4x + C2 e3x :

Problem 2.5.24 y 00 + 6y 0 27y = 81x2 63x + 27:

Final Answer: y= 1+x 3x2 + C1 e3x + C2 e 9x


:

Problem 2.5.25 y 00 + 4y 0 + 5y = 10ex :

Final Answer: y = ex + C1 e 2x
sin x + C2 e 2x
cos x:

Problem 2.5.26 y 00 6y 0 + 5y = 4ex :

Final Answer: y = xex + C1 ex + C2 e5x :

Problem 2.5.27 y 00 5y 0 + 4y = 20 sin 2x:

Final Answer: y = 2 cos 2x + C1 ex + C2 e4x :

Problem 2.5.28 y 00 + 2y 0 3y = 130 sin x + 10 cos x:

Final Answer: y = 11 cos x + 27 sin x + C1 ex + C2 e 3x


:

Problem 2.5.29 y 00 + 2y 0 + 5y = 3 sin 2x + 12 cos 2x:


x
Final Answer: y = 3 sin 2x + C1 (cos 2x) e + C2 (sin 2x) e x :

Problem 2.5.30 y 00 = 220x9 112x6 + 12x2 :

Final Answer: y = 2x11 2x8 + x4 + C1 x + C2 :

Problem 2.5.31 y 00 + 2y 0 15y = 8e3x .

Final Answer: y = xe3x + C1 e3x + C2 e 5x


:
CHAPTER 2. SECOND-ORDER LINEAR DIFFERENTIAL EQUATIONS60

Problem 2.5.32 In an RLC-circuit, the current I (t) is governed by the


equation Z
dI 1
L + RI + I dt = E (t) :
dt C
Di¤erentiating, we get the di¤erential equation:

d2 I dI 1 dE (t)
L + R + I = :
dt2 dt C dt
1
Find I (t) in the case L = 1, R = 8, C = 25
, E = 48 sin 2t 63 cos 2t,
and initially I = I 0 = 0.
4t
Final Answer: I (t) = 4e sin 3t + 6 sin 2t:

Problem 2.5.33 Solve the following second-order linear di¤erential


equations, using the Method of Undetermined Coe¢ cients:

1. y 00 + y = 6 cos x:

2. y 00 y = 6 cos x:

3. y 00 + y 0 = 6 cos x:

4. y 00 + 8y0 + 25y = 80 cos 5x:

5. y 00 2y 0 + y = 12xe3x , Solution is: yG:S: = C1 ex + C2 xex 3e3x + 3xe3x :

6. y 00 2y 0 + y = 12xex , Solution is: yG:S: = C1 ex + C2 xex + 2x3 ex :

7. y 00 2y 0 + y = 2 cos x :

8. y 00 14y 0 + 45y = 4e5x :

9. y 00 + 6y 0 + 13y = 10e2x + 29xe2x :

10. y 00 2y 0 + 10y = 50xe2x :

11. y 00 4y 0 + 4y = xe3x :

12. y 00 + 2y 0 15y = 8e3x .


(Compare your solution with that of Probem 2.4.9.)
CHAPTER 2. SECOND-ORDER LINEAR DIFFERENTIAL EQUATIONS61

2.6 Solution of Cauchy-Euler Di¤erential


Equations
Leonhard Euler (15 April 1707 Basel, Switzerland - 18 September 1783).
Augustin-Louis Cauchy (21 August 1789 Paris, France - 23 May 1857).

2.6.1 Description and Method of Solution


A Cauchy-Euler Di¤erential Equation is a linear di¤erential equation that
takes the form:
an xn y (n) + an 1 xn 1 y (n 1)
+ + a2 x2 y 00 + a1 xy 0 + a0 y = f (x) ; (2.29)
dy d2 y dk y
where y 0 = dx
; y" = dx2
and y (k) = dxk
.

Theorem 6 (Cauchy-Euler) When we use the substitution


x = et ; (2.30)
that is,
t = ln x; (2.31)
and we let be the following di¤erentail operator:
d
= ; (2.32)
dt
we …nd that, for all natural numbers r :
!
Y
r 1
r (r)
xy = ( k) y = ( 1) ( 2) ( (r 1)) y: (2.33)
k=0

Proof. For any continuously di¤erentiable function z (x) we have, by Chain


Rule:
dz det
z = dz(x(t))
dt
dz dx
= dx : dt = dx dz t
: dt = dx e:
dz
) z=x : (2.34)
dx
Induction hypothesis: For a natural number r :
!
Y
r 1
xr y (r) = ( k) y: (2.35)
k=0
CHAPTER 2. SECOND-ORDER LINEAR DIFFERENTIAL EQUATIONS62

This hypothesis is true for r = 1; by (2.34).


Suppose this hypothesis is true for some natural number r . We then …nd
that
Qr rQ1
We then have: ( k) y = ( r) ( k) y
k=0 k=0
=( r) xr y (r) (by hypothesis (2.35))
r (r) r (r)
= xy r xy
d r (r)
= x dx x y rxr y (r) using (2.34)
d r d (r)
= x dx x y (r) + xr dx y rxr y (r)
= x rxr 1 y (r) + xr y (r+1) rxr y (r)
= xr+1 y (r+1) :
This establishes that whenever the induction hypothesis (2.35) is true for
some r, it remains true for r + 1. (This is called the induction step.)
As this hypothesis is true for r = 1; by (2.34), then it must be true for
all positive integers r: This completes the proof of (2.33).

Now, by using the substitutions (2.30), and its consequence (2.33), the
Cauchy-Euler di¤erential equation (2.29) becomes a linear di¤erential
equation with constant coe¢ cients.

Instances of (2.33) are:

xy 0 = y;
2
x2 y 00 = ( 1) y = y; (2.36)
3 2
x3 y 000 = ( 1) ( 2) y = 3 +2 y:
CHAPTER 2. SECOND-ORDER LINEAR DIFFERENTIAL EQUATIONS63

2.6.2 Lecture Examples


Find the general solutions of the following di¤erential equations:

Problem 2.6.1 x2 y 00 4xy 0 + 4y = 10 sin (ln (x2 )) :

This is a Cauchy-Euler di¤erential equation.


Let x = et . Then ln x = t and
xy 0 = y; x2 y 00 = ( 1) y = 2 y:
This equation becomes the follwing di¤erential equation in the unkown
function y (t) :
2
y 4 y + 4y = 2 5 + 4 y = 10 sin (2t) :
Auxiliary Equation: m2 5m + 4 = 0
) (m 1) (m 4) = 0 ) m1 = 1; m2 = 4:
) Homogeneous solution is: yh (t) = C1 et + C2 e4t :

) Particular solution takes the form yp (t) = A sin (2t) + B cos (2t) :
) yp = 2B sin (2t) + 2A cos (2t)
) 2 yp = 4A sin (2t) 4B cos (2t)
) 2 5 + 4 yp = 10B sin (2t) 10A cos (2t) = 10 sin (2t)
) A = 0; B = 1: ) yp (t) = cos (2t) :
General Solution is yG:S: (t) = yp + yh = cos (2t) + C1 et + C2 e4t .
As a function in x; this general solution becomes
yG:S: (x) = cos (2 ln x) + C1 x + C2 x4 :

2.6.3 Classroom Exercises


18
Problem 2.6.2 x2 y 00 + 9xy 0 + 25y = x4

This is a Cauchy-Euler di¤erential equation.


Let x = et . Then t = ln x and
xy 0 = y; x2 y 00 = ( 1) y = 2 y:
This equation becomes the follwing di¤erential equation in the unkown
function y (t) :
2
y + 9 y + 25y
2
= + 8 + 25 y = 18e 4t :
Auxiliary Equation:
p m2p+ 8m + 25 =p0
) m1;2 = b b2 4ac
2a
= 8 264 100 = 8 2 36 = 82 6i
) m1;2 = 4 3i = i ) = 4 and =3
CHAPTER 2. SECOND-ORDER LINEAR DIFFERENTIAL EQUATIONS64

and yh = e t (C1 sin ( t) + C2 cos ( t)) :


yh (t) = e 4t (C1 sin (3t) + C2 cos (3t)) :
) Particular solution takes the form: yp = Ae 4t
) yp = 4Ae 4t
) 2 yp = 16Ae 4t
) 2 + 8 + 25 yp = Ae 4t (16 32 + 25) = 9Ae 4t = 18e 4t :
) A = 2: ) yp (t) = 2e 4t :
General Solution is yG:S: (t) = yp + yh = 2e 4t + e 4t (C1 sin (3t) + C2 cos (3t)) .
As a function in x; this general solution becomes
yG:S: (x) = x14 (2 + C1 sin (3 ln x) + C2 cos (3 ln x)) :

Problem 2.6.3 x2 y 00 5xy 0 + 9y = 36 cos (3 ln x)

This is a Cauchy-Euler di¤erential equation.


Let x = et . Then ln x = t and
xy 0 = y; x2 y 00 = ( 1) y = 2 y:
This equation becomes the follwing di¤erential equation in the unkown
function y (t) :
2
y 5 y + 9y
2
= 6 + 9 y = 36 cos 3t:
Auxiliary Equation: m2 6m + 9 = 0
) (m 3) = 02
) m1 = m2 = 3:
) Homogeneous solution is: yh (t) = C1 e3t + C2 te3t :
) Particular solution takes the form: yp = A sin 3t + B cos 3t
) yp = 3A cos 3t 3B sin 3t
) 2 yp = 9A sin 3t 9B cos 3t
) 2 6 + 9 yp = 18A cos 3x + 18B sin 3x = 36 cos 3t
) A = 2; B = 0: ) yp (t) = 2 sin 3t:
General Solution is yG:S: (t) = yp + yh = 2 sin 3t + C1 e3t + C2 te3t .
As a function in x; this general solution becomes
yG:S: (x) = 2 sin (3 ln x) + C1 x3 + C2 x3 ln x:
CHAPTER 2. SECOND-ORDER LINEAR DIFFERENTIAL EQUATIONS65

2.6.4 Revision Problems


Find the general solutions of the following di¤erential equations:

Problem 2.6.4 x2 y 00 2xy 0 + 2y = 5x2 :

This is a Cauchy-Euler di¤erential equation.


Let x = et . Then t = ln x and
xy 0 = y; x2 y 00 = ( 1) y = 2 y:
This equation becomes the follwing di¤erential equation in the unkown
function y (t) :
2
y 2 y + 2y
2
= 3 + 2 y = 5e2t :
Auxiliary Equation: m2 3m + 2 = 0
) (m 1) (m 2) = 0 ) m1 = 1; m2 = 2:
) yh = C1 e + C2 e :
t 2t

) Particular solution takes the form: yp = Ate2t


) yp = 2Ate2t + Ae2t
) 2 yp = 4Ate2t + 4Ae2t
) 2 3 + 2 yp = Ae2t (4t + 4 6t 3 + 2t) = Ae2t = 5e2t :
) A = 5; yp (t) = 5te2t :

) General solution is:: yG:S: (t) = yp + yh = (5t + C2 ) e2t + C1 et :


As a function in x; this general solution becomes
yG:S: (x) = (C2 + 5 ln x) x2 + C1 x:

Problem 2.6.5 x2 y 00 5xy 0 + 9y = 6x3 :

This is a Cauchy-Euler di¤erential equation.


Let x = et . Then t = ln x and
xy 0 = y; x2 y 00 = ( 1) y = 2 y:
This equation becomes the follwing di¤erential equation in the unkown
function y (t) :
2
y 5 y + 9y
2
= 6 + 9 y = 6e3t :
Auxiliary Equation: m2 6m + 9 = 0
) (m 3) = 02
) m1 = m2 = 3:
) yh (t) = e (C1 + C2 t) :
3t

) Particular solution takes the form: yp = At2 e3t


CHAPTER 2. SECOND-ORDER LINEAR DIFFERENTIAL EQUATIONS66

) yp = 3At2 e3t + 2Ate3t


) 2 yp = 9At2 e3t + 12Ate3t + 2Ae3t
) 2 6 + 9 yp = Ae3t (9t2 + 12t + 2 18t2 12t + 9t2 ) = 2Ae3t =
6e3t
) A = 3; yp (t) = 3t2 e3t :
) General solution is:: yG:S: (t) = yp (t) + yh (t) = e3t (C1 + C2 t + 3t2 )
As a function in x; this general solution becomes
yG:S: (x) = x3 C1 + C2 ln x + 3 (ln x)2 :

Problem 2.6.6 x2 y 000 + xy 00 4y 0 = 24x2 :


We mulitply both sides by x; in order to get the following Cauchy-Euler
di¤erential equation:
x3 y 000 + x2 y 00 4xy 0 = 24x3 :
Let x = et . Then xy 0 = y; x2 y 00 = ( 1) y = 2 y; and
3 000 3 2
xy = ( 1) ( 2) y = 3 + 2 y:
This equation becomes the follwing di¤erential equation in the unkown
function y (t) :
3
3 2+2 y+ 2 y 4 y = 24e3t :
) 3
2 2 3t
3 y = 24e ; that is,
2
2 3 V = 24e3t ;
where V = dydt
:
Auxiliary Equation: m2 2m 3 = 0
) (m + 1) (m 3) = 0 ) m1 = 1; m2 = 3:
) Homogeneous solution is: Vh = C1 e t
+ C2 e3t :
) Particular solution takes the form Vp = Ate3t :
) Vp = 3Ate3t + Ae3t :
) 2 Vp = 9Ate3t + 3Ae3t + 3Ae3t = 9Ate3t + 6Ae3t :
) 2 2 3 Vp = Ae3t (9t + 6 6t 2 3t) = 4Ae3t = 24e3t :
) A = 6: ) Vp = 6te3t :
) dydt
G:S:
= VG:S: = Vp + Vh = 6te3t + C2 e3t + C1 e t
R dyG:S: R
) yG:S:
R (t) = dt
dt = (6te3t + C2 e3t + C1 e t ) dt
3t 3t C2 3t 2 C2
= 2te 2 e dt + 3 e C1 e = 2t 3 + 3 e3t C1 e t :
t

) yG:S: (t) = (K2 + 2t) e3t + K1 e t :


As a function in x; this general solution becomes
yG:S: (x) = x3 (K2 + 2 ln x) + Kx1 :
CHAPTER 2. SECOND-ORDER LINEAR DIFFERENTIAL EQUATIONS67

Problem 2.6.7 Find the general solution of the following linear


di¤erential equation:

d2 y dy
x2 2
+ x + 9y = 12 sin (3 ln x) + 6 cos (3 ln x) :
dx dx
This is a Cauchy-Euler di¤erential equation.
Let x = et . Then ln x = t and
2
xy 0 = y; x2 y 00 = ( 1) y = y y:
d2 y
) x2 dx2 + x dx
dy
+ 9y = 2
y y + y + 9y = 2
y + 9y
The equation becomes the follwing di¤erential equation in the unkown
function y (t) :
2
y + 9y = 12 cos (3t) :
Auxiliary Equation: m2 + 9 = 0: ) m2 = 9
p
) m1;2 = 9 = 0 3i = i
) with = 0 and =3
Homogeneous solution is: yh (t) = C1 sin 3t + C2 cos 3t:

) Particular solution takes the form yp (t) = At sin 3t + Bt cos 3t :


) yp = A (3t cos 3t + sin 3t) + B ( 3t sin 3t + cos 3t) ;
2
yp = A ( 9t sin 3t + 6 cos 3t) + B ( 9t cos 3t 6 sin 3t)
) 2
yp + 9yp
= 9At sin 3t + 6A cos 3t 9Bt cos 3t 6B sin 3t + 9At sin 3t + 9Bt cos 3t
= 6A cos 3t 6B sin 3t = 12 sin 3t + 6 cos 3t
) A = 1; B= 2
) yp (t) = t sin 3t 2t cos 3t:
Therefore, yG:S: (t) = yp (t) + yh (t) = t sin 3t 2t cos 3t + C1 sin 3t +
C2 cos 3t
= (C1 + t) sin 3t + (C2 2t) cos 3t:
) yG:S: (x) = (C1 + ln x) sin (3 ln x) + (C2 2 ln x) cos (3 ln x) :
Chapter 3

The Laplace Transform

3.1 De…nition and Properties


De…nition. Let f (t) be a real function on the real line R, which satis…es
the following three conditions:
(i) f (t) = 0 for all negative
R t values of t,
(ii) All de…nite integrals 0 f (t) dt exist, for all (…nite) values of t ,
and
(iii) f (t) has an exponential order s . (This means that, starting from
some constant value t of the variable t, the inequality jf (t)j es t holds.)
Then the Laplace transform of f (t) is another function L ff g ; of one real
variable s, de…ned for all s > s by
Z 1
L ff g (s) = e st f (t) dt.
0

In what follows, the unit step function U (t) is a real function on R,


de…ned by
0 when t < 0
U (t) = :
1 when t 0
The Dirac function (t) is a distribution, de…ned on the real line R by
the condition that Z t
(z) dz = U (t) ;
0

for all values of the variable t. It follows that (t) = 0 for all t < 0 and all
t > 0. This (t) is an idealization of a very thin pulse that starts at t = 0
and has area 1.

68
CHAPTER 3. THE LAPLACE TRANSFORM 69

Basic Properties of the Laplace Transform:


1. L ff g (s) exists at all s > s .

2. The Laplace transform is linear; that is L ff gg = L ff g L fgg


and L fcf g = cL ff g when c is a constant.

3. The Laplace transform is injective (one-to-one); that is, distinct func-


tions have distinct Laplace transforms. Our need for this important
property is the main reason why Laplace transforms must be functions.

4. lim L ff g (s) = 0:
s!1

5. lim sL ff g (s) = f (0) :


s!1

Rules of Laplace Transform


We denote L ff g (s) by F (s), and we assume s > s :
The two symbols k; b denote real constants:

n o
dF (s) df (t)
L ft f (t)g = ds
L dt
= sF (s) f (0)
n o
dn F d2 f (t)
L ftn f (t)g = ( 1)n dsn
L dt2
= s2 F (s) sf (0) f 0 (0)

R1 nR o
t
L 1t f (t) = s F (v) dv; L 0
f (z) dz = 1s F (s)
whenever one of them exists.

Shift-in-s Theorem: (s > s + k) Shift-in-t Theorem: (b > 0)

L ekt f (t) = F (s k) = F (s)js !(s k)


L ff (t b) U (t b)g = e bs
F (s)
CHAPTER 3. THE LAPLACE TRANSFORM 70

Proof of Rules of Laplace Transform.


d d
R 1 st R1 @e st
Rule 1: ds (L
R1 ff g) = ds R 01
e f (t) dt = 0 @s
f (t) dt
= 0
t e st f (t) dt = 0 e st tf (t) dt = L ft f (t)g :
) d
ds
(L ff g) = L ftf g : Consequently,
n d d d
L ft f g = L ft (t ( (tf )))g = ds ds ds
(L ff g)
n dn
= ( 1) dsn (L ff g) :
n o R R 1 @e st
1 t=1
Rule 2: L dfdt(t) = 0 e st dfdt(t) dt = [e st f (t)]t=0 0 @t
f (t) dt
(integration by parts)
R1
=0 f (0) + s 0 e st f (t) dt (because s > s )
n o
) L dfdt(t) = sF (s) f (0) : Applying this rule twice, we get
n 2 o n o n o
L d dtf 2(t) = L dtd dfdt(t) = sL dfdt(t) f 0 (0)
= s (sF f (0)) f 0 (0) = s2 F sf (0) f 0 (0) :
R1
Rule 3: Since lim s F (v) dv = 0 then there is a function g (t) such that
s!1 R
1
L fgg (s) = s F (v) dv.
d
R1
Hence by Rule 1, L ftg (t)g (s) = ds s
F (v) dv = F (s) = L ff g (s) :
Consequently, tg = f and so g = 1t f: This means that
R1
L 1t f (t) = s F (v) dv.
d
Rt
Rule 4: Since f (t) = dt 0
f (z) dz , then
n Rt o
d
F (s) = L f f (t)g = L 0
f (z) dz dt
nR R0 o
t
= sL 0 f (z) dz 0
f (z) dz (by Rule 2). So,
nR o nR o
t t
F (s) = sL 0 f (z) dz 0; that is L 0 f (z) dz = 1s F (s) :
R1 R1
Rule 5: L ekt f (t) = 0
e st
ekt f (t) dt = 0
e (s k)t
f (t) dt = F (s k) :
R 1 st
Rule 6: L ff (t b) U (t b)g = e f (t b) U (t b) dt
R b st R01 st
= e f (t b) U (t b) dt + e f (t b) U (t b) dt
Rb 0R
1 R 1
b
= 0 0dt + b e st f (t b) dt = 0 + e bs b e s(t b) f (t b) dt:
Now we use the substitution z = t b, and …nd that
R1
L ff (t b) U (t b)g = e bs 0 e sz
f (z) dz = e bs
L ff g :
CHAPTER 3. THE LAPLACE TRANSFORM 71

Famous Laplace Transforms


f (t) F (s) =L ff g (s) f (t) F (s) =L ff g (s)

1 !
1 = U (t) s
(s > 0) sin (!t) s2 +! 2
(s > 0)
1
t s2
(s > 0)
2 s
t2 s3
(s > 0) cos (!t) s2 +! 2
(s > 0)
6
t3 s4
(s > 0)
24 k
t4 s5
(s > 0) sinh (kt) s2 k 2
(s > jkj)

n! s
tn (n = 0; 1; 2; ) sn+1
(s > 0) cosh (kt) s2 k 2
(s > jkj)

1
ekt s k
(s > k) (t) 1

Derivation of Famous Laplace Transforms.


R1 t=1
1. L f1g = 0 e st 1 dt = s1 e st t=0 = 0 + 1s (when s > 0):
dn dn
2. L ftn g = L ftn 1g = ( 1)n dsn
(L f1g) = ( 1)n dsn
1
s
= n!
sn+1
:
1 1
3. L ekt = L ekt 1 = L f1gjs !(s k)
= s js
= s k
(using
!(s k)
Rule 5, and taking s > k).
Z1
st
4. whenever s > 0 : L fsin (!t)g = e sin (!t) dt
0
h it=1
s sin(!t) ! cos(!t) s sin(0)+! cos(0) !
= s2 +! 2
e st =0+ s2 +! 2
= s2 +! 2
: :
t=0
h it=1
! sin(!t) s cos(!t) s
Similarly, L fcos (!t)g = s2 +! 2
e st = s2 +! 2
:
t=0

5. L fsinh (kt)g = 21 L ekt e kt


= 1
2
1
s k
1
s+k
= k
s2 k 2
;
L fcosh (kt)g = 12 L ekt + e kt
= 1
2
1
s k
+ 1
s+k
= s
s2 k 2
;
whenever s > k and s > k; that is s > jkj :
Z1 Z1 Z1
st s0
6. L f g = e (t) dt = e (t) dt = (t) dt = U (1) = 1:
0 0 0
Alternatively, the de…nition of the Dirac function (t) stipulates that
Rt
U (t) = 0 (z) n dz: We combine
o this with Rule 4, and …nd that
1
Rt 1
s
= L fU (t)g = L 0 (z) dz = s L f g : Therefore, L f g = 1:
CHAPTER 3. THE LAPLACE TRANSFORM 72

3.2 Problems on Direct Laplace Transform


Find the Laplace transforms of the following functions:

3.2.1 Lecture Examples


Problem 3.2.1 t sin 3t + 8t3 4e2t

L (sin 3t) = s23+9


) L (t sin 3t + 8t3 4e2t ) = d
ds
L (sin 3t) + L (8t3 4e2t )
d 3
= ds s2 +9
+ 8 s3!4 4s 12 = 6s
(s2 +9)2
+ 48
s4
4
s 2
.

sin(3t)
Problem 3.2.2 t

sin(3t) R1 R1 3 3 1 v v=1
L t
= L (sin (3t)) = v 2 +9
dv = 3
tan 3 v=s
s s

1 1 s 1 s 1 s
= tan (1) tan 3
= 2
tan 3
= cot 3
:

d2 (t cos 5t)
Problem 3.2.3 h (t) = dt2

s2 +25 s(2s)
L (t cos 5t) = d (cot
ds
s5)
= d
ds
s
s2 +25
= (s2 +25)2
s(2s) (s +25)
2 2 25
= (s2 +25)2 = (ss2 +25) 2;

Also, if we write f (t) = t cos 5t, then f (0) = 0 and


f 0 (t) = cos 5t 5t sin 5t ) f 0 (0) = 1
2 (s2 25)
) L (h) = L ddt2f = s2 L (f ) sf (0) f 0 (0) = s2 (s2 +25)2 0 1
s4 25s2 (s4 +50s2 +625)
) L (h) = (s2 +25)2
= 75s2 625
(s2 +25)2

d(e3t cos 4t)


Problem 3.2.4 k (t) = dt

s s 3
L (e3t cos 4t) = s2 +16 js
= (s 3)2 +16
!s 3
d(e 3tcos 4t) s2 3s
L (k) = L dt
= sL (e3t cos 4t) (e3t cos 4t)jt=0 = s2 6s+25
1
s2 3s (s2 6s+25)
) L (k) = s2 6s+25
= 3s 25
s2 6s+25
.
CHAPTER 3. THE LAPLACE TRANSFORM 73

Rt
Problem 3.2.5 g (t) = z 10 e 4z
dz
0

L (t10 e 4t ) = s10!
11 j
10!
= (s+4) 11
s !s+4
R 10 4z
t
)L z e dz = s(s+4) 10!
11 :
0

Problem 3.2.6 q (t) = te3t cos (4t)


d s s2 16
L (t cos (4t)) = ds s2 +16
= (s2 +16)2
(s)2 16 (s 3)2 16
) L (e3t t cos (4t)) = 2 2 = 2 .
((s) +16) ((s 3)2 +16)
js !s 3

Problem 3.2.7 h (t) = (t 5)10 e 4(t 5)


U (t 5)

L (t 5)10 e 4(t 5)
U (t 5) = L (t10 e 4t
)e 5s
= 10!
(s+4)11
e 5s :

2(t 3)
Problem 3.2.8 g (t) = (t 3) e sin 4 (t 3) U (t 3)
d d 4 8s
L (t sin (4t)) = ds
L (sin (4t)) = ds s2 +16
= (s2 +16)2

) L (e 2t
t sin (4t)) = 8(s)
2 = 8(s+2)
2
((s)2 +16) js !s+2
((s+2)2 +16)
) L (g) = L e 2(t 3)
(t 3) sin 4 (t 3) U (t 3) = e 3s
L (e 2t
t sin (4t))
) L (g) = 8(s+2)
2 e 3s
.
((s+2)2 +16)

Problem 3.2.9 Draw a neat sketch for the following function:


8
< 0 1<t<0
1
gb (t) = 0 t<b ; where b is a positive constant.
: b
0 b t<1
gb (t) = 1b U (t) 1
b
U (t b) : So, by the Second Shift Theorem:
11 11 1 e bs
L (gb ) (s) = bs bs
e bs = bs
.
Note: By L’Hopital Rule,
1 e bs 0+se bs
lim L (gb ) (s) = lim = lim = 1:
b !0 b !0 bs b !0 s

Problem 3.2.10 k (t) = 7 (t) 2 (t 4) + 9 sin 4 (t 5) U (t 5)


4s 36
L (k) = 7 2e + s2 +16
e 5s
CHAPTER 3. THE LAPLACE TRANSFORM 74

Convolution
De…nition 1 The convolution of two functions f and g is the function f ? g
de…ned as follows:
Rt
(f ? g) (t) = 0
f (z) g (t z) dz

It follows that the binary operation ? is bilinear, commutative and


associative, and it has unit .

Basic Properties of the Binary Operation Convolution:


1. Commutativity: f ?g =g?f

2. Associativity: f ? (g h) = (f ? g) h

3. Linearity: f ? (g + h) = (f ? g) + (f h) ;
f ? (cg) = c (f g) = (cf ) ? g when c is a constant
Rt
4. 1 is not a neutral element: (f ? 1) (t) = 0
f (z) dz

5. is the neutral element: f ? = f:

6. Existence of Laplace transforms: If both f and g have Laplace trans-


forms, then f ? g has Laplace transform.

Convolution Theorem. For any two functions f (t) and g (t) which
have Laplace transforms:

L ff ? gg=L ff g.L fgg

Hence, for any two Laplace transforms F (s) and G (s) :

1 1 1
L (F:G)=L (F ) ? L (G)
CHAPTER 3. THE LAPLACE TRANSFORM 75

Proof. We have 0 1 0 1
Z1 Zt Z1 Zt
L ff ? gg = @ f (z) g (t z) dz A e st
dt = @ f (z) g (t z) e st
dz A dt
0 0 0 0
This is a double integral on the following region in the tz-plane:

0 t < 1; 0 z t:

We use the substitution t = z+y, and get the following region in the yz-plane:

0 y < 1; 0 z < 1:

Then by Substitution Theorem for Double Integrals, we have


0 @t @t 1
Z1 Z1 @y @z
L ff ? gg = @ f (z) g (y) e s(z+y) dz A dy
@z @z
0 0 @y @z
01 1 01 1
Z1 Z Z1 Z
@ f (z) g (y) 1 1
= e sz sy
dz A dy = @ f (z) g (y) 1 e sz
e sy
dz A dy
0 1
0
0 0 101 1 0 0
Z1 Z
= @ f (z) e sz
dz A @ g (y) e sy
dy A :
0 0

) L ff ? gg = L ff g :L fgg :

Problem 3.2.11 Find the Laplace transforms of the following functions:

1. k (t) = t e3t sin 6t

2. g (t) = t ? (e3t sin 6t)

3. h (t) = t ? e3t ? sin 6t

d 12(s 3)
k (f ) = ds
L (e3t sin 6t) = 2 (Problem 3.2.23):
((s 3)2 +36)

1 6 6
L (g) = L (t) :L (e3t sin 6t) = s2 s2 +36 js
= s2 ((s 3)2 +36)
:
!s 3

6
L (h) = L (t) :L (e3t ) :L (sin 6t) = s2 (s 3)(s2 +36)
.
CHAPTER 3. THE LAPLACE TRANSFORM 76

3.2.2 Classroom Exercises


Rt
Problem 3.2.12 g (t) = z 4 e3z dz
0

4! 4!
L (z 4 e3z ) = L (z 4 )js !s 3
= s 5 js
= (s 3)5
!s 3

Rt
) L (g) = L z 4 e3z dz = 1s L (z 4 e3z ) = 4!
s(s 3)5
.
0

Rt 2z
Problem 3.2.13 e cosh (5z) dz;
0

2z (s) s+2
L (e cosh (5z)) = L (cosh (5z))js !s+2
= (s)2 25
= (s+2)2 25
js !s+2

Rt
)L e 2z
cosh (5z) dz = 1s L (e 2z
cosh (5z)) = s+2
s((s+2)2 25)
.
0

Rt
Problem 3.2.14 g (t) = z sin 3z dz
0

d d 3
L (z sin 3z) = ds L (sin 3z) = ds s2 +9
= (s26s
+9)2
.
Rt
) L (g) = L z sin 3z dz = 1s L (z sin 3z) = (s2 +9)
6
2 :
0

d(e 2t cos 5t)


Problem 3.2.15 k (t) = dt

2t s s+2
L (e cos 5t) = s2 +25 js
= (s+2)2 +25
!s+2
d(e 2t cos 5t) 2t 2t s2 +2s
L (k) = L dt
= sL (e cos 5t) (e cos 5t)jt=0 = s2 +4s+29
1

) L (k) = s2 +2s s2 4s 29
s2 +4s+29
= 2s 29
s2 +4s+29
.

d
Problem 3.2.16 r (t) = dt
(t sin 6t)
d d 6 12s
L (t sin 6t) = ds
(L (sin 6t)) = ds s2 +36
= (s2 +36)2

) L (r) = sL (t sin 6t) (t sin 6t)jt=0 = 12s2


(s2 +36)2
:
CHAPTER 3. THE LAPLACE TRANSFORM 77

1 cos 4(t 6)
Problem 3.2.17 k (t) = t 6
U (t 6) :

1 cos 4t
R1 R1 1 v 1 v=1
L t
= L (1 cos 4t) = v v 2 +16
dv = ln (v) 2
ln (v 2 + 16) v=s
s s
p v=1
h iv=1
= ln (v) ln v 2 + 16 = ln p v = ln 1 ln p s :
v=s v 2 +16 v=s s2 +16
p
p s s2 +16
=0 ln s2 +16
= ln s
.
p
) L (k) = e 6s
ln s2 +16
s
:

Problem 3.2.18 q (t) = t 2


cos 3 t 2
:

q (t) = t 2 cos 3 t 2 = t 2 cos 3t cos 32 + sin 3t sin 32


= t 2 sin 3t = t sin 3t + 2 sin 3t

) L (q) = d
ds
L (sin 3t) + 2 L (sin 3t) = 6s
(s2 +9)2
+ 3
2(s2 +9)
.

Problem 3.2.19 k (t) = (t 7) e3(t 7)


cos 2 (t 7) U (t 7)

d d s 2s2 (s2 +4) s2 4


L (t cos (2t)) = ds
L (cos (2t)) = ds s2 +4
= (s2 +4)2
= (s2 +4)2
(s)2 4 (s 3)2 4
) L (e3t t cos (2t)) = 2 2 = 2
((s) +4) js !s 3 ((s 3)2 +4)

) L (k) = L e3(t 7)
(t 7) cos 2 (t 7) U (t 7) = e 7s
L (e3t t cos (2t))
(s 3)2 4
) L (k) = 2 2 e 7s
= (s2
s2 6s+5
6s+13)2
e 7s .
((s 3) +4)

Problem 3.2.20 Establish the following equality on convolution:


ekt e kt = k1 sinh kt:

First Solution: We use de…nition of convolution:


R t Rt 1 z=t
ekt e kt = 0 ekz e k(t z) dz = e kt 0 e2kz dz = 2k e kt e2kz z=0
1 1 1
= 2k
e kt e2kt 1 = 2k
ekt e kt
= k
sinh kt:
Second Solution: We use Convolution Theorem:
1
L ekt e kt
= L ekt :L e kt
= : 1
(s k) (s+k)
= 1
s2 k 2
:
) ekt e kt
= k1 L 1 k
s2 k 2
= 1
k
sinh kt:
CHAPTER 3. THE LAPLACE TRANSFORM 78

Problem 3.2.21 k (t) = sin2 t sin t sin t

L sin2 t = L 1
2
1
2
cos 2x = 1
2s
s
2(s2 +4)
= 2
s(s2 +4)
.
) L (k) = L sin2 t L (sin t) L (sin t) = 2
s(s2 +4)
1
(s2 +1)2
.

Problem 3.2.22 Draw a neat sketch for the following function, and …nd its
Laplace transform:
g (t) = cos t + sin t 2
U t 2

s+e s=2
Answer: L (g) = s2 +1
.

Problem 3.2.23 h (t) = e3t t sin 6t + e3t (t sin 6t) + 3 (t 2) :


d (sin 6t) d 6 12s
L (t sin 6t) = ds
= ds s2 +36
= (s2 +36)2
:

) L (e3t t sin 6t) = L (t sin 6t)js = 12s


(s2 +36)2
= 12(s 3)
2 :
!s 3
js !s 3 ((s 3)2 +36)

) L (h) = 12(s 3)
2 + 12s
(s 3)(s2 +36)2
+ 3e 2s
.
((s 3)2 +36)

Rt Rt
Problem 3.2.24 g (t) = ze7z sin 4z dz + ((ze7z ) sin 4z) dz
0 0

1 8s 56
As in Problem 3.2.23, L (te7t ) = (s 7)2
; L (te7t sin 4t) = 2
((s 7)2 +16)

L (g) = 1s [L (te7t sin 4t) + L (te7t ) :L (sin 4t)] = 1


s
8s 56
2 + 4
(s 7)2 (s2 +16)
:
((s 7)2 +16)

Problem 3.2.25 q (t) = sin 5t cos 2t + sin 5t cos 2t

L (sin 5t cos 2t) = 21 L (sin 7t + sin 3t) = 1


2
7
s2 +49
3
s2 +9
5
L (sin 5t cos 2t) = L (sin 5t) :L (cos 2t) = : s
s2 +25 s2 +4

) L (q) = 1
2
7
s2 +49
3
s2 +9
+ 5s
(s2 +25)(s2 +4)
.
CHAPTER 3. THE LAPLACE TRANSFORM 79

3.2.3 Revision Problems


Find the Laplace Transforms of the following functions:

Problem 3.2.26 h (t) = 4e3t sinh 5t

20 20 20
L (h) = (s)2 25
= (s 3)2 25
= s2 6s 16
:
js !s 3

Another Solution: L (h) = L (2e3t (e5t e 5t


)) = L (2e8t 2e 2t
):
) L (h) = s 2 8 s+2
2 20
= (s 8)(s+2) .

2(t 6)
Problem 3.2.27 k (t) = e cos 3 (t 6) U (t 6) :

2t s s+2
L (e cos 3t) = L (cos 3t)js !s+2
= s2 +9 js
= (s+2)2 +9
!s+2

)L e 2(t 6)
cos 3 (t 6) U (t 6) = e 6s
L (e 2t
cos 3t)

) L (k) = s+2
(s+2)2 +9
e 6s :

Problem 3.2.28 g (t) = e2(t 3)


sin 4 (t 3) U (t 3) :
4 4
L (e2t sin 4t) = s2 +16 js
= (s 2)2 +16
!s 2

) L (g) = e 3s
L (e2t sin 4t) = 4e 3s
(s 2)2 +16
:

Rt
Problem 3.2.29 g (t) = z 6 e3z dz + (t 2)6 e3(t 2)
U (t 2)
0

6! 6!
L (t6 e3t ) = L (t6 )js !s 3
= s 7 js
= (s 3)7
:
!s 3

) L (g) = 1s L (t6 e3t ) + e 2s


L (t6 e3t ) = 1
s
+e 2s 6!
(s 3)7
:
CHAPTER 3. THE LAPLACE TRANSFORM 80

Problem 3.2.30 p (t) = t 2


cos 3 t 2
U t 2
:
d d s s2 9
L (t cos 3t) = ds
L (cos 3t) = ds s2 +9
= (s2 +9)2
:

) L (p) = s2 9
(s2 +9)2
e s=2
.

7(t 4)
Problem 3.2.31 k (t) = (t 4) e sin 6 (t 4) U (t 4) :
d 6 12s
L (t sin 6t) = ds s2 +6
= (s2 +36)2
:

) L (t e 7t
sin 6t) = 12s
(s2 +36)2
= 12(s+7)
2 :
js !s+7 ((s+7)2 +36)

) L (k) = e 4s
L (t e 7t
sin 6t) = 12(s+7)
2 e 4s
:
((s+7)2 +36)

Problem 3.2.32 h (t) = e6(t 5)


sin 4 (t 5) U (t 5)

L (e6t sin 4t) = L (sin 4t)js !s 6

4 4 4
= s2 +16 js
= (s 6)2 +16
= s2 12s+52
!s 6

) L (h) = L e6(t 5)
sin 4 (t 5) U (t 5) = e 5s
L (e6t sin 4t) = 4e 5s
s2 12s+52
.

2(t 6)
Problem 3.2.33 k (t) = e 3et 6
U (t 6)

6s 2t 1 3 ( 2s 7)
L (k) = e L (e 3et ) = s+2 s 1
e 6s
= (s 1)(s+2)
e 6s

Problem 3.2.34 (t 2) e9(t 2)


U (t 2)
1 1
L (t e9t ) = s 2 js !s 9
= (s 9)2
:

) L (t 2) e9(t 2)
U (t 2) = e 2s
L (t e9t ) = e 2s
(s 9)2
:

1 e4t
Problem 3.2.35 t

Z1 Z1
1 e4t
L t
= L (1 4t
e )= 1
v
1
v 4
dv = [ln (v) ln (v 4)]v=1
v=s
s s
v v=1 s s 4
= ln v 4 v=s
= ln 1 ln s 4
= ln s
:
CHAPTER 3. THE LAPLACE TRANSFORM 81
Rt Rt
Problem 3.2.36 g (t) = 0
e6z sin 4z dz 0
(e6z sin 4z) dz
4 4
L (e6t sin 4t) = s2 +16 js
= (s 6)2 +16
:
!s 6

L (g) = 1s L (e6t sin 4t) 1


s
L (e6t ) L (sin 4t) = 4
s((s 6)2 +16)
4
s(s 6)(s2 +16)
:

Rt Rt Rt
Problem 3.2.37 g (t) = e3z cos (4z) dz+ z cos (4z) dz (z cos (4z)) dz:
0 0 0

We use our results in Problems (3.2.4) and (3.2.6):


L (g) = 1s L (e3z cos (4z)) + 1s L (t cos (4t)) 1
s
L (t) L (cos (4t))
s 3 s2 16 s
= s((s 3)2 +16)
+ s(s2 +16)2 s3 (s2 +16)
.

Problem 3.2.38 Establish the following equalities on convolution. Try more


than one method of solution, for each equality, as you did in Problem 3.2.20:

1. t t = 16 t3 :
1
2. t tn = (n+1)(n+2)
tn+2 :

3. ekt ekt = tekt :


1
4. ekt emt = k m
ekt emt ; when k 6= m:

Problem 3.2.39 cos (t ) U (t ) cos (t ) (Draw a sketch):

s(1+e s
)
Final Answer: s2 +1
:

Problem 3.2.40 h (t) = (t 4)2 U (t 6) :

h (t) = ((t 6) + 2)2 U (t 6)


= (t 6)2 + 4 (t 6) + 4 U (t 6)
= (t2 + 4t + 4)jt !t 6

) L (h) = e 6s
L (t2 + 4t + 4) = e 6s 2
s3
+ 4
s2
+ 4
s
= 2+4s+4s2
s3
e 6s .
CHAPTER 3. THE LAPLACE TRANSFORM 82

Problem 3.2.41 e7t cos 2t + t cos 2t + t cosh 2t + te7t cosh 2t:


s 7 s2 4 s2 +4 (s 7)2 +4
Final Answer: (s 7)2 +4
+ (s2 +4)2
+ (s2 4)2
+ 2
((s 7)2 4)

Problem 3.2.42 sin 2t cos 2t + t sin 2t cos 2t


2 4s
Final Answer: s2 +16
+ (s2 +16)2

Problem 3.2.43 (t sin 2t) cos 2t + t (sin 2t cos 2t) :


4s2 2
Final Answer: (s2 +4)3
+ s2 (s2 +16)

Problem 3.2.44 te2t sin 4t + t (e2t sin 4t) + e2t (t sin 4t) :
8(s 2) 4 8s
Final Answer: 2 + s2 ((s 2)2 +16)
+ (s 2)(s2 +16)2
((s 2)2 +16)

Problem 3.2.45 3e4t cosh 5t + 4e3t + 2 cos 3t e 4t


:
3(s 4) 4 2(s+4)
Final Answer: (s 4)2 25
+ s 3
+ (s+4)2 +9

2t
Problem 3.2.46 1 3e + 2e2t cos 3t + e2t sin 3t:
1 3 2s 1
Final Answer: s s+2
+ (s 2)2 +9

Problem 3.2.47 (t 3)6 e 4(t 3)


U (t 3) :
6!
Final Answer: (s+4)7
e 3s

d
Rt
Problem 3.2.48 dt
(t9 e 5t
)+ v9 e 5v
dv + (t9 e 5t
) cos 7t:
0

9!(s) 9! 9!(s)
Final Answer: (s+5)10
+ s(s+5)10
+ (s+5)10 (s2 +49)

Problem 3.2.49 t9 (e 5t
cos 7t) + t9 e 5t
cos 7t:
9!(s+5) 9!(s)
Final Answer: s10 ((s+5)2 +49)
+ s10 (s+5)(s2 +49)

Problem 3.2.50 U (t 5) + (t 8) (Draw a sketch).


e 5s
8s
Final Answer: s
+e
CHAPTER 3. THE LAPLACE TRANSFORM 83

Problem 3.2.51 Find the Laplace transforms of:

1. e2t e5t + e2t e5t

2. e2t e5t e4t

3. e2t e5t cos 3t

4. U (t 3) U (t 7) :

Problem 3.2.52 g (t) = sin 2 t 2


U t 6

2
g (t) = sin 2 t 6 3
U t 6
= sin 2 t 6 3
U t 6

= sin 2 t 6
cos 23 cos 2 t 6
sin 2
3
U t 6
p
1 3
= 2
sin 2 t 6 2
cos 2 t 6
U t 6
p
1 3
= 2
sin 2t + 2
cos 2t :
jt !t =6

p p
) L (g) = e s=6
L 1
2
sin 2t + 2
3
cos 2t = -e s=6 2+ 3s
2(s2 +4)
.

Problem 3.2.53 h (t) = cos t U t 6

h (t) = cos t+ 6 6
U t 6
= cos t + 6 jt
:
!t 6
p
3 1
But cos t + 6
= cos (t) cos 6
sin t sin 6
= 2
cos (t) 2
sin (t)
p
) L cos t + 6
= 1+ 3s
2(s2 +1)

) L (h) = L cos t + 6 jt
=e s=6
L cos t + 6
!t 6

p
) L (h) = 1+ 3s
2
2(s +1)
e s=6
.
CHAPTER 3. THE LAPLACE TRANSFORM 84

3.3 Inverse Laplace Transform


1
f (t) = L (F (s)) if f L (f (t)) = F (s) :
Shortly.
1
f =L (F ) if f L (f ) = F:

3.3.1 Lecture Examples


Find the inverse Laplace transforms of the following functions:
s+8
Problem 3.3.1 H (s) = s2 +6s+34
e 4s
By completing square in the denominator
s+8 (s+3)+5 s 5
L 1 s2 +6s+34 = L 1 (s+3) 2
+25
=L 1 s2 +25
+ s2 +25 js !s+3
3t 1 s 5 3t
=e L s2 +25
+ s2 +25
=e (cos 5t + sin 5t) .
)L 1
(H) = e 3t
cos 5tjt !t 4 =e 3(t 4)
(cos 5 (t 4) + sin 5 (t 4)) U (t 4) .

Problem 3.3.2 M (s) = 4 + 3e 2s


7e 6s
+ 5s e 9s

1
L (M ) = 4 (t) + 3 (t 2) 7 (t 6) + 5U (t 9) .

3s+41
Problem 3.3.3 H (s) = (s 3)(s+7)
e 9s

We use partial fractions: (s 3s+41


3)(s+7)
= A
s 3
+ B
s+7
) 3s + 41 = A (s + 7) + B (s 3)
at s = 3 : 50 = 10A ) A = 5;
at s = 7 : 20 = 10B )B= 2:
) 3s+41
(s 3)(s+7)
= 5
s 3
2
s+7
)L 1 3s+41
(s 3)(s+7)
= 5e3t 2e 7t

)L 1
(H) = (5e3t 2e 7t
)jt !t 9
= 5e3(t 9)
2e 7(t 9)
U (t 9)

16+s2
Problem 3.3.4 G (s) = ln s2

Let L (g (t)) = G (s) = ln (16 + s2 ) ln (s2 )


) L (tg (t)) = dG(s)
ds
= ds d
(ln (16 + s2 ) 2 ln (s)) = 2s
16+s2
+ 2
s
) tg (t) = L 1 2s
16+s2
+ 2s = 2 2 cos 4t:
)L 1
(G) = g = 2 2 cos 4t
t
. (Compare with Problem 3.2.17.)
CHAPTER 3. THE LAPLACE TRANSFORM 85

3.3.2 Classroom Exercises


s 3 s
Problem 3.3.5 F (s) = s2 6s+13
+ s2 +4
e 7s
s 3 s 3 s
s2 6s+13
= (s 3)2 +4
= s2 +4 js !s 3

)L 1 s 3
s2 6s+13
=L 1 s
s2 +4 js !s 3
= e3t L 1 s
s2 +4
= e3t cos 2t
Also, L 1 s2s+4 e 7s = L 1 s2s+4 j = cos 2tjt !t 7
t !t 7

)L 1
(F ) = e3t cos 2t + cos 2 (t 7) U (t 7) .

s 1
Problem 3.3.6 F (s) = s2 2s+10
e 4s :

1 s 1 1 s 1 1 s
L s2 2s+10
=L (s 1)2 +9
=L s2 +9 js !s 1
t 1 s t
=eL s2 +9
= e cot 3t
)L 1
(F ) = et 4
cos 3 (t 4) U (t 4) .

s 3
Problem 3.3.7 K (s) = s(s2 6s+25)

C(s 3)
By partial fractions K (s) = s(s2 s 6s+25)
3
= As + B
s2 6s+25
+ s2 6s+25
) s 3 = A (s2 6s + 25) + Bs + C (s 3) s
at s = 0 : 3 = 25A ) A = 3=25;
at s = 3 : 0 = 16A + 3B ) B = 16=25;
coe¢ cients of s2 : 0=A+C = ) C = 3=25:
) K (s) = 1
25
3 1s + 4 (s 4
3)2 +16
+ 3 (s s 3
3)2 +16
1
= 25
3 1s + 4 s2 +16
4 s
+ 3 s2 +16 js !t 3

)L 1
(K) = 1
25
( 3 + 4e3t sin 4t + 3e3t cos 4t) .
(Compare with Problem 3.2.37.)

3
Problem 3.3.8 F (s) = s2 4s+13
e 7s

1 3 1 3 1 3
L s2 4s+13
=L (s 2)2 +9
=L s2 +9 js !s 2

3
= e2t L 1
s2 +9
= e2t sin 3t

)L 1
(F ) = (e2t sin 3t)jt!t 7
= e2(t 7)
sin 3 (t 7) U (t 7) .
CHAPTER 3. THE LAPLACE TRANSFORM 86

s2 +4s+1
Problem 3.3.9 H (s) = (s+3)2 (s+2)
e 7s :

We use partial fractions:


s2 +4s+1 A B C
(s+3)2 (s+2)
= (s+3)2
+ s+3
+ s+2

) s2 + 4s + 1 = A (s + 2) + B (s + 3) (s + 2) + C (s + 3)2
at s = 3 : 2= A ) A = 2;
at s = 2 : 3 = C;
coe¢ cients of s2 : 1 = B + C = B 3; ) B = 4:
) s2 +4s+1
(s+3)2 (s+2)
= 2
(s+3)2
+ 4
s+3
3
s+2

)L 1 s2 +4s+1
(s+3)2 (s+2)
= 2te 3t
+ 4e 3t
3e 2t

)L 1
(H) = L 1 s2 +4s+1
(s+3)2 (s+2)
= (2te 3t
+ 4e 3t
3e 2t
)jt !t 7
jt !t 7

)L 1
(H) = 2 (t 7) e 3(t 7)
+ 4e 3(t 7)
3e 2(t 7)
U (t 7) .

1 s
Problem 3.3.10 F (s) = cot 7

1 s
Let L (f ) = F (s) = cot 7

) L (tf (t)) = dF (s)


ds
= d
ds
cot 1 s
7
( 1) 1 1 7 7
= s 2 7
= s2 49
= s2 +49
( )
7
+1 49
+1

) tf (t) = L 1 7
s2 +49
= sin 7t
)L 1
(F ) = f (t) = sin 7t
t
.
CHAPTER 3. THE LAPLACE TRANSFORM 87

3.3.3 Revision Problems


Find the inverse Laplace Transforms of the following functions:
3s 25
Problem 3.3.11 F (s) = s2 6s+25

By completing square in the denominator


F (s) = 3(s 3) 16
(s 3)2 +16
s
= 3 s2 +16 4
4 s2 +16 j s !t 3

)L 1
(F ) = (3 cos 4t 4 sin 4t) e3t . (Compare with Problem 3.2.4.)

2s 29
Problem 3.3.12 F (s) = s2 +4s+29

By completing square in the denominator,


2(s+2) 25 s+2
F (s) = (s+2)2
+25
= 2 (s+2)2
+25
5 (s+2)52 +25 = s
2 s2 +25 5
5 s2 +25 js !s+2

)L 1
(F ) = ( 2 cos 5t 5 sin 5t) e 2t
(Compare with Problem 3.2.15.)

4
Problem 3.3.13 Q (s) = (s 2)2 +16
e 3s

1 4 1 4
L (s 2)2 +16
=L s2 +16 js
= e2t sin 4t
!s 2

)L 1
(Q) = e2(t 3)
sin 4 (t 3) U (t 3) .

20
Problem 3.3.14 G (s) = (s 8)(s+2)
e 5s

20 A B
By partial fractions (s 8)(s+2)
= s 8
+ s+2
) 20 = A (s + 2) + B (s 8)
at s = 2 : 20 = 10B )B= 2;
at s = 8 : 20 = 10A ) A = 2:
) 20
(s 8)(s+2)
= 2s 18 1
2 s+2 :

)L 1 20
(s 8)(s+2)
= 2e8t 2e 2t
(Compare with Problem 3.2.26.)
)L 1
(G) = 2e8(t 5)
2e 2(t 5)
U (t 5) .
CHAPTER 3. THE LAPLACE TRANSFORM 88

4s2 24s+36
Problem 3.3.15 H (s) = (s 2)2 (s 4)

4s2 24s+36 A B C
(s 2)2 (s 4)
= s 2
+ (s 2)2
+ s 4
(partial fractions)
) 4s 2
24s + 36 = A (s 2) (s 4) + B (s 4) + C (s 2)2
at s = 2 : 4 = 2B ) B = 2;
at s = 4 : 4 = 4C ) C = 1;
coe¢ cients of s2 : 4 = A + C; ) A = 4 C = 3:
) H (s) = s 2 (s 2)2 + s 4
3 2 1
) L 1 (H) = (3 2t) e2t + e4t .

s+4
Problem 3.3.16 F (s) = s2 +8s+41
e 2s

1 s+4 s+4 s
L s2 +8s+41
= L 1 (s+4) 2
+25
= L 1 s2 +25 js !s+4
s
= e 4t L 1 s2 +25 = e 4t cot 5t
)L 1 s+4
(F ) = L 1 s2 +8s+41 jt !t 2
= e 4(t 2) cos 5 (t 2) U (t 2) .

2s 7
Problem 3.3.17 W (s) = (s 1)(s+2)
e 6s

2s 7 A B
We use partial fractions: (s 1)(s+2) = s 1
+ s+2
) 2s 7 = A (s + 2) + B (s 1)
at s = 1 : 9 = 3A ) A = 3;
at s = 2 : 3 = 3B ) B = 1;

) 2s 7
(s 1)(s+2)
= 3s 11 + 1
s+2
)L 1 2s 7
(s 1)(s+2)
= 3et + e 2t

)L 1
(W ) = L 1 2s 7
(s 1)(s+2)
= (e 2t
3et )jt !t 6
jt !t 6

)L 1
(W ) = e 2(t 6)
3et 6
U (t 6)

s+2
Problem 3.3.18 F (s) = (s+2)2 9
e 4s

1 s+2 1 s
L (s+2)2 9
=L s 2 9 js !s+2
2t 1 s 2t
=e L s2 9
=e cosh 3t
)L 1
(F ) = (e 2t
cosh 3t)jt!t 4
=e 2(t 4)
cosh 3 (t 4) U (t 4) .
CHAPTER 3. THE LAPLACE TRANSFORM 89

X
1
1 3ns
Problem 3.3.19 1 e 3s = e (in…nite geometric series)
n=0

3Ks
X1
K
1 e 3ns
1 e 3s = e (K is a positive integer, …nite geometric series),
n=0

X
1 X1
K
)L 1 1 e 3Ks
1 1
1 e 3s = (t 3n) , L 1 e 3s = (t 3n) .
n=0 n=0

Problem 3.3.20 Find the inverse Laplace Transforms of the following


functions:
2 1
1. A (s) = s(s2 +4)
: Answer: 2
(1 cos 2t).
1 1 5t 5t
2. B (s) = s(s+5)2
: Answer: 25
(1 e 5te ).
10
3. C (s) = (s 9)(s+1)
e 7s : Answer: e9(t 7)
e (t 7)
U (t 7).
s2 6s+2 t
4. D (s) = (s+1)(s 2)2
: Answer: e 2te2t .
4
5. E (s) = s2 4s+20
e 3s : Answer: sin 4 (t 3) e2(t 3)
U (t 3).
3s 12 s 7
6. F (s) = s2 8s 9
+ (s 7)2 +9
e 2s :
Answer: 3e4t cosh 5t + e7(t 2)
cos 3 (t 2) U (t 2).
2 s
7. H (s) = (s+4)3
e 7s + s2 +9
e s
:

Answer: (t 7)2 e 4(t 7)


U (t 7) + cos 3 (t ) U (t ):
s2 +7s+6
8. J (s) = (s 3)(s+3)2
e 4s +e 2s
:
s 5
9. K (s) = s2 10s+29
e 3s .
s+2 s 4
10. N (s) = s2 +4s+5
e 6s + s2 8s+80
e 3s :
2s
R1 4
11. P (s) = 3 + e + s v 2 +16
dv.
d 4
R1 4 3s
12. R (s) = ds s2 +16
+ s v 2 +16
dv e :
sin 4(t 3)
Answer: (t 3) sin 4 (t 3) + t 3
U (t 3)

13. T (s) = 7s e 9s
+ 5 + 4e 2s
:
CHAPTER 3. THE LAPLACE TRANSFORM 90

s2 +5s 6
14. V (s) = (s 2)2 (s+6)
:

6s2 +7s 2
15. W (s) = (s 2)(s+1)2
e 4s :
s 4
16. X (s) = s2 8s+25
e 7s :
13
17. Y (s) = (s+2)(s2 +9)
e 6s :
7!
18. Z (s) = (s 3)8
e 4s :
s+2
19. A (s) = s2 2s+2
:
6s2 +34s+76
20. C (s) = (s 3)(s2 +8s+25)
.
8+8s2
21. D (s) = (s+1)2 (s 3)
:
s 7
22. E (s) = s2 +s 2
e 5s +e 2s
:
4s2 24s+36
23. G (s) = (s 2)2 (s 4)
:
s 2
24. H (s) = s2 4s 5
e 6s +e 4s
:
5s+5
25. J (s) = (s 1)(s2 +4s+5)
:
6s s 4
26. K (s) = 1 2e + s2 8s+20
e 3s :
8!
27. N (s) = 1 + (s+4)9
e 2s :
1
28. P (s) = 2
tan (s).
s+7 1 e 7t
29. R (s) = ln s
: Answer: t
s
30. Q (s) = ln s+7
:
s s+7
31. Y (s) = ln s+7
+ ln s
:

s2 9
32. T (s) = ln s2
:

33. V (s) = ln (s2 + 9) 2 ln s:


s2 1 3s
34. W (s) = ln s2 +1
e
CHAPTER 3. THE LAPLACE TRANSFORM 91

3.3.4 Problems on the Inverse Form of the


Convolution Theorem
4s
Problem 3.3.21 F (s) = (s2 +4)2

Since, F (s) = 2 s22+4 s2s+4 (multiplication), then


1 1 2 1 s
L (F ) = 2L s2 +4
?L s2 +4
(convolution)
Zt
)L 1
(F ) = 2 sin 2t ? cos 2t = 2 sin (2z) cos 2 (t z) dz
0
Zt
1 z=t
= sin (2t) + sin (4z 2t) dz = z sin (2t) 4
cos (4z 2t) z=0
0
1
= t sin (2t) 0 4
cos (2t) + 41 cos ( 2t)

)L 1
(F ) = t sin (2t) , because cos ( 2t) = cos (2t) :

2s2
Problem 3.3.22 F (s) = (s2 +4)2

Since, F (s) = 2 s2s+4 s2s+4 (multiplication), then


1 1 s 1 s
L (F ) = 2L s2 +4
?L s2 +4
(convolution)
Zt
)L 1
(F ) = 2 cos 2t ? cos 2t = 2 cos (2z) cos 2 (t z) dz
0
Zt
2 z=t
= 2
cos (2t) + cos (4z 2t) dz = z cos (2t) + 41 sin (4z 2t) z=0
0
= t cos (2t) 0 + 14 sin (2t) 1
4
sin ( 2t)

)L 1
(F ) = t cos (2t) + 12 sin (2t) , because sin ( 2t) = sin (2t) :
CHAPTER 3. THE LAPLACE TRANSFORM 92

3.4 Fredholm Integral Equations


Use the Convolution Theorem to solve the following
Fredholm integral equations:
3.4.1 Lecture Examples
Rt
Problem 3.4.1 f (t) = e3t cos t + 3e3t sin t 3 0
f (z) e3(t z)
dz
f (t) = e3t cos t + 3e3t sin t 3 (f ? e3t ) :
We denote L (f ) by F (s), and apply the Laplace transform to both sides.
We get by Convolution Theorem:
F = (s s3)32 +1 + 3 (s 3)1 2 +1 3F: s 1 3
)F 1+ 3
s 3
= (s 3)s 2 +1 )F s
s 3
= s
(s 3)2 +1
)F = s 3
(s 3)2 +1
= s2s+1 j
s !s 3

) f (t) = L 1 3t
(F ) = e cos t .

3.4.2 Classroom Exercises


Rt
Problem 3.4.2 f (t) = 2 + 3 sinh t 0
f (z) sinh (t z) dz
f (t) = 2 + 3 sinh t f ? sinh t: We denote L (f ) by F (s), and apply
the Laplace transform to both sides. We get by Convolution Theorem:
) F = 2s + s23 1 L (f ) :L (sinh t) = 2s + s23 1 F: s21 1
) F 1 + s21 1 = 2s + s23 1 = 2ss(s22+3s
2
1)

)F s2
s2 1
= 2s2 +3s 2
s(s2 1)
)F = 2s2 +3s 2
s3
= 2 1s + 3 s12 2
s3

) f (t) = L 1
(F ) = 2 + 3t t2 :

Rt
Problem 3.4.3 f (t) = e3t e2t + 0
f (z) e2(t z)
dz
f (t) = e3t e2t + f ? e2t :
Denote L (f ) by F (s).
) F = s 1 3 s 1 2 + F: s 1 2
) F 1 s 12 = s 13 s 12 = (s 2) (s 3)
(s 3)(s 2)
) F ss 32 = (s 3)(s
1
2)

)F = s 2
s 3
1
(s 3)(s 2)
= 1
(s 3)2
= 1
s 2 js !s 3

) f (t) = L 1
(F ) = te . 3t
CHAPTER 3. THE LAPLACE TRANSFORM 93
Rt
Problem 3.4.4 f (t) = 8te4t 4 0
f (z) e4(t z)
dz

f (t) = 8te4t 4 (f ? e4t ) :


Denote L (f ) by F (s).
) F = (s 84)2 4F: s 1 4
)F 1+ 4
s 4
= 8
(s 4)2
)F s
s 4
= 8
(s 4)2
)F = 8
s(s 4)
= As + sB4
(partial fractions)
) 8 = A (s 4) + Bs
at s = 0 : 8 = 4A ) A = 2;
at s = 4 : 8 = 4B ) B = 2:
) F = 2s + s 2 4
) f (t) = L 1
(F ) = 2 + 2e4t .

3.4.3 Revision Problems


Rt
Problem 3.4.5 f (t) = e3t 0
f (z) e4(t z)
dz

f (t) = e3t (f ? e4t ) :


We denote L (f ) by F (s), and apply the Laplace transform to both sides.
We get by Convolution Theorem:
F = s 1 3 F: s 1 4
) F 1 + s 14 = s 13
) F ss 43 = s 1 3
) F = (ss 3)4 2 = (s(s 3)3)21 = s 1 3 (s 13)2 = 1s s12 js !s 3
) f (t) = L 1
(F ) = (1 t) e3t .

Rt
Problem 3.4.6 f (t) = 3t2 + et 0
f (z) et z
dz:

f (t) = 3t2 + et (f ? et ) :
Denote L (f ) by F (s), and apply the Laplace transform to both sides of
the equation. Then we get by the Convolution Theorem:
F = s63 + s 1 1 F: s 1 1
) F 1 + s 1 1 = s63 + s 1 1 i.e., F s s 1 = s63 + s 1 1
) F = s s 1 s63 + s 1 1 = 6(ss4 1) + s(ss 11) = s63 s64 + 1s
) f (t) = L 1
(F ) = 3t2 t3 + 1 .
CHAPTER 3. THE LAPLACE TRANSFORM 94

Rt
Problem 3.4.7 f (t) = et cos t + 3 0
f (z) e 2(t z)
dz: .

f (t) = et cos t + 3f ? e 2t : Denote L (f ) by F (s).


) F = L (et cos t) + 3L (f e 2t ) = s2s+1 j + 3L (f ) :L (e 2t
)
s !s 1

)F = s 1
(s 1)2 +1
1
+ 3F: s+2 : )F 1 3
s+2
= s 1
(s 1)2 +1
:

)F s 1
s+2
= s 1
(s 1)2 +1
: F = s+2
(s 1)2 +1

)F = s 1
(s 1)2 +1
+ 3
(s 1)2 +1
= s
s2 +1
+ 3
s2 +1 js
:
!s 1

) f (t) = L 1
(F ) = et (cos t + 3 sin t) .

Rt
Problem 3.4.8 3f (t) = e7t e4t 9 0
f (z) e7(t z)
dz:

3f (t) = e7t e4t 9 (f ? e7t ) :


Denote L (f ) by F (s).
) 3F = s 1 7 s 1 4 9F: s 1 7
) F 3 + s 9 7 = s 1 7 s 1 4 = s(s 47)(s
(s 7)
4)

)F 3(s 4)
s 7
= 3
(s 7)(s 4)

)F = s 7
s 4
1
(s 7)(s 4)
= 1
(s 4)2
)F = 1
s2 js !s 4

) f (t) = L 1 4t
(F ) = te .

Rt
Problem 3.4.9 f (t) = 3 sin 5t 8 0
f (z) cos 5 (t z) dz .

f (t) = 3 sin 5t 8f ? cos 5t:


Denote L (f ) by F (s).
) F = 3L (sin 5t) 8L (f cos 5t) = 3L (sin 5t) 8L (f ) :L (cos 5t)
)F = 15
s2 +25
s
8F: s2 +25 : )F 1+ 8s
s2 +25
= 15
s2 +25

)F s2 +8s+25
s2 +25
= 15
s2 +25

)F = 15
s2 +8s+25
= 15
(s+4)2 +9
=5 3
s2 +9 js !s+4

) f (t) = L 1
(F ) = 5e 4t
sin 3t .
CHAPTER 3. THE LAPLACE TRANSFORM 95
Rt
Problem 3.4.10 f (t) = e2t + 0
f (z) et z
dz:

Rt
Problem 3.4.11 f (t) = cos t + 10 0
f (z) sin (t z) dz:

Rt
Problem 3.4.12 f (t) = e5t sinh 3t 6 0
f (z) e8(t z)
dz:

Rt
Problem 3.4.13 f (t) = 4 sin 5t 6 f (z) cos 5 (t z) dz:
0

Rt
Problem 3.4.14 f (t) = e3t e2t + 0
f (z) e2(t z)
dz:

Rt
Problem 3.4.15 f (t) = 8e3t 2 0
f (z) cos (t z) dz:

Rt
Problem 3.4.16 f (t) = et 9 0
f (z) e7(t z)
dz:

Rt
Problem 3.4.17 f (t) = e2t + sin 2t 4 0
f (z) e2(t z)
dz:

Rt
Problem 3.4.18 f (t) = 4tet + 2 0
f (z) et z
dz:

Rt
Problem 3.4.19 f (t) = sinh 2t 4 0
f (z) e2(t z)
dz:

Rt
Problem 3.4.20 f (t) = sin 3t 6 0
f (z) cos 3 (t z) dz:

Rt
Problem 3.4.21 f (t) = cos 2t + sin 2t 4 0
f (z) e2(t z)
dz:
CHAPTER 3. THE LAPLACE TRANSFORM 96

3.5 Laplace Transform and Initial-value


Problems:
Use the Laplace transform to solve the following initial-
value problems:

3.5.1 Lecture Examples


d2 y
Problem 3.5.1 dt2
8 dy
dt
+ 16y = 6te4t ; y (0) = 1; dy
dx
(0) = 4

Let L (y) = Y (s) : ) L dxdy


= sY y (0) = sY 1:
) L (y ) = s Y sy (0) y (0) = s2 Y s 4:
00 2 0

Apply the Laplace transform to both sides:


) s2 Y s 4 8 (sY 1) + 16Y = (s 64)2 :
) (s2 8s + 16) Y = (s 64)2 + s + 4 8
) (s 4)2 Y = 6
(s 4)2
+ (s 4)
)Y = 6
(s 4)2 (s 4)2
+ (s 4)
(s 4)2
= 3!
(s 4)4
+ 1
s 4
:
) y (t) = L 1
(Y ) = L 1 3!
s 4 js !s 4
+ 1
s 4
= t3 e4t + e4t = (t3 + 1) e4t :

Problem 3.5.2 y 00 + 4y 0 + 3y = 6e 3t
; y (0) = 1; y 0 (0) = 2

Let L (y) = Y (s) : ) L (y 0 ) = sY y (0) = sY 1;


L (y ) = s Y sy (0) y 0 (0) = s2 Y s + 2:
00 2

We apply the Laplace transform to both sides:


s2 Y s + 2 + 4sY 4 + 3Y = s+3 6
: ) (s2 + 4s + 3) Y = 6
s+3
+s+2
) (s + 3) (s + 1) Y = 6+(s+3)(s+2)
2
s+3
= s +5s+12
s+3
) Y = (s+3)2 (s+1) = s+3 + (s+3)2 + s+1
s2 +5s+12 A B C

) s2 + 5s + 12 = A (s + 3) (s + 1) + B (s + 1) + C (s + 3)2
at s = 3 : 6 = 2B ) B = 3;
at s = 1 : 8 = 4C ) C = 2;
coe¢ cients of s2 : 1 = A + C; ) A = 1 C = 1:
) Y (s) = s+3 (s+3)2 + s+1
1 3 2

) y (t) = L 1
(Y ) = e 3t
3te 3t
+ 2e t
= (1 + 3t) e 3t
+ 2e t
.
CHAPTER 3. THE LAPLACE TRANSFORM 97

3.5.2 Classroom Exercises


d2 y
Problem 3.5.3 dt2
2 dy
dt
+ y = 6e2t cos t; y (0) = 0; y 0 (0) = 3

Let L (y) = Y (s) : )L dy


dt
= sY y (0) = sY;
d2 y 2 0 2
L = s Y sy (0) y (0) = s Y 3:
dt2
We apply the Laplace transform to both sides:
s 6(s 2)
s2 Y 3 2sY + Y = 6 s2 +1 js
= (s 2)2 +1
:
!s 2
) (s2 2s + 1) Y = 6s 12
s2 4s+5
+3
1)2
) (s 1)2 Y = 6s 12+3s2 12s+15
s2 4s+5
= 3s2 6s+3
s2 4s+5
= s3(s
2 4s+5

) Y (s) = s2
3
4s+5
= (s 2)3 2 +1 = 3 1
s2 +1 js !s 2

) y (t) = L 1
(Y ) = 3e2t sin t .

Problem 3.5.4 y 00 6y 0 + 9y = 6t2 e3t ; y (0) = 1; y 0 (0) = 3

Let L (y) = Y (s) : ) L (y 0 ) = sY y (0) = sY 1;


L (y 00 ) = s2 Y sy (0) y 0 (0) = s2 Y s 3:
We apply the Laplace transform to both sides:
s2 Y s 3 6sY + 6 +9Y = (s 123)3 : ) (s2 6s + 9) Y = s 3+ (s 123)3
(s 3)2 Y = s 3+ 12
(s 3)3
) Y = s 1 3 + (s 123)5 = 1
s 3
+ 12 4!
4! s5 js !s 3

) y (t) = L 1
(Y ) = e3t + 12 t4 e3t = 1 + 21 t4 e3t .

Problem 3.5.5 y 00 6y 0 + 13y = 13; y (0) = 2; y 0 (0) = 3

Let L (y) = Y (s) : ) L (y 0 ) = sY y (0) = sY 2;


L (y 00 ) = s2 Y sy (0) y 0 (0) = s2 Y 2s 3:
Apply the Laplace transform to both sides:
) s2 Y 2s 3 6 (sY 2) + 13Y = 13 s
:
) (s 2 13
6s + 13) Y = s + 2s 9 = 2s2 9s+13
s
) Y = s(s2 6s+13) = s + s2 6s+13 + s2 6s+13
2s2 9s+13 A B C(2s 6)

) 2s2 9s + 13 = A (s2 6s + 13) + Bs + 2Cs (s 3)


at s = 0 : 13 = 13A ) A = 1;
at s = 3 : 4 = 4A + 3B ) B = 0;
coe¢ cients of s : 2
2 = A + 2C; ) C = 1 12 A = 12 :
Y (s) = 1
s
+ s 3
s2 6s+13
= 1
s
+ s 3
(s 3)2 +4
) y (t) = L 1
(Y ) = 1 + e3t cos 2t .
CHAPTER 3. THE LAPLACE TRANSFORM 98

Problem 3.5.6 y 00 4y = 16e2t ; y (0) = 3; y 0 (0) = 6

Let L (y) = Y (s) : ) L (y 0 ) = sY y (0) = sY 3;


L (y 00 ) = s2 Y sy (0) y 0 (0) = s2 Y 3s 6:
We apply the Laplace transform to both sides:
) s2 Y 3s 6 4Y = s162 :
) (s2 4) Y = s162 + 3s + 6 = 3ss +4
2
2
) Y = (s 2)(s2 4) = (s 2)2 (s+2) = sA2 + (s B2)2 + s+2
3s2 +4 3s2 +4 C

) 3s2 + 4 = A (s 2) (s + 2) + B (s + 2) + C (s 2)2
at s = 2 : 16 = 4B ) B = 4;
at s = 2 : 16 = 16C ) C = 1;
coe¢ cients of s2 : 3 = A + C; ) A = 2:
) Y (s) = s 2 + (s 2)2 + s+2
2 4 1

) y (t) = L 1
(Y ) = 2e2t + 4te2t + e 2t
.

3.5.3 Revision Problems


Use the Laplace Transform to solve the following initial-value problems:

Problem 3.5.7 y 00 6y 0 + 8y = 4e2t ; y (0) = 4; y 0 (0) = 8 :

Let L (y) = Y (s) : ) L (y 0 ) = sY y (0) = sY 4;


L (y 00 ) = s2 Y sy (0) y 0 (0) = s2 Y 4s 8:
We apply the Laplace transform to both sides:
s2 Y 4s 8 6sY +24+8Y = 4
s 2
: ) (s2 6s + 8) Y = 4
s 2
+4s 16
) (s 2) (s 4) Y = 4+(s 2)(4s 16)
s 2
= 4s2 24s+36
s 2

) Y (s) = 4s2 24s+36


(s 2)2 (s 4)
= A
s 2
+ B
(s 2)2
+ C
s 4
(partial fractions)

) 4s2 24s + 36 = A (s 2) (s 4) + B (s 4) + C (s 2)2


at s = 2 : 4 = 2B ) B = 2;
at s = 4 : 4 = 4C ) C = 1;
coe¢ cients of s2 : 4 = A + C; ) A = 4 C = 3:
) Y (s) = s 2 (s 2)2 + s 4
3 2 1

and so ) y (t) = L 1
(Y ) = (3 2t) e2t + e4t .
CHAPTER 3. THE LAPLACE TRANSFORM 99

d2 y
Problem 3.5.8 dt2
+ 2 dy
dt
8y = 36te2t ; y (0) = 0; y 0 (0) = 1:

Let L (y) = Y ) L (y 0 ) = sY y (0) = sY


) L (y") = s2 Y sy (0) y 0 (0) = s2 Y 1
Apply Laplace Transform
) s2 Y 1 + 2sY 8Y = 36
(s 2)2

) (s2 + 2s 8) Y = 36
(s 2)2
+1
36+(s2 4s+4)
) (s + 4) (s 2) Y = (s 2) 2 = s2 4s 32
(s 2)2

)Y = (s 8)(s+4)
(s 2)3 (s+4)
= s 8
(s 2)3
= s 2
(s 2)3
6
(s 2)3
= 1
(s 2)2
6
(s 2)3

)Y = 1
s2
3 s23 js !s 2

)y=L 1
(Y ) = (t 3t2 ) e2t :

Problem 3.5.9 y 00 + 5y 0 + 6y = 2e 3t
; y (0) = 1; y 0 (0) = 4:

Let L (y) = Y (s) : ) L (y 0 ) = sY y (0) = sY 1;


L (y 00 ) = s2 Y sy (0) y 0 (0) = s2 Y s + 4:
We apply the Laplace transform to both sides:
2
s2 Y s + 4 + 5sY 5 + 6Y = s+3 :
) (s + 5s + 6) Y = s + 1 s+3
2 2

) (s + 3) (s + 2) Y = (s+3)(s+1) 2
s+3
= s2 +4s+1
s+3

)Y = s2 +4s+1
(s+3)2 (s+2)
= A
(s+3)2
+ B
s+3
+ C
s+2
(partial fractions)
) s + 4s + 1 = A (s + 2) + B (s + 3) (s + 2) + C (s + 3)2
2

at s = 3 : 2= A ) A = 2;
at s = 2 : 3 = C;
coe¢ cients of s2 : 1 = B + C = B 3; ) B = 4:
)Y = s2 +4s+1
(s+3)2 (s+2)
= 2
(s+3)2
+ 4
s+3
3
s+2

) y (t) = L 1
(Y ) = 2te 3t
+ 4e 3t
3e 2t
.
(See also Problem 2.5.17.)
CHAPTER 3. THE LAPLACE TRANSFORM 100

Problem 3.5.10 y 00 + ! 2 y = 0, y (0) = 1 , y 0 (0) = 0:

Final Answer y = cos !t

Problem 3.5.11 y 00 ! 2 y = 0, y (0) = 0 , y 0 (0) = !:

Final Answer y = sinh !t

Problem 3.5.12 y 00 ! 2 y = 0, y (0) = 1 , y 0 (0) = 0:

Final Answer y = cosh !t:

Problem 3.5.13 y 00 + y 0 6y = 30 , y (0) = 0 , y 0 (0) = 30:

Final Answer y = 9e2t 4e 3t


5:

Problem 3.5.14 y 00 2y 0 + y = 12tet , y (0) = 2 , y 0 (0) = 7:

Final Answer y = 2et + 5tet + 2t3 et :

Problem 3.5.15 y 00 8y 0 + 16y = 2e3t , y (0) = 2 , y 0 (0) = 8:

Final Answer y = 2e3t + 2te4t :

Problem 3.5.16 y 00 8y 0 + 16y = 2e4t , y (0) = 2 , y 0 (0) = 8:

Final Answer y = 2e4t + t2 e4t :

Problem 3.5.17 y 00 2y 0 + y = 2et cos t; y (0) = 1; y 0 (0) = 1 .

Final Answer y= 2et cos t + 3et :

Problem 3.5.18 y 00 y0 6y = 15e 2t


; y (0) = 0; y 0 (0) = 3:

Problem 3.5.19 y 00 + 2y 0 + y = 80e3t ; y (0) = 8; y 0 (0) = 8:


CHAPTER 3. THE LAPLACE TRANSFORM 101

Problem 3.5.20 y 00 + 5y 0 + 6y = 2e 3t
; y (0) = 1; y 0 (0) = 4:

Problem 3.5.21 y 00 y 0 6y = 13 cos 3t 13 sin 3t; y (0) = 2; y 0 (0) = 4.

Problem 3.5.22 y 00 y0 2y = 3e t ; y (0) = 6; y 0 (0) = 7:

Problem 3.5.23 y 00 + 4y 0 12y = 8e2t ; y (0) = 1; y 0 (0) = 3:

Problem 3.5.24 y 00 4y 0 + 4y = te3t ; y (0) = 0; y 0 (0) = 1:

Problem 3.5.25 y 00 6y 0 + 5y = 8et ; y (0) = 3; y 0 (0) = 1:

Problem 3.5.26 y 00 + y 0 2y = 9e 2t
; y (0) = 0; y 0 (0) = 3.

Problem 3.5.27 y 00 2y 0 + 10y = 10e2t ; y (0) = 0; y 0 (0) = 7 .

Problem 3.5.28 y 00 9y = 6e 3t
; y (0) = 1; y 0 (0) = 4 .

Problem 3.5.29 y 00 + y 0 6y = 6e t ; y (0) = 1; y 0 (0) = 1:


CHAPTER 3. THE LAPLACE TRANSFORM 102

3.6 Laplace Transform and Systems of


Di¤erential Equations
Use the Laplace transform to solve the following initial-value problems (sys-
tems of di¤erential equations):

3.6.1 Lecture Examples


dy dx
Problem 3.6.1 6x + dt
= 6e t ; dt
+x y = 0; x (0) = 0; y (0) = 3 :

Let L (x) = X (s) : ) L dx


dt
= sX x (0) = sX;
let L (y) = Y (s) : ) L dt = sY y (0) = sY 3:
dy

6
) 6X + sY 3= : (3.1)
s+1
Also, (s + 1) X Y = 0; and when we multiply by s,

s2 + s X sY = 0; (3.2)

We add the two equations (3.1) and (3.2), and get:


6
(s2 + s 6) X = 3 + s+1 = 3s+3+6
s+1
= 3s+9
s+1
(s + 3) (s 2) X = 3(s+3)
s+1
) X = (s 2)(s+1)
3
= sA2 + s+1
B
(partial fractions)
) 3 = A (s + 1) + B (s 2)
at s = 1 : 3 = 3B ) B = 1;
at s = 2 : 3 = 3A ) A = 1;
) X = s 1 2 s+11
) x (t) = L 1 (X) = e2t e t
.
dx dx
As dt
+x y = 0; then y = dt
+ x = 2e2t + e t
+ e2t e t
= 3e2t .

3.6.2 Classroom Exercises


dx dy
Problem 3.6.2 dt
6x 3y = 0; 3x + dt
= 2te3t ;
x (0) = 1; y (0) = 1:

dy
L dx
dt
= sX x (0) = sX 1; L dt
= sY y (0) = sY + 1:
Apply the Laplace transform:
CHAPTER 3. THE LAPLACE TRANSFORM 103

) sX 1 6X 3Y = 0; 3X + sY + 1 = 2
(s 3)2
: This gives

s2 X 6sX 3sY = s; (3.3)


6
9X + 3sY = 3: (3.4)
(s 3)2
We add the two equations (3.3) and (3.4), and get:
(s2 6s + 9) X = s 3 + (s 63)2 ) (s 3)2 X = s 3+ 6
(s 3)2
)X= 1
s 3
+ 6
(s 3)4
) x (t) = L 1
(X) = (1 + t3 ) e3t .
dx 1 dx
As dt
6x 3y = 0; then y = 3 dt
2x
) y = 13 (3 + 3t2 + 3t3 ) e3t 2 (1 + t ) e : 3 3t
) y (t) = ( 1 + t2 t3 ) e3t .

dx dy
Problem 3.6.3 dt
+y = 2t; x 2y + dt
=t;
x (0) = 0; y (0) = 1 :

Let L (x) = X (s) : ) L dxdt


= sX x (0) = sX:
Let L (y) = Y (s) : ) L dt = sY
dy
y (0) = sY 1:
Apply the Laplace transform to all sides:
2
) sX + Y = ; (3.5)
s2
Also, X 2Y + sY 1 = s12 ;
s2 +1
that is, X + (s 2) Y = s12 + 1 = s2
We multiply by s :
s3 + s
) sX + s 2s Y = 2
: (3.6)
s2
We add equations (3.5) and (3.6), and get:
3
(s2 2s + 1) Y = s +s s2
2
:
) Y = s2 (s 1)2 = s + s2 + sC 1 + (s D1)2 (partial fractions)
s3 +s 2 A B

) s3 + s 2 = As (s 1)2 + B (s 1)2 + Cs2 (s 1) + Ds2 :


at s = 0 : 2 = B;
at s = 1 : 0 = D;
coe¢ cients of s : 1 = A 2B; ) A = 1 + 2B = 3;
3
coe¢ cients of s : 1 = A + C; ) C = 1 A = 4:
) Y (s) = 3 1s 2 s12 + 4 s 1 1
) y (t) = 3 2t + 4et :
) x = t 2y + dy dt
= t 2( 3 2t + 4et ) + ( 2 + 4et ) :
) x (t) = 4 + 3t 4et :
CHAPTER 3. THE LAPLACE TRANSFORM 104

3.6.3 Revision Problems


Use the Laplace transform to solve the following initial-value problems (sys-
tems of di¤erential equations):
dy dx
Problem 3.6.4 x dt
= 1; dt
2x + y = t 2,
x (0) = 2; y (0) = 0 .

Let L (x) = X (s) : )L dx


dt
= sX x (0) = sX 2:
Let L (y) = Y (s) : )L dy
dt
= sY y (0) = sY:
1
)X sY = ; (3.7)
s
1 2
Also, sX 2 2X + Y = s2 s
; and so,
1 2s + 2s2
) s2 X 2sX + sY = : (3.8)
s
2s+2s2 2s(s 1)
We add (3.7) and (3.8): (s2 2s + 1) X = s
= s
) (s 1)2 X = 2 (s 1) ) X = s 2 1 ) x (t) = L (X) = 2e . 1 t

As dx
dt
2x + y = t 2; then y = t 2 dx dt
+ 2x
) y = t 2 2et + 4et ) y (t) = t 2 + 2et .

Problem 3.6.5 4x 2 dy
dt
= 6e3t 8te3t ; dx
dt
3x + 2y = 8 ;
x (0) = 0; y (0) = 3 :
dy
L dx
dt
= sX x (0) = sX and L dt
= sY y (0) = sY 3:
We apply the Laplace transform: ) 4X 2 (sY 3) = s 6 3 8
(s 3)2
;

6s 26 42s 6s2 80
) 4X 2sY = 6+ = : (3.9)
(s 3)2 (s 3)2
Also, (s 3) X + 2Y = 8s . We multiply by s, and get
s2 3s X + 2sY = 8: (3.10)
We add the two equations (3.9) and (3.10), and get:
6s 26 2(s 3)2 +6s 26 2 2(s2 3s 4)
(s2 3s 4) X = 2 + (s 3)2
= (s 3)2
= 2s(s 6s
3)2
8
= (s 3)2
)X= 2
(s 3)2
) x (t) = L 1
(X) = 2te . 3t

dx dx
As dt
3x + 2y = 8; then 2y = 8 dt
+ 3x:
) 2y = 8 2e 3t
6te + 6te3t 3t
) y (t) = 4 e3t .
CHAPTER 3. THE LAPLACE TRANSFORM 105

Problem 3.6.6
dx
+ 3x 2y = 3 sin 2t 6e t ;
dt
dy
2x + = 3e t ;
dt

x (0) = 0 , y (0) = 4:

Let L (x) = X (s) : )L dx


dt
= sX x (0) = sX:
Let L (y) = Y (s) : )L dy
dt
= sY y (0) = sY 4:
We apply the Laplace Transform to all sides:
6 6
sX + 3X 2Y = ; (3.11)
s2 +4 s+1
3
2X + sY : 4= (3.12)
s+1
These 2 equations become, after multiplying equation (3.11) by s and
equation (3.12) by 2 :
6s 6s
s2 X + 3sX 2sY = : (3.13)
s2 + 4 s+1
6
) 4X + 2sY = 8: (3.14)
s+1
We add equations (3.13) and (3.14), and get:
6s 6s 6
(s2 + 3sX + 4) X = s2 +4 s+1
+8 s+1
6s 6s
= s2 +4
6+8= s2 +4
+ 2;
2s2 +6s+8
that is, (s2 + 3sX + 4) X = s2 +4

)X= 2
s2 +4
) x (t) = L 1
(X) = sin 2t .
dx
As dt
+ 3x 2y = 3 sin 2t 6e t ; then
dx t
2y = dt
+ 3x 3 sin 2t + 6e = 2 cos 2t + 3 sin 2t 3 sin 2t + 6e t :
) y (t) = cos 2t + 3e t
.
CHAPTER 3. THE LAPLACE TRANSFORM 106

dy dx
Problem 3.6.7 x + dt
= 2et ; dt
y=0;
x (0) = 0; y (0) = 1 :

Let L (x) = X (s) : ) L dx


dt
= sX x (0) = sX:
Let L (y) = Y (s) : ) L dt = sY y (0) = sY 1:
dy

We apply the Laplace transform to all sides: ) X + sY 1 = 2


s 1
;

2 s 3
) X + sY = 1 = : (3.15)
s 1 s 1
Also, sX Y = 0. We multiply by s, and get

s2 X sY = 0: (3.16)

We add the two equations (3.15) and (3.16), and get:


(s2 + 1) X = ss 13 ) X = (s 1)(s
s 3 A B Cs
2 +1) = s 1 + s2 +1 + s2 +1 (partial
fractions)
) s 3 = A (s2 + 1) + B (s 1) + Cs (s 1)
at s = 1 : 2 = 2A ) A = 1;
at s = 0 : 3=A B ) B = A + 3 = 2;
2
coe¢ cients of s : 0 = A + C; ) C = A = 1:
X = s 1 + 2 s2 +1 + s2 +1 ) x (t) = L 1 (X) = et + 2 sin t + cos t .
1 1 s

As dx
dt
y = 0; then y = dx
dt
) y (t) = et + 2 cos t sin t .

Problem 3.6.8 x (0) = 1 , y (0) = 0 ,

dx dy
+ y = 0; x+ = 2 cos t:
dt dt
(Answer: x = cos t, y = sin t.)

Problem 3.6.9 x (0) = 0 , y (0) = 1 ,

dx dx dy
2y = 6e3t sin t; 10x 3 + 2 = 0:
dt dt dt
(Answer: x = 2e3t sin t, y = e3t cos t.)
CHAPTER 3. THE LAPLACE TRANSFORM 107

Problem 3.6.10 x (0) = 1 , y (0) = 0 ,


dx
3x + 2y = 0;
dt
dy
x+y = 0:
dt
(Answer: x = e2t sin t + e2t cos t, y = e2t sin t.)

Problem 3.6.11 x (0) = 0 , y (0) = 1 ,


dx
2x 3y = 0;
dt
dy
x+ = 3te2t + 2e2t :
dt
(Answer: x = 3te2t , y = e2t .)

Problem 3.6.12 x (0) = 1 , y (0) = 0 ,


dx dy
= 2e2t ;
dt dt
dx
3x + y = 0:
dt
(Answer: x = (1 + t) e2t , y = te2t .)

Problem 3.6.13 x (0) = 0 , y (0) = 1 ,


dx
+ 3x y = 0;
dt
dx dy 3t
3 + = 9te :
dt dt
(Answer: x = te 3t , y = e 3t .)

Problem 3.6.14 x (0) = 0 , y (0) = 1 ,


dx
3x y = t2 e3t ;
dt
dy
2x + 3y = 0:
dt
(Answer: x = te3t , y = e3t t2 e3t .)
CHAPTER 3. THE LAPLACE TRANSFORM 108

Problem 3.6.15 x (0) = 0 , y (0) = 3 ,


dx
3 y = 12e4t sin t;
dt
dy
3x + = 12e4t cos t:
dt
(Answer: x = e4t sin t, y = 3e4t cos t.)

Problem 3.6.16 x (0) = 0 , y (0) = 3 ,


dy
6x + = 0;
dt
dx
3 + x 2y = sin 2t:
dt
(Answer: x = sin 2t, y = 3 cos 2t.)

Problem 3.6.17 x (0) = 1 , y (0) = 4 ,


dx
+ 4x y = 3e2t ;
dt
dy
12x = 2e2t :
dt
2t
(Answer: x = (1 + t) e , y = (4 + 6t) e2t .)

Problem 3.6.18 x (0) = 0 , y (0) = 1 ,


dy
4x = 2e2t 16te2t ;
dt
dx
4x y = 8te2t 5e2t :
dt
2t
(Answer: x = 4te , y = e2t .)

Problem 3.6.19 x (0) = 1 , y (0) = 0 ,


dx
+ y = 0;
dt
dy
x = 0:
dt
(Answer: x = cos t, y = sin t.)
Chapter 4

The Fourier Series

4.1 Basics of the Fourier Series


De…nition. Let f (x) be a periodic function with period 2 . Its Fourier
series is the following in…nite series of functions:
a0 X
1
Ff (x) = + (an cos nx + bn sin nx)
2 n=1

whose coe¢ cients a0 ; an ; bn are computed as follows:

a0 = an = bn =

1
R 1
R 1
R
general case f (x) dx f (x) cos nx dx f (x) sin nx dx

2
R
f (x) is odd 0 0 0
f (x) sin nx dx

2
R 2
R
f (x) is even 0
f (x) dx 0
f (x) cos nx dx 0

Dirichlet Theorem Ff (x) = 12 [f (x ) + f (x+)] .

The following applet, for computing and drawing Fourier approximations


of periodic functions, is copied from the following cite on the Web:
http://www.jhu.edu/~signals/fourier2/index.html

109
CHAPTER 4. THE FOURIER SERIES 110

Figure 4.1: Fourier Approximation of Bipolar Pulse


CHAPTER 4. THE FOURIER SERIES 111

4.2 Classroom Exercises on Fourier Series


The following are periodic functions with period 2 .
Graph each function over three complete periods. Then
compute …ve nonzero terms in its Fourier series:
Problem 4.2.1 f (x) = jxj ; x<

This function is even. ) bn = 0 for all n.


R R h 2i
a0 = 2 0 f (x) dx = 2 0 x dx = 2 x2 = :
0
R R R
an = 2 0 f (x) cos nx dx = 2 0 x cos nx dx = 2 x
n
sin nx 1
n
sin nx dx
= 2 x
n
sin nx + 1
n2
cos nx 0
= 2
0+ 1
n2
( 1)n 0 1
n2
:
) an = 2
n2
[( 1)n 1] :
) a1 = 4
; a2 = 0; a3 = 9
4
; a4 = 0; a5 = 4
25
; a6 =
4
0; a7 = 49
:
a0
P
1
Ff (x) = 2
+ (an cos nx + bn sin nx)
n=1

) Ff (x) = 2
4
cos x 9
4
cos 3x 4
25
cos 5x 4
49
cos 7x .
CHAPTER 4. THE FOURIER SERIES 112

1 x<0
Problem 4.2.2 f (x) = ;
1 0 x<

f (x)

3 2 0 + +2 +3 x

This function is odd. ) a0 = 0; an = 0 for all n.


R R
bn = 2 0 f (x) sin nx dx = 2 0 sin nx dx = n 2 cos nx 0

= n
2
cos n + n
2
= n
2
( 1)n + n
2
) bn = n
2
( ( 1)n + 1) :

) b1 = 4 ; b2 = 0; b3 = 3
4
; b4 = 0; b5 = 5
4
;
4 4
b6 = 0; b7 = 7
; b8 = 0; b9 = 9
:
a0
P
1
Ff (x) = 2
+ (an cos nx + bn sin nx)
n=1

) Ff (x) = 4
sin x + 13 sin 3x + 15 sin 5x + 71 sin 7x + 91 sin 9x + .
CHAPTER 4. THE FOURIER SERIES 113

Problem 4.2.3 f (x) = x; x<

This function is odd.


) a0 = 0; and an = 0 for all n.
R
bn = 2 0 f (x) sin nx dx
R
= 2 0 x sin nx dx
R
= 2 x
n
cos nx + n1 cos nx dx
2 x 1
= n
cos nx + n2
sin nx 0
2 n
= n
( 1) + 0 0 0
= 2
n
( 1)n+1 :
) b1 = 2
1
; b2 = 2
2
; b3 = 23 ; b4 = 2
4
; b5 = 25 :
a0
P
1
Ff (x) = 2
+ (an cos nx + bn sin nx)
n=1

) Ff (x) = 2 sin x sin 2x + 32 sin 3x 1


2
sin 4x + 52 sin 5x .
CHAPTER 4. THE FOURIER SERIES 114

Problem 4.2.4 f (x) = x2 ; x<

f(x)

This function is even. ) bn = 0 for all n.


R R h 3i 2
a0 = 2 0 f (x) dx = 2 0 x2 dx = 2 x3 = 23 :
0
2
R 2
R 2
an = 0
f (x) cos nx dx = 0
x cos nx dx
h 2 i R
= n2x sin nx n
4
x sin nx dx
0
R
= 0 0 + n24 x cos nx 0 n24 cos nx dx

= n2
4
cos n 0 n3
4
sin nx 0
: ) an = 4
n2
( 1)n :

) a1 = 4; a2 = 1; a3 = 4
9
; a4 = 14 ; a5 = 4
25
:
a0
P
1
Ff (x) = 2
+ (an cos nx + bn sin nx)
n=1

) Ff (x) =
2 4
3
4 cos x + cos 2x 9
cos 3x + 14 cos 4x 4
25
cos 5x + .
Appendix A

Table of Famous Integrals

R 1
R p R
xr dx = xr+1 (r 6= 1) p dx = ln x + x2 a2 sec2 x dx = tan x
r+1 x2 a 2

R 1
R p R
dx = ln (Cx) p dx = ln x + x 2 + a2 csc2 x dx = cot x
x x2 +a2
R R R
ex dx = ex + C sin x dx = cos x + C sec x tan x dx = sec x
R dx 1 1 x
R R
x2 +a2
= a
tan a
+C cos x dx = sin x + C csc x cot x dx = csc x
R dx 1 x a
R R
x2 a 2
= 2a
ln x+a
+C sec x dx = ln jsec x + tan xj tan x dx = ln jsec xj
R 1
R R
p dx = sin x
+C csc x dx = ln jcsc x + cot xj cot x dx = ln jsin xj
a 2 x2 a

Trigonometric Formulas
sin x 1 2 tan x
tan x = cos x
sec x = cos x
tan 2x = 1 tan2 x

cos x 1 1 cot2 x 1
cot x = sin x
= tan x
csc x = sin x
cot 2x = 2 cot x

d
dx
sin x = cos x sin2 x + cos2 x = 1 sin x cos x = 21 sin 2x

d
dx
tan x = sec2 x sec2 x = 1 + tan2 x sin2 x = 12 (1 cos 2x)

d
dx
sec x = sec x tan x csc2 x = 1 + cot2 x cos2 x = 21 (1 + cos 2x)

cos 2x = cos2 x sin2 x = 1 2 sin2 x = 2 cos2 x 1

115
Appendix B

Assignments

116
APPENDIX B. ASSIGNMENTS 117

B.1 Assignment 1 on Separable and


Homogeneous Di¤erential Equations
Assignment 1 - Mathematics 3
Separable and Homogeneous Di¤erential Equations

Name: Registration No.:

Specialization: Date: / 20

Assignment Problem B.1.1 Solve the following di¤erential equation:


dy
(y + yx2 ) dx = y 2 + 1.
APPENDIX B. ASSIGNMENTS 118

Name: Registration No.:

Assignment Problem B.1.2 Solve the following initial-value problem:


dy
2y cos2 2x dx = e4y ; y 8 = 0:
APPENDIX B. ASSIGNMENTS 119

Name: Registration No.:

Assignment Problem B.1.3 Solve the following di¤erential equation:


dy
(2y 3 x 6x3 y) dx = 3y 4 12x2 y 2 + 9x4 :
APPENDIX B. ASSIGNMENTS 120

Name: Registration No.:

Assignment Problem B.1.4 Solve the following di¤erential equation:


dy
3x2 dx = 9x2 + 3xy + y 2 :
APPENDIX B. ASSIGNMENTS 121

Final Answer of Assignment Problem B.1.1:


Separable di¤erential equation
1
y 2 + 1 = Ce2 tan x :
Final Answer of Assignment Problem B.1.2:
Separable di¤erential equation
(4y + 1) e 4y = 5 4 tan 2x:
Final Answer of Assignment Problem B.1.3:
Homogeneous di¤erential equation
y 4 6x2 y 2 + 9x4 = C 2 x6 :
Final Answer of Assignment Problem B.1.4:
Homogeneous di¤erential equation
y = 3x tan (ln (Cx)) :
APPENDIX B. ASSIGNMENTS 122

B.2 Assignment 2 on Exact Di¤erential


Equations
Assignment 2 - Mathematics 3
Exact Di¤erential Equations

Name: Registration No.:

Specialization: Date: / 20

Assignment Problem B.2.1 Solve the following di¤erential equation:


(y 3 y 2 sin x 6e3x ) dx + (3xy 2 + 2y cos x + tan y) dy = 0:
APPENDIX B. ASSIGNMENTS 123

Name: Registration No.:

Assignment Problem B.2.2 Solve the following di¤erential equation:


dy
( sin x csc2 y + 3y 2 ) dx + x3 + cos x cot y = 0:
APPENDIX B. ASSIGNMENTS 124

Name: Registration No.:

Assignment Problem B.2.3 Solve the following …rst-order initial-value prob-


lem:
dy
cos x ln y + sec2 x + y1 sin x + p 1 dx
= 0 ; y 4 = 1:
1 y2
APPENDIX B. ASSIGNMENTS 125

Name: Registration No.:

Assignment Problem B.2.4 Solve the following …rst-order initial-value prob-


lem:
2x 3x
y + 1+x 2 + 3e cos y dx + (x + sec2 y e3x sin y) dy = 0:
y (0) = 0:
APPENDIX B. ASSIGNMENTS 126

Final Answer of Assignment Problem B.2.1:


Exact di¤erential equation,
(x; y (x)) = xy 3 + y 2 cos x 2e3x + ln (sec y) = C:
Final Answer of Assignment Problem B.2.2:
Exact di¤erential equation,
(x; y (x)) = sin x cot y + y 3 + 3 ln x = C:
Final Answer of Assignment Problem B.2.3:
Exact di¤erential equation,
(x; y (x)) = sin x ln y + tan x + sin 1 y = 1 + 2 :
Final Answer of Assignment Problem B.2.4:
Exact di¤erential equation,
(x; y (x)) = xy + tan y + e3x cos y + ln (x2 + 1) = 1:
APPENDIX B. ASSIGNMENTS 127

B.3 Assignment 3 on Linear Di¤erential


Equations
Assignment 3 - Mathematics 3
Linear Di¤erential Equations

Name: Registration No.:

Specialization: Date: / 20

Assignment Problem B.3.1 Solve the following di¤erential equation:


dy
x dx + 4y = x3 x:
APPENDIX B. ASSIGNMENTS 128

Name: Registration No.:

Assignment Problem B.3.2 Solve the following di¤erential equation:


dy
dx
+ 2 (tan 2x) y = 3e3x cos 2x
APPENDIX B. ASSIGNMENTS 129

Name: Registration No.:

Assignment Problem B.3.3 Solve the following …rst-order initial-value prob-


lem:
dy
dx
+ (cos x) y = 4x3 e sin x ; y (0) = 8:
APPENDIX B. ASSIGNMENTS 130

Name: Registration No.:

Assignment Problem B.3.4 Solve the following …rst-order initial-value prob-


lem:
dy 1
dx
+ x
+ cot x y = 2x cos x:
APPENDIX B. ASSIGNMENTS 131

Final Answer of Assignment Problem B.3.1:


Linear di¤erential equation,
y = Cx 4 + 17 x3 51 x:
Final Answer of Assignment Problem B.3.2:
Linear di¤erential equation,
y = cos 2x (C + e3x ) :
Final Answer of Assignment Problem B.3.3:
Linear di¤erential equation,
y = cos x (6 + 2e5x ) :
Final Answer of Assignment Problem B.3.4:
Linear di¤erential equation,
y = Cx csc x + x sin x:
APPENDIX B. ASSIGNMENTS 132

B.4 Assignment 4 on Bernoulli Di¤erential


Equations
Assignment 4 - Mathematics 3
Bernoulli Di¤erential Equations

Name: Registration No.:

Specialization: Date: / 20

dy
Assignment Problem B.4.1 dx
+ x2 y = (x4 sec2 x) y 3 :
APPENDIX B. ASSIGNMENTS 133

Name: Registration No.:

Assignment Problem B.4.2 Solve the following di¤erential equation:


dy 1
dx 4
(tan x) y = (sin x) y 3 :
APPENDIX B. ASSIGNMENTS 134

Name: Registration No.:

Assignment Problem B.4.3 Solve the following di¤erential equation:


dy
6 dx + y cos x = y 5 cos x:
APPENDIX B. ASSIGNMENTS 135

Name: Registration No.:

Assignment Problem B.4.4 Solve the following di¤erential equation:


dy
dx
= y (3xy 3 1) :
APPENDIX B. ASSIGNMENTS 136

Final Answer of Assignment Problem B.4.1:


Bernoulli di¤erential equation,
1
y2
= x4 (C 2 tan x) ) y = x2 pC 12 tan x :
Final Answer of Assignment Problem B.4.2:
Bernoulli di¤erential equation,
y 4 = sec x C + 2 sin2 x :
Final Answer of Assignment Problem B.4.3:
Bernoulli di¤erential
p equation,
6 sin x:
y= 1 + Ce
Final Answer of Assignment Problem B.4.4:
Bernoulli di¤erential equation,
1
y= p 3
Ce3x +3x+1
:
APPENDIX B. ASSIGNMENTS 137

B.5 Assignment 5 on Method of Variation of


Parameters
Assignment 5 - Mathematics 3
Method of Variation of Parameters

Name: Registration No.:

Specialization: Date: / 20

Assignment Problem B.5.1 Verify that the function yh (x) = C1 x + C2 x2


is the homogeneous solution of the following linear di¤erential equation:
x2 y 00 2xy 0 + 2y = 5x2 .
Then use this yh (x) to …nd the its general solution, by means of the
Method of Variation of Parameters.
APPENDIX B. ASSIGNMENTS 138

Name: Registration No.:

Assignment Problem B.5.2 Find the general solution of the following


di¤erential equation, using the Method of Variation of Parameters:
y 00 5y 0 + 4y = 18xex :
APPENDIX B. ASSIGNMENTS 139

Name: Registration No.:

Assignment Problem B.5.3 Find the general solution of the following


di¤erential equation, using the Method of Variation of Parameters:
y 00 + 4y 0 + 4y = e 2x sec2 x.
APPENDIX B. ASSIGNMENTS 140

Name: Registration No.:

Assignment Problem B.5.4 Find the general solution of the following


di¤erential equation, using the Method of Variation of Parameters:
y 00 2y 0 + y = 6ex 24x2 ex .
APPENDIX B. ASSIGNMENTS 141

Final Answer of Assignment Problem B.5.1


yG:S: (x) = (C2 + 5 ln x) x2 + C1 x:
Final Answer of Assignment Problem B.5.2
yG:S: = y = (3x2 + 2x) ex + C1 ex + C2 e4x :
Final Answer of Assignment Problem B.5.3
2x
yG:S: = (ln (sec x) + C1 + C2 x) e :
Final Answer of Assignment Problem B.5.4
yG:S: = (3x2 2x4 + C1 + C2 x) ex :
APPENDIX B. ASSIGNMENTS 142

B.6 Assignment 6 on Method of Undetermined


Coe¢ cients
Assignment 6 - Mathematics 3
Method of Undetermined Coe¢ cients

Name: Registration No.:

Specialization: Date: / 20

Assignment Problem B.6.1 Find the general solution of the following second-
order linear di¤erential equation, using the Method of Undetermined
Coe¢ cients:
y 00 + 4y 0 + 5y = 4ex 10xex :
APPENDIX B. ASSIGNMENTS 143

Name: Registration No.:

Assignment Problem B.6.2 Find the general solution of the following second-
order linear di¤erential equation, using the Method of Undetermined
Coe¢ cients:
y 00 2y 0 8y = 6e 2x :
APPENDIX B. ASSIGNMENTS 144

Name: Registration No.:

Assignment Problem B.6.3 Find the general solution of the following second-
order linear di¤erential equation, using the Method of Undetermined
Coe¢ cients:
y 00 + 6y 0 + 45y = (37x 35) e 2x :
APPENDIX B. ASSIGNMENTS 145

Name: Registration No.:

Assignment Problem B.6.4 Find the general solution of the following second-
order linear di¤erential equation, using the Method of Undetermined
Coe¢ cients:
y 00 3y 0 18y = 36 sin 3x 108 cos 3x:
APPENDIX B. ASSIGNMENTS 146

Name: Registration No.:

Assignment Problem B.6.5 Find the general solution of the following second-
order linear di¤erential equation, using the Method of Undetermined
Coe¢ cients:
y 00 2y 0 + 5y = 15x2 12x 29:
APPENDIX B. ASSIGNMENTS 147

Final Answer of Assignment Problem B.6.1:


yG:S: = yp + yh = ex xex + e 2x (C1 sin x + C2 cos x) :
Final Answer of Assignment Problem B.6.2:
yG:S: = yp + yh = xe 2x + C1 e 2x + C2 e4x :
Final Answer of Assignment Problem B.6.3:
yG:S: = yp + yh = (x 1) e 2x + C1 e 3x sin 6x + C2 e 3x
cos 6x:
Final Answer of Assignment Problem B.6.4:
yG:S: = yp + yh = 4 cos 3x + C1 e 3x + C2 e6x :
Final Answer of Assignment Problem B.6.5:
yG:S: = yp + yh = 3x2 7 + C1 ex cos 2x + C2 ex sin 2x:
APPENDIX B. ASSIGNMENTS 148

B.7 Assignment 7 on Cauchy-Euler


Di¤erential Equations
Assignment 7 - Mathematics 3
Cauchy-Euler Di¤erential Equations

Name: Registration No.:

Specialization: Date: / 20

Assignment Problem B.7.1 Find the general solution of the following


di¤erential equation:
x2 y 00 2xy 0 + 2y = 5x2 .
APPENDIX B. ASSIGNMENTS 149

Name: Registration No.:

Assignment Problem B.7.2 Find the general solution of the following


di¤erential equation:
x2 y 00 + 6xy 0 14y = 27x2 .
APPENDIX B. ASSIGNMENTS 150

Name: Registration No.:

Assignment Problem B.7.3 Find the general solution of the following


di¤erential equation:
3x2 y" + 11xy 0 + 5y = 352x 9
APPENDIX B. ASSIGNMENTS 151

Final Answer of Assignment Problem B.7.1


This is a Cauchy-Euler di¤erential equation,
yG:S: (x) = 5x2 ln x + C1 x + C2 x2 :
Final Answer of Assignment Problem B.7.2
This is a Cauchy-Euler di¤erential equation,
yG:S: (x) = 3x2 ln x + C1 x2 + C2 x 7 :
Final Answer of Assignment Problem B.7.3
This is a Cauchy-Euler di¤erential equation,
9 1 5=3
yG:S: (x) = 2x + C1 x + C2 x :
APPENDIX B. ASSIGNMENTS 152

B.8 Assignment 8 on Direct Laplace


Transform
Assignment 8 - Mathematics 3
Direct Laplace Transform

Name: Registration No.:

Specialization: Date: / 20

Assignment Problem B.8.1 Find the Laplace Transforms of the following


functions:

1. q (t) = e6t (t cos 3t)

2. g (t) = t (e6t cos 3t)

3. w (t) = t e6t cos 3t

4. k (t) = te6t cos 3t

5. p (t) = (t 2) e6(t 2)
cos 3 (t 2) U (t 2)

d
6. h (t) = dt
(te6t cos 3t)
Rt
7. m (t) = 0
z e6z cos 3z dz

8. v (t) = cosh 4t cos 3t + cosh 4t cos 3t:


APPENDIX B. ASSIGNMENTS 153

Final Answer of Assignment Problem B.8.1:


s2 9
1. L (q) = (s 6)(s2 +9)2
:
s 6
2. L (g) = s2 ((s 6)2 +9)
:

1
3. L (w) = s(s 6)(s2 +9)
:
s2 12s+27
4. L (k) = (s2 12s+45)2
:

s2 12s+27
5. L (p) = (s2 12s+45)2
e 2s :

s(s2 12s+27)
6. L (h) = (s2 12s+45)2
:

s2 12s+27
7. L (m) = s(s2 12s+45)2
:

s 4 s+4 s2
8. L (v) = 2((s 4)2 +9)
+ 2((s+4)2 +9)
+ (s2 16)(s2 +9)
:
APPENDIX B. ASSIGNMENTS 154

B.9 Assignment 9 on Inverse Laplace


Transform
Assignment 9 - Mathematics 3
Inverse Laplace Transform

Name: Registration No.:

Specialization: Date: / 20

Assignment Problem B.9.1 Find the Inverse Laplace Transforms of the


functions F; G; K:
5s 44
1. F (s) = (s 6)2 (s+1)
e 3s :
s+7
2. G (s) = ln s 1
:
s+3
3. K (s) = (s+3)2 +4
e 6s :
APPENDIX B. ASSIGNMENTS 155

Name: Registration No.:

Assignment Problem B.9.2 Find the Inverse Laplace Transforms of the


functions F; G; K:

1. F (s) = 7e 3s
+ 3s e 4s
+ 10
s2 25
e 6s 12
(s+6)4
e 8s :
1 s
2. G (s) = cot 4
:
s 6
3. K (s) = s2 12s+40
e 7s :
APPENDIX B. ASSIGNMENTS 156

Name: Registration No.:

Assignment Problem B.9.3 Find the Inverse Laplace Transforms of the


functions F; G; K:
2s2 13s 18
1. F (s) = (s 2)(s2 +2s+10)
e 5s :
s 4
2. G (s) = s2 +4s+13
e 7s :

3. K (s) = 3e 2s
+ 4s e 5s
+ s
s2 4
e 3s :
APPENDIX B. ASSIGNMENTS 157

Final Answer of Assignment Problem B.9.1:


1
1. L (F ) = e6(t 3)
2 (t 3) e6(t 3)
e (t 3)
U (t 3) :
1 et e 7t
2. L (G) = t
:
1 3(t 6)
3. L (K) = e cos 2 (t 6) U (t 6) :

Final Answer of Assignment Problem B.9.2:


1
1. L (F ) =
7 (t 3)+3U (t 4)+2 sinh 5 (t 6) U (t 6) 2 (t 8)3 e 6(t 8)
U (t 8) :
1 sin 4t
2. L (G) = t
:
1
3. L (K) = e6(t 7)
cos 2 (t 7) U (t 7) :

Final Answer of Assignment Problem B.9.3:


1 (t 5)
1. L (F ) = e (sin 3 (t 5) + 4 cos 3 (t 5)) 2e2(t 5)
U (t 5) :
1 2(t 7)
2. L (G) = e [cos 3 (t 7) 2 sin 3 (t 7)] U (t 7) :
1
3. L (K) = 3 (t 2) + 4U (t 5) + cosh 2 (t 3) U (t 3) :
APPENDIX B. ASSIGNMENTS 158

B.10 Assignment 10 on Solution of Fredholm


Integral Equations
Assignment 10 - Mathematics 3
Solution of Fredholm Integral Equations by Laplace Transform

Name: Registration No.:

Specialization: Date: / 20

Assignment Problem B.10.1 Use the Convolution Theorem to solve the


following integral equation:
Z t
f (t) = cos 3t sin 3t + 6 f (z) cos 3 (t z) dz:
0
APPENDIX B. ASSIGNMENTS 159

Name: Registration No.:

Assignment Problem B.10.2 Use the Convolution Theorem to solve the


following integral equation:
Z t
2t
f (t) = 2 sin 2t + e 4 f (z) cos 2 (t z) dz:
0
APPENDIX B. ASSIGNMENTS 160

Name: Registration No.:

Assignment Problem B.10.3 Use the Convolution Theorem to solve the


following integral equation:
Zt
f (t) = 3 sin 5t 8 f (z) cos 5 (t z) dz:
0
APPENDIX B. ASSIGNMENTS 161

Name: Registration No.:

Assignment Problem B.10.4 Use the Convolution Theorem to solve the


following integral equation:
Zt
f (t) = cosh 3t 3 f (z) e3(t z)
dz:
0
APPENDIX B. ASSIGNMENTS 162

Final Answer of Assignment Problem B.10.1:


f (t) = e3t :
Final Answer of Assignment Problem B.10.2:
f (t) = e 2t (1 + 4t2 ) :
Final Answer of Assignment Problem B.10.3:
f (t) = 5e 4t sin 3t:
Final Answer of Assignment Problem B.10.4:
f (t) = e 3t :
APPENDIX B. ASSIGNMENTS 163

B.11 Assignment 11 on Laplace Transform


and Initial-value Problems (1)
Assignment 11 - Mathematics 3
Laplace Transform & Initial-value Problems (1)

Name: Registration No.:

Specialization: Date: / 20

Assignment Problem B.11.1 Use the Laplace Transform to solve the


following initial-value problem:
y 00 + 4y 0 + 5y = 10et ; y (0) = 0; y 0 (0) = 5:
APPENDIX B. ASSIGNMENTS 164

Name: Registration No.:

Assignment Problem B.11.2 Use the Laplace Transform to solve the


following initial-value problem:
y 00 2y 0 8y = 6e 2t ; y (0) = 0; y 0 (0) = 1:
APPENDIX B. ASSIGNMENTS 165

Name: Registration No.:

Assignment Problem B.11.3 Use the Laplace Transform to solve the


following initial-value problem:
y 00 3y 0 + 2y = e2t , y (0) = 5 , y 0 (0) = 10
APPENDIX B. ASSIGNMENTS 166

Name: Registration No.:

Assignment Problem B.11.4 Use the Laplace Transform to solve the


following initial-value problem:
y 00 8y 0 + 16y = 6te4t ; y (0) = 1; y 0 (0) = 4:
APPENDIX B. ASSIGNMENTS 167

Final Answer of Assignment Problem B.11.1:


y (t) = et + e 2t (2 sin t cos t)
Final Answer of Assignment Problem B.11.2:
y (t) = te 2t :
Final Answer of Assignment Problem B.11.3:
y = et + 4e2t + te2t
Final Answer of Assignment Problem B.11.4:
y = e3t + te4t
APPENDIX B. ASSIGNMENTS 168

B.12 Assignment 12 on Laplace Transform


and Initial-value Problems (2)
Assignment 12 - Mathematics 3
Laplace Transform & Initial-value Problems (2)

Name: Registration No.:

Specialization: Date: / 20

Assignment Problem B.12.1 Use the Laplace Transform to solve the


following initial-value problem (system of di¤erential equations):

dx
3x + 2y = 0;
dt
dx dy
3 13x + 2 = 0;
dt dt
x (0) = 1 , y (0) = 0:
APPENDIX B. ASSIGNMENTS 169

Name: Registration No.:

Assignment Problem B.12.2 Use the Laplace Transform to solve the


following initial-value problem (system of di¤erential equations):

dx
4x + 2y = 2te4t ;
dt
dy
x+3 12y = 0:
dt
x (0) = 3 , y (0) = 0 .
APPENDIX B. ASSIGNMENTS 170

Name: Registration No.:

Assignment Problem B.12.3 Use the Laplace Transform to solve the


following initial-value problem (system of di¤erential equations):

dy 2t
8x = 8te ;
dt
dx 2t
2x y = 2te ;
dt
x (0) = 0 , y (0) = 0:
APPENDIX B. ASSIGNMENTS 171

Name: Registration No.:

Assignment Problem B.12.4 Use the Laplace Transform to solve the


following initial-value problem (system of di¤erential equations):

dx dy
+ 4 + 4y = 3e t ;
dt dt
dy
x = te t ;
dt
x (0) = 1 , y (0) = 0 .
APPENDIX B. ASSIGNMENTS 172

Final Answer of Assignment Problem B.12.1:


x (t) = e3t cos 2t y (t) = e3t sin 2t .
Final Answer of Assignment Problem B.12.2:
x (t) = 3e4t , y (t) = te4t .
Final Answer of Assignment Problem B.12.3:
x (t) = t2 e 2t , y (t) = 4t2 e 2t .
Final Answer of Assignment Problem B.12.4:
x=e t, y = te t .

You might also like